You are on page 1of 153

F irs t Edition; 2 0 1 5

Edited by
Mohsen El-Darawy
Professor o f Internal Medicine
Head o f In ternal Medicine Department
Faculty O f Medicine, Zagazig University

Emad AbdEI-lateef
Professor of Internal Medicine
Head of Gastroenterology & Hepatology unit

Ayman Ramadan
Assistant Professor of Internal Medicine

Don't w alk behind me, I ma\ not lead.


Don’ t w alk ahead of me, I ma\ not follow.
W alk next to me and be m \ friend.’*
Albert Camus
Dedication
l ’d like to dedicate this book to all
my sons and future doctors to help
them in understanding internal
medicine and train them on how
to answer multiple choice
questions
Contents
Questions page Answers Page
1 Cardiovascular Diseases 5 1 Cardiovascular Diseases 52
Hypertension 5 Hypertension 52
Coronary Artery Diseases 7 Coronary Artery Diseases 56 1
Arrythnnas 9 Arrythmias -11—
Valvular heart diseases 11 Valvular heart diseases 65 |
Heart Failure 13 Heart Failure 68
Rheumatic Fever A Infective 14 Rheumatic Fever & Infective 73
Endocarditis Endocarditis
Cardiomyopathy 16 Cardiomyopathy 75
Pericarditis 16 Pericarditis 77
Aortic Aneurysm 17 Aortic Aneurysm 77
Pulmonary Embolism and 18 Pulmonary Embolism and 79
Anticoagulatioo Anticoagulation
Miscellaneous 18 Miscellaneous 81
2 Pulmonary and critical care 20 2 Pulmonary and critical care 83
medicine medicine
Normal Physiology and Function 20 Normal Physiology and Function 83
COPD A Asthma A Respiratory 20 COPD A Asthma A Respiratory 85
Failure Failure
Respiratory Infections 23 Respiratory Infections 91
Intestitial Lung Diseases 24 Intestitial Lung Diseases 94
Lung Turnon 25 Lung Tumors 96
Hemoptysis 26 Hemoptysis 97
Pleural Diseases 27 Pleural Diseases 99
Miscellaneous 27 Miscellaneous 100
3 Gastroenterology and 29 3 Gastroenterology and Disorders 102
Disorders Of The pancreas And Of The pancreas And Liver
Liver 1
Liver functions 29 Liver functions r~ l 0 2

Hepatitis 30 Hepatitis 104


Cirrhosis and Portal Hypertension 31 Cirrhosis and Portal Hypertension 109
Hcpatocclhiar carcinoma 35 Hepatocelluar carcinoma 116
Esophagus and Stomach Disorders 35 Esophagus and Stomach Disorders 117
Inflammatory Bowel Diseases 38 Inflammatory Bowel Diseases 121
Malabsorption 39 Malabsorption 124
Diarrhea 41 Diarrhea ! 127 1
Gastrointestinal Tumors 42 Gastrointestinal Tumors 130
Pancreatic Diseases 43 Pancreatic Diseases 132 ;
Miscellaneous 43 Miscellaneous 134
4 Nephrology 45 4 Nephrology 135
Glomerulopathy • 45 Glomerulopathy 135
Renal Failure 46 Renal Failure 140 j
Miscellaneous 49 Miscellaneous 145 !
I CARDIOVASCULAR DISKASL
Hypertension & Hypotension
1. Which of the following tests is C. Diuretic
considered routine (recommended) D Calcium Channel Blocker
in the initial evaluation o f a patient E u-Blockcr
with hypertension?
A Chest x-ray 5. An ankle-brachial index of
B. TSH is considered normal.
C Uric acid level A. 0.95
D. 24-hour unne protein B 0.75
EECG C 0 50
D. 0.25
2. Which of the following is E. 0.15
considered a first-line medication
in the treatment of hypertension? 6. A 62-ycar-old business executive
A. Chlorthalidone w ith a history of migraines is noted
B. Lisinopril to have hypertension; otherwise, he
C. Clonidine is healthy. Which of the following is
D. Losanan the best medication for the
E. Amlodipine treatment of his hypertension?
A. (3-BIocker
3. Which of the following B. ARB
statements about systolic C. a-Blocker
hypertension is true? D. ACE inhibitor
A. It represents relatively little risk to E. Thiazide diuretic
the patient
B. it is defined as a systolic pressure 7. Which of the following
>140 mm Hg with a diastolic statements about orthostatic
pressure > 100 mm Hg. hypotension is true?
C. It does not increase the risk of A. It is a decrease in systolic blood
stroke. pressure that occurs when moving
D. It is often caused by mitral from a standing to a sitting position.
regurgitation. B. It is commonly associated with a
E. It is more dangerous to elderly decrease in pulse rate.
patients than an elevated diastolic C. In some cases, it is associated with
pressure. autonomic dysfunction (as seen in
diabetic patients). ^
4. Which of the following is the D. It is rarely associated with
most appropriate medication for symptoms.
the treatment of hypertension in a E. The condition results from volume
diabetic patient? overload.
A. p-Blocker
B. ACE inhibitor
8. Which of the following is recently become pregnant and
associated with rebound intends to keep the baby. The most
hypertension? appropriate first-line therapy is:
A. Oral contraceptives A ACE-inhibitor
B. Hyperthyroidism B Angiotensin 11 receptor blocker
C. Excessive alcohol consumption C. Calcium channel blocker
D Abrupt withdrawal of (^-blockers D. Diuretics
E. Pheochromocytoma E ^-blocker

9. Id the elderly, a rise in the 13. Captopril:


systolic blood pressure with no A Decreases plasma renin activity
change in the diastolic blood B. Increases degradation of
pressure most likely suggests circulating brady kinin.
A. Anemia C Increases formation of angiotensin
B. Thyrotoxicosis II
C. Aortic insufficiency D Cannot be safely used in
D. Stiffening o f the arteries combination with a beta-blocking
E. None of the above agent.
E. Is contraindicated in patients with
10. Causes for orthostatic bilateral renal artery stenosis.
hypotension in the elderly include
all of the following except 14. Which of the following may be
A. Declining haroreceptor sensitivity responsible for a hypokalaeniic
B. Decreased arterial compliance hypertension?
C. Increased venous tortuosity A. Non-classical congenital adrenal
D. Decreased renal sodium hyperplasia
conservation B. Barter's syndrome
E. Increased plasma volume C. Diabetic nephropathy
D Liddlc’s syndrome
11. Each of the following a one of E Type IV renal tubular acidosis
the actions of hydrochlorthiazide
except for one. Which among those 15. A 45 year old hypertensive
listed does not apply to female presents with a two day
hydrochlorthiazidc (hetz)? history o f fever. On examination
A. Distal loop diuresis she has a facial rash and slight
B. Reduction of insulin resistance swelling with tenderness o f the
C. Kaliuresis (excretion of potassium) w rist and ankle joints. Which of the
D. Natriuresis (excretion of following anti-hypertensives may
potassium) be responsible for her
E. Preservation of bone calcium presentation?
A. Minoxidil
12. A 34-year-old diabetic woman B. Phcnox> benzanune
present with blood pressure 155/93 C. Hydralla/ine
ininHg. She informs you she has D. Alpha-meihvldopa
C o ro n a ry A rtery Diseases 19. A 58-ycar-old man prcscuts to
your office 4 weeks after being
16. Which of the following hospitalized for Ml. He is
statements are true o f coronary complaining of chest pain, fever,
artery anatomy? and multiple joint pain. Laboratory
A. RBBB m acute anterior Ml tests do not show an increase in
suggests obstruction prior to the first cardiac enzymes. The most likely
septal branch of the left anterior diagnosis is
descending coronary artery A Dressier’s syndrome
B. The posterior descending artery is B. Costochondritis
usually a branch of the circumflex C. Meigs' syndrome
artery D. Recurrent MI
C. A branch of the right coronary' in E. Pneumonia
over 90®o of subjects supplies the
sinus node. 20. Which of the following
D. The AV node is supplied by the medications would improve
left anterior descending coronary survival following an Ml?
artery. A. Metoprolol
E. The left main stem is about 4 cm B. Hydrochlorothiazide
long C. Warfarin
D. Nitroglycerin
17. Which of the following factors E. Morphine
is included in the criteria for
administering streptokinase with 21. Which of the following
MI? sequences represents how a typical
A. Cardiogenic chest pain lasting al anteroseptal MI progresses on
least 6 hours ECG?
B. ECG changes of at least 1 to 2 mm A. Q-wave development, peaked T-
of ST elevation in two adjacent waves, ST-segment elevation, T-
precordial leads wave inversion
C. Streptokinase should not be B. T-wave inversion, Q-wave
administered 6 hours after the onset development, ST-segment elevation,
of chest pain peaked T-waves
D. Q waves noted in the lateral C. Peaked T-waves, ST-segment
precordial leads elevation, Q-wave development, T-
wave inversion
18. Which o f the following is NOT D. Peaked T-waves, Q-wavc
considered a risk factor for M l? development. ST-segment elevation,
A. Alcoholism T-wave inversion
B. Homocystinemia E. ST-segment elevation, T-wave
C. Diabetes Mellitus inversion. Q-wave development,
D. Male sex peaked T-waves
26. What is the most important
22. Which of the following values is clinical complication of
acceptable for a 65-year-old man nitroglycerin use?
who had a previous coronary A Headache
artery bypass graft? B. Persistent hypotension
A. Total cholesterol of 215 mg per dL C. Methemoglobinemia
B. HDL cholesterol o f 32 mg per dL D Thiocyanate toxicity
C. LDL cholesterol o f 68 mg per dL E. Transient hypotension
D. Triglycerides o f 228 mg per dL
E. Blood glucose of 130 mg per dL 27. The left anterior descending
coronary artery usually’ supplies:
23. Which of the fallowing A The anterior wall o f the left
statements about angina pectoris is ventricle and the a in o-ventricular
true? code
A. It typically lasts l to 2 hoirs B. The anterior wall o f the left
B. It may be associated with ventricle and the inter-ventricular
epigastric pain. septum
C. It causes predictable ECG C. The anterior wall of the left
changes. ventricle, airio-vcntricular node and
D. It is typically associated with chest the intcr-ventricular septum
wall tenderness. D The inter-ventncular septum and
E. It rarely radiates to the neck, jaw , the tnferior part of the left ventricle
or left arm. E The sino-atrial node, the atrio­
ventricular node and the inferior pan
24. Cardiac troponins may remain of the left ventricle
elevated up to ------ after acute
coronary syndrome 28. A 55-year-old man has just
A. 24 hours arrived in emergency complaining
B. 48 hours of 20 minutes of central crushing
C. 72 hours chest pain. Which feature is most
D. 1 week indicative of myocardial infarction
E. 2 weeks at this moment in time?
A. Inverted T waves
25. Which of the fallowing B. S T depression
symptoms is more likely to C. ST elevation
represent myocardial ischemia in D. Q waves
older patients? E. Raised troponin
A. Chest pain
B. Dyspnea 29. Which of the following is a
C. Diaphoresis characteristic feature of angina?
D. Back pain A Retrosternal chest para
E Jaw pain B. Pain that is worse with arm
movement
C A split second heart sound
D SI 'icgmcnl elevation on the 33. Which of the following is true
resting I CG regarding the action of
E. A tender chest wail Clopidogrel?
A. It inhibits cyclo-oxygcnase
30. ST elevation on ECG in leads B. It is an ADP receptor antagonist
11, 111 and aVF alone: C. It is a glycoprotein llh/llla
A Indicates anterior myocardial inhibitor
infarction D. It is a selective factor Xa uiliibitor
ti. Indicates inferior myocardial E. It is Hydroxymethyl Coenzyme A
ii\farction inhibitor
C. Could indicate acute pericarditis
D. Could be a trivial finding A r n th m ia s
E. Is normal in children
34. A 32-ycar-old man with a
31. Which of the following history of WolfT-Parkinson-White
statem ent regarding the use of syndrome (WPW) presents by
cardiac m arkers in the evaluation palpitation. He is alert and in no
of a patient with an acu te coronary acute distress. His blood pressure is
syndrom e is true: 130/70 mm Hg, pulse rate 220
A. Myoglobin levels are not affected beats/minute, and O2 saturation
by renal failure. 96%, His ECG is consistent with
B. A single troponin measurement on WPW. Which one of the follow ing
presentation can effectively rule out would be the treatment of choice?
myocardial infarction. A Verapamil (Calan)
C. Cardiac troponins are the best B. Adenosine (Adenocard)
markers for identifying myocardial C. Procainamide
cell injury. D. Digoxin
D. Total CPK levels are as specific as
cardiac troponins in identifying 35. Which of the following ECG
myocardial cell injury. findings is associated with sudden
E. CPK-MB fraction is not elevated cardiac death?
in cardiac myocarditis. A. Prolonged QT interval
B. First-degree AV block
32. T he benefits associated with the C. Sinus arrhythmia
use of nitroglycerin in patients with D. Right bundle branch block
acute coronary syndrom es result E. Premature ventricular contractions
prim arily from:
A. Pulmonary artery vasoconstriction. 36. The best medication to use in
B. Decreasing myocardial preload the emergent treatment of
C. Increasing afterload. supraventricular tachycardia is
D. Coronary vasoconstriction. A. Digoxin
E. Inotropic support. B. Verapamil
C. Adenosine
D. Diltiazem
T- Isoproterenol D They may represent a risk tor
sudden death in healthy patients
37. Indications for pacemaker E. Caffeine use is not associated with
placement include which of the PVCs.
following?
A. Asymptomatic bradyarrhythmias 41. A 67-year-old man presents
D. Mobitz II AV block with an irregular heart beat with
C. Atrial fibrillation an overall rate of 140 bpon and
D. Atrial flutter ECG reveals atrial fibrillation.
E. Mobitz 1 AV block Which o f the following would you
expect to sec when assessing the
38. A hemodvnamically unstable JVP?
patient is noted to have A Raised JVP with normal
supraventricular tachycardia. The waveform
most appropriate treatment is B Large kv waves’
A. Electrical synchronized C. Cannon ‘a waves’
cardioversion D Absent ‘a waves’
B. Carotid massage E Large ‘a waves’
C. Adenosine
D. Verapamil 42 Regarding atrial fibrillation:
E. Digoxin A- It is a risk factor for ischaemic
heart disease
39. Which o f the following can B. It is associated with
distinguish atrial flutter from sinus hypothyroidism
tachycardia? C. b-blockers are an ineffective
A. Carotid sinus massage treatment
B. Administration o f diltiazem D. It is reversible with digoxin
C. Administration o f isoproterenol E. It can be caused by pneumonia
D. Temporal artery massage
E. Administration of adenosine 43. A 56-year-old man presents
with ECG which reveals that the
40. Which of the following patient has first degree heart block.
statements about premature Which of the following ECG
ventricular complexes (PVCs) is abnormalities is typically seen in
correct? first degree heart block?
A. They are oarrow A. PR interval >120 ms
electrocardiographic wave (QRS) B PR interval >300 ras
complexes dial arc preceded by P C PR interval <200 ms
waves. D. PR interval >200 ms
B. In most cases, they disappear with E. PR interval <120 ms
exercise.
C. They are treated with type IC 44 Which of the following
antiarrhythmics. arrhythmias is associated with a
regular heart rate?
A. Atrial fibrillation
*19. A 5 2-year-old woman
B. Ventricular fibrillation complains of syncope. Her heart
C First-degree heart block rate is ISts/min and her blood
D Mobitz 1 block pressure is 152/76 raniHg. She has
E Mobitz D block a history of WnlfT-Parkinson-Whifc
syndrome. Her ECG shows a wide
45. Bradyarrhythmias include complex irregular tachycardia with
which one o f the following? ventricular precxcitarion. The
A. WolfT-Parkinson-White syndrome medication of choice is:
B. Torsades dc pointes A. Adenosine
C. Atrial flutter B. Propranolol
D. Ventricular fibrillation C. Diltiazem
E. Wenckebach’s phenomenon D. Digoxin.
E. Procainamide
46. In patients newly diagnosed
with arterial fibrillation on digoxin 50. A 30-year-old man presented
therapy, which of the following with a history of transient loss of
electrolytes is most important to consciousness and palpitation. His
monitor? ECG showed ventricular
A. Serum sodium. tachycardia. Which one of the
B. Serum potassium. following treatments should be
C. Serum calcium. avoided?
D. Serum magnesium. A. Adenosine
E. None o f the above. B. Amiodarone
C. DC cardioversion
47. Which o f the following drugs D. Flccainidc
has both a treatment and E. Verapamil
diagnostic role in narrow com plei
tachycardias? 51. Which of the following
A. Atenolol. antiarrbythmjc drugs may be used
B. Amiodarone. in the treatmentof long Q T
C. Adenosine. syndrome?
D. Atorvastadn. A. Amiodarone
E. Amlodipine. B. Atenolol
C. Flccainidc
48. The medication of choice to D. Propofanone
treat a patient in torsades de E Sotaloi
pointes is:
A. Epinephrine.
Valvular heart diseases
B. Flccainidc.
C. Calcium gluconate
52. Which of the following findings
D. Magnesium sulfate
rharactcrizc(s) aortic stenosis?
f . Procainamide.
A. Lack o f pressure gradient across
the aortic valve
I' Angina, dyspnea and ssncopc 56. Apatient in acute cardiogenic
t N Diastolic murmur lh.it ladi-ues to shock presents with a new soft
the axilla holosystolic apical m urm ur that
D Palpitations radiates to the axilla. W hich of the
I- Crescendo murmur with following is the most likely
midsystolic click diagnosis?
A Acute aortic insufficiency
53. Which of the folio wing B Acute acm e stenosis
statements about Still's murmur is C Acute mitral insufficiency
true? . D. Acute tricuspid insufficiency
A. It is benign and resolves over tunc. F. Acute ventral septal defect
B. It is common in the elderly and
results from decreased ventncular 57. A 75-year-old w om an in the
compliance. pre-assessment clinic tells you she
C. It is associated with severe chronic has mitral stenosis. W hich o f the
aortic regurgitation. following is not a sign o f mitral
D. It should always be assessed with stenosis?
an echocardiogram. A. Bifid P-wave
E. CHF is usually coexistent. B Diastolic opening snap heart sound
C Double impulse apex beat
54. Mitral valve prolapse is D. Mid-diastolic murmur
associated with E. Peripheral cyanosis
A. Elderly, obese men
B. Diastolic click that disappears with 58. A patient is adm itted with
Valsalva’s maneuver pneumonia. A murmur is heard on
C. Chest pain, dyspnea, and syncope examination. What finding points
D. Rheumatic heart disease to m itral regurgitation?
E. Myxomatous transformation <of the A. M urm ur louder on inspiration
valve leaflet B. M urmur louder with patient in left
lateral position
55. A 75-year-old man presents C. M urmur louder over the right 2nd
with exertional dyspnea and intercostal space m idclavicular line
generalized weakness. On D. Corrigan’s sign
examination, you discover a fiigh- E Narrow pulse pressure
pitched, blow ing diastolic m urm ur
and a wide pulse pressure with 59. In clinic, a retired 62-year-old
bounding pulses. The most likely man presents with shortness of
diagnosis is breath on exertion. You find a
A. Aortic stenosis collapsing pulse and subsequent
B. Aortic insufficiency echocardiography confirm s aortic
C. Mitral stenosis regurgitation. W hich of (be
D. Mitral insufficiency following is NOT associated with
E. Coarctation o f the aorta aortic regurgitation?
A. Ankylosing spondylosis
B. Aortic dissection D Nifedipine
C Marfan syndrome E. Isoproterenol
D. Rheumatic fever
E. Systemic lupus erythematosus 64. Which of the following drugs
(SLE) used in the treatment of C IIF has
been shown to increase survival?
H eart Failure A. Digoxin
B. rurosenhde
60. l^Type natriuretic peptide C Hydralazine
(BNP) is used in the diagnosis of D. Enalapril
A. Pulmonary Embolism E. Isosorbide dinitratc
B. Congestive Heart Failure
C. Chronic renal failure 65. In which of the following
D. Asthma clinical situations is digoxin best
E. Acute coronary syndrome used?
A. CHF in the setting of diastolic
61. All the following medication dysfunction
can reduce the P-type natriuretic B. Idiopathic hypertrophic subaortic
peptide (BNP) level except stenosis
A. Spironolactone C. Recent MI with CHF
B. Lisinopril D. Supraventricular arrhythmia with
C. Valsarlin the development of CHF
D. Digoxin E. Emergent treatment of ventricular
fibrillation
62. Which of the following
combinations would be best utilized 66. Which of the following
in the treatment of diastolic heart medications should be avoided in
failure? stable patients with a history of
A. Calcium Channel Blocker + a- heart failure?
blocker A. Acetaminophen
B. P-Blocker + diuretic B. Angiotensin-converting enzyme
C. ACE inhibitor + a-blocker (ACE) inhibitors
D. Calcium Channel Blocker + C. Angiotensin receptor blockers
.Angiotensin Receptor Blocker D. Beta-blockers
E. Angiotensin Receptor Blocker + E. Nonsteroidal anti-inflammatory
Calcium Channel Blocker agents

63. Which of the following drugs is 67. Which of the following signs or
commonly used in the treatment of symptoms has the highest
CHF related to systolic specificity in the diagnosis of acute
dysfunction? heart failure?
A. Verapamil A. Dyspnea
B Diltiazcm B. Edema
C. Ramipril C. Orthopnea
D Kales E Improving myocardial coniractilily
l: Weight gain and slowing the heart rate in high-
output heart failure.
68. A 69-year-old man presents
with a range of signs and symptoms 72. A 55 year old man presents
that give the impression of heart with gynaecomastia while receiving
failure. Which of the following is treatment for Heart failure. Which
not a feature of heart failure? of the following drugs is most likely
A. Hepatomegaly to be the cause of his
B. Non-pitting oedema gynaecomastia?
C. Pulsus altemans A. Amiloride
D. Raised jugular venous pressure B. Frusemide
E. Tricuspid regurgitation C. Nifedipine
D. Ramipnl
69. Which o f these is a cause of E. Spironolactone
high-output heart failure?
A. Scurvy 73. A 67 year old man presents
B. Thyrotoxicosis with sudden onset atrial fibrillation
C. Aortic stenosis (ventricular rate o f 150/minute).
D. Constrictive pericarditis His serum creatinine concentration
E. Ventricular fibrillation was 2.5 mg/dL. W hat is the main
factor that determ ines the choice of
70. At rest, a patient with mild loading dose o f digoxin in this
right-sided heart failure will have: patient?
A. Low blood pressure. A. Absorption
B. Increased central venous pressure. B. Apparent volume o f distribution
C. Reduced cardiac output. C. Lipid solubility
D. Increased cardiac reserve. D. Plasma half-life
E. Increased left atrial pressure. E. Renal clearance

71. The most important effect of Rheumatic Fever & Infective


digitalis therapy in heart failure is: Endocarditis
A. Controlling the ventricular
response o f atrial fibrillation.
74. Which of the following is the
B. Controlling paroxysmal atrial
standard prophy laxis treatment for
tachycardia with block or accelerated
subacute bacterial endocarditis
junctional rhythms.
(SBE) for dental procedures in low-
C. Improving myocardial
risk adult patients?
contractility, thus attenuating
A Amoxicillin: 1 g given
symptoms o f congestive heart failure. intravenously at the time o f the
D. Improving myocardial
procedure
contractility to improve survival.
B. Amoxicillin. 2 g given orally 1
hour before the procedure
C Ampidllin: 2 g given A. Full blood count
intravenously plus gentamicin (1.5 B FCG
mg/kg intravenously) 30 minutes C AutoanUbody screen
before the procedure; dose repealed 8 D. Blood culture
hours after the procedure F Coronary angiography
D. Axnpicillin: 500 mg given orally 1
hour before the procedure and 250 77. A 50-y car-old roan undergoes a
mg given 6 hours after the procedure prosthetic aortic valve
E. None replacement. Following the
operation, he presents with fo cr,
75. Which of the following patients janeway lesions, splinter
meets the Modified Duke criteria haemorrhages and night sweats.
for having definite endocarditis? The most definitive investigation
A. One blood culture positive for a for prosthetic valve endocarditis is:
Gram-negative bacillus, pyrexia of A. Auscultation
38.5°C and splinter haemorrhages B. Transthoracic echocardiography
and Osier nodes C. T ransocsophageal
B. One blood culture positive for echocardiography
Streptococcus viridans and a new soft D. Chest x-ray
pansystolic murmur loudest at the E. ECG
apex
C. Pyrexia of 38.5°C, intravenous 78. Subacute bacterial endocarditis
drug abuse. Osier nodes and splinter is:
haemorrhages A. Always treated with valve
D. Two blood cultures positive for replacement
Strep, viridans and a new crcscendo- B. Commoner in intravenous drug
decresccndo ejection systolic murmur users
E. Two blood cultures positive for C. Treated with at least 6 weeks of
Strep, viridans, pyrexia o f 38.5°C, oral penicillin
splinter haemorrhages and Osier D. Commoner in people with diabetes
nodes, and new small cerebral E. A self-limiting illness
abscesses
79. Which of the following is not a
76. A 25-ycar-old woman with clinical finding associated with
known mitral valve prolapse infective endocarditis?
develops a low grade fever, malaise A. Osier’s nodes.
and night sweats within a couple of b. Retinal haemorrhages.
weeks o f a major dental procedure. c. Splinter haemorrhages.
Examination reveals a pulse rate of d. Clubbing.
110/minute, tender vasculitic e. Erythema nodosum.
lesions on the finger pulps and
microscopic hacmaturia. Which 80. Empirical antibiotic therapy for
investigation is most likely to infective endocarditis is:
provide a definitive diagnosis?
\ Flucloxacillin end A Eaxly diastolic sound t>0-120
bcnzylpenicillin. milliseconds after
B Bcnzvlpemcilhn and gemamyem. B Dip and plateau filling pattern
C Genlamycin and flucloxacillin. C. Jugular venous distention
D. Amoxicillin and metronidazole. D. Knssmaul sign
F Cefuroximc and flucloxacillin. E- Square root sign

81. Wbich of (he following is a 85. Which of the following is the


major criterion for the diagnosis of most common cause of restrictive
Rheumatic fever? cardiomyopathy?
A. Fever A. Amyloidosis
B. Raised ESR B. Endomyocardial fibrosis
C. Polyarthritis C. Hemachromatosis
D. Migratory erythema D. Idiopathic
5) Prolonged PR interval E. Sarcoidosis

82. A 30-year-old IV drug abuser 86. Which of these is a cause of


develops acute aortic regurgitation sudden cardiac death?
due to infective endocarditis. A. Hyponatraemia
W hich of the following is not B. Atnal fibrillation
typical of acute aortic C. Hypertrophic obstructive
regurgitation? cardiomyopathy
A. Increased cardiac output D. Sinus arrhythmia
B. Dccrcsccndo diastolic murmur E. Acute pericarditis
C. Hypotension
D. Mmitral valve pre-closure 87. A 40-year-old lady collapsed in
E. Peripheral vasodilatation asystole. She has no past cardiac
history o f note and has been
Cardiomyopathy generally fit and well recently.
Which o f (be following is the most
83. Which o f the following likely cause o f her arrest?
medications should be avoided in A. Pulmonary embolus.
patients with hypertrophic B. Hypertrophic obstructive
obstructive cardiomyopathy? cardiomyopathy.
A. Aspirin C. Acute myocardial infarction.
B. Digoxin D. Severe pneumonia.
C. Disopyramide E. Pneumothorax.
D. Acetaminophen
E. Atenolol Pericarditis

84. Which of the following can S8. Which o f the following


distinguish constrictive pericarditis regarding the anatomy o f the heart
from restrictive cardiomyopathy? is true?
A. The aortic valve is tricuspid
B. The ascending aorta is entirely 92. Which one of the following is
outside the pericardial sac. the greatest risk factor for
C. The left atnal appendage is abdominal aortic aneurysm
identified readily by transthoracic (AAA)?
echocardiography. A Cigarette smoking
D. The pulmonary trunk lies anterior B Diabetes mellttus
to the ascending aorta. C Hypertension
L. The right atrium is posterior to the D AIncan American race
left atrium. L female gender

89. A 45-year-old woman bad 93. A 65-year-old man presents to


recovered from pericarditis about a your office complaining of
year earlier. The chest x-ray shows abdominal pain. His vital signs arc
pericardial calcification. The stable. Exam ination reveals a
presumptive diagnosis is pulsatile mass in the midabdoincn.
constrictive pericarditis. Which of The most ap p ro p riate test is
the following physical signs would A Magnetic resonance imaging
be consistent with this? (MR1) of the abdomen
A. Increased jugular distention on B Laboratory tests, including
inspiration complete blood count, electrolytes,
B. Third heart sound and FSR
C. Fourth heart sound C Ultrasound examination of the
D. Rales at both lung bases abdomen
E. Loud first and second heart sounds D. Upper GI series
E. Barium enema
90. The chest X-ray is o f use in the
diagnosis of which one of the 94. A 46-year-old man develops
following conditions? sudden severe central chest pain
A. Pericardial effusion after lifting heavy cases. The pain
B. Third-degree heart block radiates to the back and both
C. Unstable angina shoulders but not to either arm. His
D. Ventricular tachycardia BP is 155/90 mmHg, pulse rate is
E. Bacterial endocarditis 92 beats/m inute and the ECG is
normal. He ls distressed and
91. Which of the following is not a sweaty. W hat would you consider
feature o f cardiac tamponade? the most likely diagnosis?
A. Brady card ia. A Pneumothorax
B. Pulsus paradoxus. B Ml
C. Hypotension. C. Pulmonary embolism
D. Raised jugular venous pressure D. Aortic dissection
E. Diminished heart sounds. E Musculoskeletal pain

Aortic Aneurysm 95 Causes o f arte ria l aneurysms


include which o f the following?
A Polycythacmia 99. In diagnosing a Pulmonary
B. Raynaud’s disease Embolism, which of the following
C Budd-Chiari syndrome tests is considered the diagnostic
D. Syphilis test of choice?
L Subarachnoid haemonrhage A. Ventilation-perfusion (V/Q) lung
scan
96. The most appropriate B. Venous com pression
diagnostic investigation in a patient ultrasonograph) of the legs
presenting with chest pain and a C. Pulmonary angiography
widened mediastinum is: D Spiral CT of the chest
A. 4 limb blood pressure E. Multidetector com puted
measurements. tomography (MDCT) angiography
B. LFT’s.
C. Lateral chest x-ray. 100. The anticoagulation effects
D. CT chest associated with heparin therapy
E. ECG. are best reversed with the use o f
A. Vitamin K
Pulmonary Embolism and B Fresh-frozen plasma
Anticoagulation C Cryoprccipitatc
D. Prolamine sullatc
E Platelet administration
97. A 71-year-old presents to the
emergency room with shortness of
breath, hemoptysis, and chest pain. 101. Which of the following
Further tests include an ECG with statements regarding e n o u p a r in
findings of right-axis deviation, an (Lovenox) is true?
S1-Q3-T3 pattern, and a right A. Its use has not been shown to be
bundle branch block. The most cost-effective in an outpatient setting.
likely diagnosis is B. The medication does not require
A. Acute MI laboratory monitoring.
B. Community-acquired pneumonia C. The incidence of
C. Bronchogenic carcinoma thrombocytopenia is the same as with
D. Pulmonary Embolism heparin.
E. Pericarditis D. It mast be given through an IV
route
98. Which of the following is a F It is safe to asc in renal failure
potentially severe complication o f patients.
using warfarin that is unrelated to
excessive bleeding? M is c e lla n e o u s
A. Pancreatic neoplasm
B. Hepatitis 102. The most common cause of
C. Skin necrosis superior vena cava syndrome is
D Peripheral neuropathy A. Carcinoma o f the lung
E. Pulmonary fibrosis B. Aortic aneurysm
C. Tuberculosis
D. Metastatic carcinoma from a A Pencarditis
distant site B. Atrial myxoma
E Constrictive pencarditis C Malignant hypertension
D Heart failure
103. What is the MOST common H Rheumatic fever
cause of cardiogenic shock?
A Acute aortic insufficiency 107. Which of the following
B. Aortic stenosis statements is incorrect?
C. Hypertropluc cardiomyopathy A A bicuspid aortic valve is more
D. Acute myocardial infarction likely lo calcify than a tricuspid
(AMJ) valve.
C Pericardia) tamponade B. A patent ductus arteriosus (PDA)
is not compatible with hte
104. You find a loud systolic C A machinery murmur is heard with
murmur in a 14-ycar-old girl PDA.
during a preoperative assessment D Coarctation ol the aorta is
for an appcodiccctomy. Which of associated with I inner 's syndrome
the following signs or investigations L Chronic hypothyroidism
do NOT help confirm a diagnosis of predtsposes to atherosclerosis
coarctation of toe aorta?
A. AnkJc brachial pressure index 108. You arc managing a patient
B. Electrocardiogram who is hypotensive, refractory to
C Magnetic resonance angiography intravenous fluids. You decide to
D. Radial pulse asymmetry use a pure alpha-adrenergic agent,
E. Radio-femoral delay so choose:
A Dopamine
105. A 72-year-old roan with B. Dobulamine
refractory hypotension is C Amrinone.
commenced on dopamine in the D. Isoproterenol.
intensive care unit. Which of the E. Phenylephrine
following statements describes
dopamine?
A. Ip high doses it causes peripheral
vasodilatation
B. In low doses it increases renal
blood flow
C. It decreases cardiac output
D. It decreases splanchnic blood flow
E. It is able to cross the blood-brain
barrier

106 Cardiovascular causes of


clubbing include which of the
following?
2 PULMONARY MEDICINE

Normal Physiology and of severe chronic obstructive


Function pulmonary disease (COPD)?
A. FEVI 2 0 L (66% of predicted).
FVC 4 0 L (100% of predicted).
109. What u the approximate tidal
volume of a 70 kg male? FEVI/FVC 0 5 (66% of predicted)
A 100 mL B FFV 1 1 0 L (33% of predicted).
D. 450 mL FVC 3 0 L (75% of predicted),
C. 700 mL FFV 1/FVC 0 33 (44% of predicted)
D. 2400 mL C. FEVI 1.0 L (33% of predicted),
E 5000 mL FVC 1.25 L (31% of predicted),
FEVI/FVC 0.8 (107% of predicted)
D. FEVI 3.0 L (100% of predicted),
110. Wbat Is the approximate total
FVC 4.0 L (100% of predicted),
lung capacity of a 70 kg male?
A 2L FEVI/FVC 0.75 (100% of predicted)
B . 4L E FEVI 2.0 L (66% of predicted),
C . 6L FVC 2 67 L (67% of predicted),
D . 7L FEVI/FVC 0.75 (100% o f predicted)
E. 10 L
114. Which of the following
111. What is the approximate vital represent increased dead space
capacity of a 70 kg male? ventilation?
A. 1 L A. Acute respiratory distress
B . 3L syndrome (ARDS) -
C. 4.5 L B. Cyanide toxicity
D . 7L C. Pulmonary embolus
E. 10 L D. Pulmonary hemorrhage
E. Pneumonia
112. What is the approximate
volume of the anatomical dead 115. Which of the following is most
space in a 70 kg male? likely to shift the oxygen
A. 50 mL haemoglobin dissociation curve to
B. 100 mL the left?
C. 150mL A. Decreased pH
D. 200 mL B. Extreme altitude
E. 250 mL C. Increased 2,3-dipbosphoglycerate
concentration
113. Which of the following D Increased PaC02
pulmonary function test results, E. Increased temperature
recorded at a time when the patient
is free of an exacerbation, is typical COPD & Asthma &
Respiratory Failure
C. pH 7.45 Pa02 7 2 kl'a PaC'02 (>4
116. Which of the following kPa Bicarbonate 34 1 mmol/l
medications used to treat D. pH 7.27 Pa02 7 2 kPa PaC02 0.9
obstructive lung diseases is most kPa Bicarbonate 34 8 mmol/1
likely - to cause interm ittent E. pH 7 27 Pa02 7.1 kPa PaC02 5
hoarseness? kPa Bicarbonate 34.6 mmol'1
A. Salmcterol
B. Fluticasone 119. Causes of type l respiratory
C. Tiotropium failure include all of the following
D. Albuterol except?
E. Ipratropium A. Pneumonia
B. Pulmonary embolus.
117. Which of the following C Acute exacerbation of asthma.
statements about the treatment of D. Acute respiratory distress
an exacerbation of COPD is true? syndrome.
A. An 8-week course of oral steroids E. Pulmonary fibrosis.
is associated with fewer relapses than
a 2-week course. 120. Which of the following
B. Intravenous steroids are more infections is least likely in a patient
effective and have fewer with COPD?
gastrointestinal side effects than oral A. Streptococcus pneumoniae
steroids. B. Haemophilus influenzae
C. Bilevel positive airway pressure C. Moraxella catarrhalis
ventilation is contraindicated because D. Mycoplasma pneumonia
o f the risk of hyperinflation and auto-
PEEP. 121. A 19-year-old man with
D. Antibiotics are not indicated in the asthma complains of shortness of
absence o f sinusitis or pneumonia. breath and wheezing when playing
E. Sputum Gram stain and culture are sports. Otherwise he has no
not routinely necessary to guide the symptoms. The best preventive
choice of antibiotics. treatment is
A. Antileukotrienes before exercise
118. A 71 year old man with a long B. Regular inhaled steroids
history of COPD. An arterial blood C. Inhaled P agonist before exercise
gas is performed which is D. Long-acting [1 agonist
interpreted as showing type 2 E Anxiolytic medication
respiratory failure. Which of the
results is most consistent with that 122. A 27-year-old patient with
diagnosis? asthma presents to your office
A. pH 7.55 Pa02 7.1 kPa PaC02 6.1 complaining of shortness of breath
kPa Bicarbonate 34.8 mmol/1 with wheezing. Which of the
B. pH 7.45 Pa02 8.4 kPa PaC02 7.1 following medications is indicated
kPa Bicarbonate 29.4 mmol/l in the initial treatment of this
patient?
A. SaJmcterol arterial blood gases tak en which
B. Albuterol give the following results: pO j
C. Cromolyn sodium 65mmHg, pCOj 33m raH g), pH
D. Ipratropium bromide 7.46, {HC03-J 24. W hich of the
E. Theophylline following is the m ost likely
diagnosis?
123. Which of the following drugs A. Hyperventilation syndrome
is a Icukotricne B Acute severe asthma
antagonist/inhibitor? C. Emphysema
A. Theophylline D Kyphoscoliosis
B. Prednisone E. Opiate overdose
C. Salmeterol
D. Terbutaline 127. W hich of the following
E. Zafirlukast and Montelukast statem ents about homozygous
alpha-1 antitrypsin deficiency is
124. A 30-year-old man presents to true?
your clinic with a cough and finger A. The diagnosis is suggested by the
clubbing. From the list below, presence o f widespread emphysema
which of these answers is not a on chest computed tom ography (CT)
respiratory canse of finger scan in a nonsmoker.
dubbing? B. The diagnosis is established if the
A. Empyema serum alpha-1 antitrypsin level is less
B. Mesothelioma than the lower limits o f the normal
C. Bronchogenic carcinoma range.
D. Cystic fibrosis C. All patients writh alpha-1
E. COPD antitrypsin deficiency develop
obstructive lung disease.
125. A 64-year-old man presents D. The majority o f persons with
with a urinary tract infection. On alpha-1 antritrypsin deficiency’
examination he has a barrel-shaped develop clinically significant liver
chest; there is evidence of use o f the disease at some point during their
accessory muscles of respiration lifetime.
and pursed lip breathing. Which of E A characteristic feature that
the following routine blood results distinguishes alpha-1 antitrypsin
is most likely to be found in this deficiency from em physema o f other
patient? causes is a preserved diffusion
A. Lymphopaenia. capacity for carbon monoxide
B. Anaemia. (DLCO) in alpha-1 antitrypsin
C. Raised MCV. deficiency
D. Polycythaemia.
E. Thrombocytopenia. 128. A 67-year-old m an presents
with dyspnea an d peripheral
126. A 47 year old woman oedem a. The p atien t has a
presenting with breathlessness has longstanding history of COPD. You
suspect cor pulmonale. W hich of L ACE inhibitors
the following is not a cause of cor
pulmonale? 132. The treatment of choice for
A. Pulmonary fibrosis adult respiratory distress syndrome
B. Primary' pulmonary hypertension (ARDS) is
C. Myasthenia gravis A l oop diuretics
D. COPD R Corticosteroids
E. Multiple sclerosis C Positive end-expiratory pressure
D agonist
Respiratory Infections L- High-dose immunosuppressive
dmgs
129. Which of the following tests is
most likely to identify an etiology of 133. Which of the following
bronchiectasis for which a specific organisms would typically be found
therapy is available? in a patient with atypical
A. Antinuclear cytoplasmic antibody community-acquired pneumonia?
(ANCA) A Staphylococcus aureus
B. Serum immunoglobulin G B Pseudomonas spp
C. Gluten antoantibodies (serum C. Streptococcus pneumonia
endomysial and anti tissue D. Legionella pneumophilia
transglutaminase antibodies) E Haemophilus influenza
D. Bronchial biopsy examined by
electron microscopy 134. Which of the drugs below
E. Full pulmonary' function tests, would be the most appropriate to
including measurement of lung treat pulmonary' Aspergillus spp.
volumes and diffusion capacity for infection?
carbon monoxide. A. Amoxicillin
B. Erythromycin
130. The most common cause of a C. Amphotericin B
community-acquired pneumonia in D. Flucloxaciliin
a 45-year-old otherwise healthy E. Fluconazole
man is
A. Streptococcus pneumoniae 135. A 45-year-old man diagnosed
B. Haemophilus influenzae with pulmonary TB and on
C. Mycoplasma pneumoniae treatment 2 months ago, presents
D. Legionella pneumoniae to you with a week’s history o f pins
E. Klebsiella pneumoniae and needles in bis hands and feet
with associated numbness. Which
131. The most common cause of of the following drugs is
chronic cough is responsible for the symptoms
A. Postnasal drip described by the patient?
B. Bronchiectasis A. Pyrazinamide
C. Gastroesophageal reflux B. Rifampicin
D Asthma C. Ethambutol
0 Isoniuzid
l*. None ot the above 139. Which of the following
collagen-vascular diseases is more
136. A 55 year old man who has a often associated with airway
25 year pack history of smoking disease thao interstitial lung
presents with productive cough disease?
with mucoid sputum of 2 year A Scleroderma
duration. On examination he has B Polymyositis
scattered ronebi and whcezing.The C Relapsing polychondritis
likeliest diagnosis is: D Systemic lupus erythematosus
A. Broncliial Asthma E Mixed connective tissue disease
B. Bronchiectasis
C. Chronic Bronchitis 140. A 59 year old lady with Grade
D. Pneumonitis 3 congestive cardiac failure has
E. Fibrosing Alveolitis become increasingly short of
breath. A chest radiograph is
137. W hich of the following best reported to dem onstrate bi-basal
describes Ludw ig’s angina? lung fibrosis which w as not present
A. Substemal chest pain that radiates 2 years earlier. W hich of her
to the right arm. cardiac medications is the most
B. An infection involving the likely cause of the fibrosis?
sublingual and submaxillary space. A. Atenolol
C. Abdominal pain secondary to an B. Spironolactone
enlarging Abdominal Aortic C. Digoxin
Aneurysm. D. Amiodaronc
D. A tonsillar infection that leads to E Atorvastatin
chronic abscess formation.
E. Ischemic pain related to 141. A 50-year-old man complains
insufficient blood flow to an of shortness o f breath with
extremity. evidence of honeycom bing o f
pulmonary architecture on a chest
138. A 46-year-old w om an has been radiograph. Pulmonary function
diagnosed with bronchiectasis tests show both decreased vital and
following a three-m onth history of diffusing capacities as well as
a m ucopurulcot cough. W hich of reduced total lung volumes with a
the following from the list below is normal forced expiratory volume
not a cause of bronchiectasis? in I second to forced vital capacity’
A. Kartagener’s syndrome ratio. The most likelv diagnosis is
B. Cystic fibrosis A COPD
C. Pneumonia B. Tuberculosis
D. Left ventricular failure C Asthma
E. Bronchogenic carcinoma D Idiopathic pulmonary fibrosis
E. Chronic Pulmonary Embolism
I n te s titiu l L u n g D is c u s e s
142. W hich of the following the histological subr> pc most likdv
interstitial lung diseases responds to be?
poorly to corticosteroid treatm en t? A. Broncho-alveolar cell carcinoma
A Acute eosinophilic pneumonia B Squamous cel! carcinoma
B Allergic bronchopulmonary C. Pulmonary hamartoma
aspergillosis D Ovarian carcinoma metastasis
C Idiopathic pulmor.ury fibrosis E. Small tout) cell carcinoma
D Bronchiolitis obir.erans vs.ih
organizing pneumonia \BOOP) 146. Which of the following lung
E Sarcoidosis cancers is most commonly
associated with the syndrome of
143. A patient with Rheumatoid inappropriate secretion of
arthritis complains of progressive anddiuretic hormone (SIADH)?
breathlessness. Which of the A- Squamous cell carcinoma
following is the most like)} cause? B. Small cell (oat cell) carcinoma
A. Pulmonary Eosinophilia C. Large cell carcinoma
B Asthma D. Adenocarcinoma
C. Pulmonary nodules E. Mesothelioma
D. Fibrosing Alveolitis
E. Pulmonary Embolus 147. A 56-year-old woman is
recently diagnosed with small cell
144. A breathless patient undergoes carcinoma of the long. Which of
pulmonary Function testing. The the following non-mctastatic
following results are obtained; manifestations is she most likely to
FEV1 = 74% of predicted, FVC = develop?
68% of predicted, TLC = 77% of A. Myasthenia gravis
predicted, Tlco = 46% of predicted, B. Eaton-Lambert syndrome
Kco = 53% o f predicted. Which of C. Ectopic PTH-related peptide
the following is the most likely secretion
cause? D. Erythema gyratum repens
A COPD E. Hypertrophic pulmonary
B. Asthma osteoarthropathy (HPOA)
C. Cryptogenic Fibrosing Alveolitis
D. Morbid obesity 148. A 62 year old man is found to
E. kyphoscoliosis have squamous cell carcinoma of
the lung after being investigated for
Lung Tumors haemoptysis. Which one of the
following woyld be a
145. A 68 y ear old sm oker is contraindication to surgical
adm itted with syndrom e of resection?
inappropriate anti-diuretic A. Finger clubbing
hormone secretion (SLADH) and a B. Hypercalcaemia
chest radio g rap h dem onstrates a C. Hypertrophic pulmonary
3cm right u p p e r lobe mass. W hat is osteoarthropathy
D. Pleural effusion 152. Which of the follow ing tests b
E- Superior vena cava obstruction used in the initial evaluation of
persistent hemoptysis?
149. A 67-year-old man who has A_ Fiber-optic bronchoscopy
been referred to the chest clink B Chest radiograph
following a three-month history of C Upper gastrointestinal (GJ)
weight loss and signs which may D MR] of chest
suggest a Pancoast's tumour. E. CT scan of chest
Which of the following symptoms
from the list below is not associated 153. A patient taking warfarin
with a Pancoast's tumour? arrives to the Emergency
A. Hoarse voice Department with severe
B. Miosis hemoptysis. Which of the following
C. Anhydrosis would be best initial management?
D. Exopthaimos A. Administer Fresh Frozen Plasma
E. Ptosis B. Check complete blood cell count
C Cryoprecipitate
Hemoptysis D. Transfusion o f uncross matched
red blood cells
150. A 36 year old woman is F Withhold warfarin for l day
admitted with a working clinical
diagnosis of pulmonary embolus 154. A 66-year-old woman presents
and a CT-pulmonary angiogram is to your clinic with a 1-week history
requested. What is the most of haemoptysis. W hkh of the
important consideration before the following from the list of answers
test can go ahead? below Is not ' a cause of
A. Normal liver function tests haemoptysis?
B. Normal serum glucose A. Pulmonary tuberculosis
C. Normal renal function B. Bronchiectasis
D. Normal thyroid function C Asper gil Ionia
E. If she is on the oral contraceptive D Wegener’s granulomatosis
pill E. Asthma

151. A 52-y car-old teacher, 155 According to the British


presents with a sudden onset of Thoracic Society, which of the
dyspnea. Which one of the following is a minor risk factor for
following makes a pulmonary a pulmonary embolus?
embolus more likely? A. Abdominal malignancy.
A. Fever >38 0*0 (100 4°F) 8 Recent hip replacement.
B. Chest pain C. Oral contraceptive pill.
C Orthopnea D Previous venous
D Wheezes thromboembolism.
E Rhone hi E Immobility
156. Which of the following to inidzoncs of the right lung.
associations is incorrect? There is also reduced chest
A. Pulmonary fibrosis - line end expansion on the right. The most
inspiratory crackles. likely diagnosis is:
B Extrinsic allergic alveolitis - type 1 A Right middle lobe pneumonia
respiratory failure. B. Pulmonary embolism
C Pulmonary embolus - hypoxia. C Right-sided pleural effusion
D. Cystic fibrosis - fine crepitations D. Right-sided bronchial carcinoma
E. Pneumonectomy - reduced breath E Right lower lobe pneumonia
sounds.
160. A 2S-> car-old man developed
157. Which of the following an acute onset of pleuritic chcsl
vascular structures is the source of pain and shortness of breath while
the hemoptysis seen iu patients playing football. On examination,
with bronchiectasis? there is decreased expansion of the
A. Bronchial artery chest on the left side, hyper-
B Bronchial vein resonant to percussion and reduced
C. Pulmonary capillaries air entry on the left. The most
D. Pulmonary artery likely diagnosis is:
E. Pulmonary vein A. Leri-sided pneumothorax
B. Left-sided pneumonia
Pleural Diseases C. Left-sided pleural effusion
D. Lung fibrosis
158. A 21-year-old is brought to the E. Traumatic chest injury
emergency room after developing
sudden shortness of breath. A chest Miscellaneous
radiograph shows a 10%
pneumothorax. The patient 161. Which of the following
remains stable. Appropriate attributes is a risk factor far
management includes: obstructive sleep apnea?
A. Immediate chest tube placement A. Retrognathia
B. Intubation and mechanical B. Dress shin neck circumference
ventilation >15 inches
C. Pulmonary function testing C. Nasal polyps with complete nasal
D. Large-bore needle placed in the airflow obstruction
second intercostal space D. Short stature (<60 inches tall in
E Observation women, <65 inches tall in men)
E. Insomnia
159. A 54-year-old woman with a
history o f weight loss, loss of 162. Moderate and severe sleep
appetite and shortnesss of breath, apnea syndrome is associated wrth
present with respiratory rate is increased morbidity and/or
19and reduced air entry and mortality due to which of the
dullness to percussion ou the lower following?
A Aspiration ray reveals bilateral hilar
B Laryngospasm lypbadenopathy. From the list
C Diabetic ketoacidosis below, select the most likely
D Motor vehicle uccidents diagnosis:
E Hypoxemic respiratory failure A. Rheumatoid arthritis
B. Systemic lupus erythematosus
163. Which of the following (SLE)
statements about cystic fibrosis Ls C Sarcoidosis
true? D. Idiopathic pulmonary fibrosis
A. It is an autosomal-dominant E. Bronchogenic carcinoma
transmitted disease.
B. The condition is associated with 165. A 76-year-old man has the
pancreatic insufficiency following clinical signs: unilateral 4
C. It is commonly diagnosed with a chest expansion, 1 breath sounds
pulmonary function test. and i percussion note. Which o f the
D. Those affected do not live beyond following clinical conditions would
20 years o f age. provide these clinical signs?
E. Fertility is not affected in those A. Pneumothorax.
with cystic fibrosis. B. Fibrosis.
C. Consolidation.
164. A 50-ycar-old with no past D. Pleural effusion.
medical history, presents with a 4- E. Extensive
raonth history of dry cough and collapse/lobectomy/pncuraonectomy.
shortness of breath on exertion.
Blood tests revealed a raised ESR
and serum angiotensin-converting
enzyme level and patient’s chest x-
Gastroenterology and Disorders
O f The pancreas And Liver

‘ Liver Functions 170. Choose the correct statement


about laboratory studies available
166. Gilbert's disease is associated to test hepatic function:
with A AST is more specific than Al I as
A. Overproduction o f glucuronyl a marker of hepatocyte injury
transferase B. Prothrombin lime anJ albumin
B. Mild (benign) elevations of reflect hepalocue synthetic function
indirect (unconjugated) bilirubin C. Elevated alkaline phosphatase
C. Intravascular hemolysis (AP) reflects catabolic function of the
D. Increased risk for liver disease liver.
D. Lactate dehydrogenase iLDH)
167. Which of the following causes elevation is highly specific for
pre-hepatic jaundice? hepalocue damage.
A. Alcohol E. Elevated serum ammonia levels
B. Drugs reliably correlate with acute
C. Hemolysis worsening of hepatic function.
D. Toxins
E. Viral infection 171. Jaundice is visible in the eyes
before the skin because:
168. Which of the below results is A. Scleral tissue has a high collagen
the best indicator o f poor liver content than skin, allowing more
function? efficient accumulation of bilirubin.
A. Raised alanine transferase B. Scleral tissue is supplied with
B. Raised albumin nutrients by perfusion rather than
C. Raised alkaline phosphatase direct circulation, allowing a more
D. Raised bilirubin highly concentrated accumulation of
E. Prolonged prothrombin time bilirubm.
C. Sclera are lighter than skin.
169. A patient present with D. Scleral tissue is high in clastin
jaundice and right upper quadrant content, and elastin has a high affinity
pain. What levels o f plasma for bilirubin.
bilirubin would this patient have in E. Bilirubin is secreted by the
order for jaundice to be clinically lacrimal glands and directly absorbed
visible? into the sclera.
A. >1 8mg/dL
B. > 1 5mg/dL 172. Which of the following
C. >2mg/dL concerning the conjugation of
D. >0 9mg/dL bilirubin is correct?
E. >1.2ing/dL A. is catalysed by a giucuronyj
transferase

IQ
13 occurs in Ihc Kuplcr cells uf the positive for hepatitis C to prevent
liver transmission to the newborn.
C. is increased by valproate D Hepatitis C can be spread by
D. is inhibited by nfampicin contaminated water supplies
I£ is impaired in Dubin-Johnson E. Hepatitis C does not appear to be
syndrome transmitted in breast milk-

173. Which of Ihc following is true 176. Which of the following forms
of Gilbert's syndrome? of hepatitis does NOT have □
A. inheritance is autosomal recessive chronic state?
B. serum conjugated bilirubin levels A Hepatitis A
.are elevated B. Hepatitis B
C. serum bilirubin levels are C. Hepatitis C
decreased by fasting D. Hepatitis D
D. scrum bilirubin levels are
decreased by liver enzyme inducers 177. Which o f the following tests is
E. there is bilirubinuria used to detect hepatitis B infection
during the “Window period”?
Hepatitis A. Hepatitis B surface antigen
B. Hepatitis B surface antibody
174. A 45-year-old man is found to C Hepatitis B core antibody (IgM )
have HAVab Negative; HBVcab D. Hepatitis B e antigen
Positive; HBVsAg Negative; E. Hepatitis B antibody to the delta
HCVab Positive. The following agent
conclusion that can be made about
the patient is: 178. Which of the following
A. He is a carrier for hepatitis A statements regarding hepatitis C is
B. He is a carrier for hepatitis B correct?
C. He is a carrier for hepatitis C A. Most patients are symptomatic
D. He has been vaccinated against with development o f the disease.
hepatitis B B. The course o f the disease shows
E. He does not have viral hepatitis no variability and is progressive.
C. Most patients develop chronic
175. Which o f the following hepatitis.
statements is correct concerning D. The disease is not transferred
hepatitis C virus? through sexual contact.
A. There is no risk to infants if the E. Immune globulin is effective for
mother is affected. postexposure prophylaxis.
B. There is no nsk associated with
sexual intercourse with an individual 179. A 45-year-old woman presents
with hepatitis C. with malaise* joint pains and
C Cesarean section should be jaundice. Laboratory results show
performed on mothers who lest positive HBsAg, HBcAg, and
HBcAg. Which of (he following is
the most likely diagnosis? 183. Which of the following
A. Acute hepatitis B infection medications causes predictable,
B. Chronic hepatitis B infection dose-dependent hepatocellular
C Hepatitis B immunity following injury?
prcvioiLS infection A. Morphine
D. Hepatitis B immunity following B INH
previous vaccination C Gold
E. No hepatitis B infection D. Acctamuiophen
F Acctylsalicyclic aciJ (ASA)
180. Patients with autoimmune
hepatitis disease inny have the 184. A 52-ycar-old woman presents
following presentations except with elevated scrum liver values
A. AST 2000, ALT 2500, T Bili 4 and positive scrum Anti-
U. New’ onset jaundice and hepatic Mitochondrial Antibodies test.
encephulopathy \Miich of the following scrum blood
C. ALP 300, ALT 50,T Bili 3.5 tests should be followed to assess
D. Multiple relapses over several her treatment response?
years A. ALT
Ii Dramutie responsiveness of B. Alkaline Phosphatase
AST/ALT levels to oral prednisone C AST
therapy D. Bilirubin
E. Prothrombin time
181. Which of the following ty pes of
hepatocellular injury is commonly Cirrhosis and Portal
seen after acetaminophen Hypertension
overdose?
A. Acute hepatitis 185. The first step in the
B. Ccntrilobular necrosis management o f a lower GI
C. Fibrosis hemorrhage is to
D. Granuloma formation A. Obtain a CT scan o f the abdomen
E. Microvesicular fatty chan B. Perform a bleeding scan
C. Resuscitate the unstable patient
182. A 33-ycar-old man has never D. Perform a colonoscopy
been vaccinated for hepatitis B. E. Obtain a surgical consult
Serologic tests reveal negative
HBsAg and positive HBsAg 186. What is the MOST common
antibody. Which of the following cause of LOWER GI bleeding?
conditions this patient has? A. Adenomatous polyps
A. previous hepatitis B infection B. Diverticular disease
B. chronic active hepatitis C. Infectious colitis
C. acute hepatitis B infection D Inflammatory colitis
D. poor prognosis E. Malignancy
E. need for vaccine to hepatitis B
187. What is the most common L Anti-smooth muscle an tib o d u s
cause of rectal bleeding?
A Anal fissures 192. A 56-year-old man with a long
B Colon cancer history of alcohol abuse presents
C. Divcrticulosis with abdominal pain. On
D llem onhoids examination he has a distended
F. Ulcerative colitis abdomen with shifting dullness and
has a temperature of 38.2°C . What
188- Which of the following test is the most likely diagnosis?
rcsalts MOST accurately predicts A. Bowel obstruction
spontaneous bacterial peritonitis? B Liver cirrhosis
A. Low serum glucose C Mallory-W eiss syndrome
B. High protein in peritoneal fluid D. Perforated peptic ulcer
C. High protein in semm E. Spontaneous bacterial peritonitis
D. Neutrophil count greater than (SBP)
250/ml in the peritoneal fluid
E. Total white count greater than 193. A 52-year-old man with a
500/mJ history o f alcohol excess present
with his finger is perm anently
189. Which of the following partially flexed. W hich o f the
conditions .does not classically following b the m ost likely
cause hepatomegaly? diagnosis?
A- End-stage cirrhosis A. Astcrixis
B. Fatty liver B. Dupuytren contracture
C. Hepatocellular carcinoma C. Palmar erythema
D. Myeloproliferative disease D Trigger finger
E. Right-sided heart failure E. Xanthelasma

190. Which o f the following 200. A 15-year-old girl is being


conditions is associated with investigated for behavioural
sclerosing cholangitis? changes and is found to have a
A. Autoimmune hepatitis brown-green discolouration
B. Coeliac disease around her iris on slit lamp
C. Irritable bowel syndrome examination. What is the name of
D. Pernicious anaemia this feature?
E. Ulcerative colitis (UC) A. Arcus senilis
B. Caput medusae
19L Which antibody can you C. Kayser-FIcischer rings
expect to see in primary biliary D. Lens dislocation
cirrhosis? E. Xanthoma
A. .ANA
B. ANCA 201. A 69-ycar-old man present
C. Anti-mitochondrial antibody with a 2-week history o f abdominal
D. Anti-phospholipid antibodies pain. Ou examination, thejm tieut is
jaundiced and the abdomen is C. Heriditary haemochromotosis
distended with tenderness iu the D. Primary sclerosing cholangitis
epigastric region with smooth E. Alcoholic liver disease
hepatomegaly and shifting dullness.
Which of the following is a cause of 205. Which of the following
hepatomegaly? statements about pulmonary
A. Iron deficiency anaemia manifestations of liver disease is
B. Budd-Chiari syndrome incorrect?
C. Ulcerative colitis A. Patients with hepatopulmonary
D. Crohn’s disease syndrome typically show decreased
E. Left-sided heart failure oxygen saturation when going from
sitting to lying
202. A 60-year-old man undergoes B Emphysema can be the initial
endoscopic variceal banding. presentation o f alpha-1 antitrypsin
Which of the following medications deficiency
would act as prophylaxis in C. Hypoxemia can develop from
preventing a rebleed from his right-to-left intrapulmonary shunting
oesophageal varices? of blood
A. Fruseraide D. Primary biliary cirrhosis has been
B. Amlodipine associated with fibrosing alveolitis
C. Ramipril
D. Propranolol 206. A 66-year-old man has long­
E. Irbesartan standing portal hypertension.
Which of the following anatomical
203. A 67-year-old man presents areas does not represent a site of
with deranged liver function tests. portosystemic varices?
Which of the following clinical A Intrahepatic, between the portal
signs does not form part of chronic vein and the inferior vena cava
liver disease? B Lower oesophagus, between the
A. Finger clubbing left gastric vein and oesophageal
B. Palmer erythema branches o f the azygous vein
C. Spider naevia C Rectal, between the superior rectal
D. Koilonychia vein and the pudendal vein
E. Jaundice D Retroperitoneal, between the
ovarian vessels and the renal veins
204. You see a 52-year-old woman E Umbilical, between the obliterated
with rheumatoid referred because umbilical vein and the left portal vein
her ALP levels were found to be
abnormally high at 300 iu/L and 207. Melena:
has serum anti-mitochondrial A- Is present in 95% of patients with
antibody positive. The most likely upper gastrointestinal bleedings
diagnosis is: (UGB).
A. Primary biliary cirrhosis B. Is from blood that has been in the
B. Wilson’s disease GI tract for at leasf 24 hourt.
C Is present in approximately one- 210. A 48-year-old woman
third of lower gastrointestinal complains of pruritis, steatorrhoea
bleedings (LGIB) and bruising. On examination, she
D. May occur from as little as 10 ml is jaundiced, pigmented with spider
of blood in the GI tract. naevi and hepatosplenomegaly.
E. Will remain black and tarry for no What is the most likely underlying
more than 24 hours after bleeding diagnosis?
stops. A. Autoimmune hepatitis
B Primary biliary cirrhosis
208. Lactulose removes ammonia C Alcoholic liver disease3
ions from the gut by: D AJpha-l antitrypsin deficiency
A. Being convened to lactic acid by E Wilson's disease
colonic bacteria; this acidifies the
feces, resulting in the trapping of 211. A 62-year-old m ale with
ammonia as ammonium in the stool. cirrhosis present with a severe nose
B. Causing ammonia to be bleed. His investigations reveal: lib
transmitted to the intracellular region 10.9 g/dL; WBCs 5000/dL; platelet
through the sodium-potassium count 60000; PT 17.5s ( l 1.5-15.5);
membrane pump, where it is safely APPT 42s (30-40); and fibrinogen
used in the tricarboxylic acid cycle. 0.7 g/L (1.8-5.4). What is tbc most
C. Forcing renal excretion of appropriate blood product for this
chloride, leading to compensatory patient?
hyperpnea and respiratory alkalosis, A. Cryoprecipitatc
which keeps ammonia nontoxic. B. Factor VIII
D. Providing a protective coating on C. Platelets
the gut’s brush border, which blocks D Prothrombin complex concentrate
ammonia absorption. E. Whole blood
E. Mycelizing preammonia prior to
its harmonic conversion. 212. A 22-ycar-old patient presents
with valproate hepatotoiicity and
209. Which of the following is NOT significant (grade H i) hepatic
true of a patient with ascites due to eocephalopatby. What should be
liver cirrhosis? the treatment of choice?
A. Spontaneous bacterial peritonitis is A. Antioxidants
a recognised feature B N-acctylcystcinc
B. The usual source of the ascitic C Prednisone
fluid is mainly from the exudation D IJDCA
frornthe surface o f the liver E Carnitine
C. Hepatic mlrasinusoidal pressure is
elevated 213. Which of the following
D Urinar>r sodium concentration is histologic featuresbest
usually < 10 mmo 1/1 distinguishes Non-alcoholic
E. Cardiac output is often elevated Steatohepatitis (NASH) from
Alcoholic Steatohcpatitis (ASH) on
histology? Esophagus and Stomach
A. Mallory bodies Disorders
B. Hepatocyte balloon degeneration
C. Apoptotic bodies 217. Which of the follow ing inhibits
D. Steatosis the production of gastric acid by
E None of the above the parietal cell?
A. Histamine
Hepatocelluar carcinoma B. Acetylcholine
C. Prostaglandins
214. A 6S-y ear-old man with ' D. Gastrin
chronic HBV infection presents ! E~ Lipase
with a 1-month history of weight lI
loss, malaise and right upper 218. W hich of the following b no! a
q uad rat discomfort. On I known mechanism of injury
examination he has mild ascites j induced by H. py lori?
and appears cachectic. What is the A_ Injury to D ceus 'dial produce
most likely diagnosis? somatostatin. allowing nr inhibited
A. Cholaogiocarcinofna gastric acid production
8. Cirrhosis o f the liver 3. Disruption of the function of a
C. Hepezocelluiar carcinoma mismatch repair gene that, in turn,
D. Reactivation o f bepantis B leads to the development o f gastnc
E. Right-heart failure cancer
C. Production of a protease that thins
215. You are asked to perform a the mucous layer, leading the mucosa
tumor marker level for a patient more prone to injury
with hepatocellular carcinoma. D. Inciting the development of
Which of the following tumor autoantibodies that cross-reaci with
markers are elevated in him? gastric epithelial ceils
A. a-fetoprotein E. Causing lymphoid tissue
B. Carcinoembryonic antigen (CEA) aggregation that may develop into
C. CA 15-3 MALT (mucosa-associated lymphoid
D. HcG tissue) lymphoma
E. CA 125
219. Which of the following clinical
216. Which of the following is not a scenarios may result in a decrease
known predisposing factor for in the serum concentration of
developing hepatocellular gastrin?
carcinoma? A. Taking an H2-rcceptor antagonist
A. Hepatitis B virus for 6 months
B. Liver cirrhosis B. H. pylori infection
C. Hepatitis C virus C. Chronic atrophic gastritis
D. Hepatitis A virus D. Administration of secretin to a
E. Aflaioxin healthy subject
E Administration of secnrtm to a B Vitamin D deficiency
patient with Zoiltnger-Ellison C Vitamin B12 deficiency
syndrome D Folate deficiency

220. Gastroduodenal manometry is 224. Which of the follow ing is an


a useful clinical test in which of the acceptable treatm ent for
situations below? Helicobacter pylori infection?
A. In the evaluation of a paficnl with A B;smuth. amoxicillin,
scleroderma who h2s delayed gastric raetiororiazpfg, and omeprazole
emptying 3 TM P-sd fametboxazo !e. sccxalrasc,
B. In the evaluation of a pafiesa with and cecroodzzole
suspected rumination syndroms C. Omeprazole, cl:rd am > n n . a&3
C. In a patient with cnexpiafccd sucralfate
obstructive symptoms (negative D. Doccsair. M racyclite. enz
small-bowd fcliowthrough) and j aicrromdazc4e
history o f previous abcSajuna: j E_ Ramcdire, metxctudazole. aac
operations amptcfilin
D. In the evaluation of a pacer.: with
severe upper gastroitnestmal tract 225. Barren's esophagus is
symptoms when all other tests are | associated with
entirely normal A Overuse o f PPls
E. In the evaluation of a patient with B Tracheoesophageal fistula
amyloidosis who has delayed gastric C. Trauma associated with poor
emptying esophagogasmoduodenoscopy
D Transfonnation o f columnar
221. Which patients) should epitbehum to squamous epithelium
receive prophylaxis for stress E. Adenocarcinoma o f the esophagus
ulcer?
A. Patient with sepsis and acute renal 226. Which is the CORRECT drag
failure match used to treat peptic ulcer
B. Patient with chronic renal failure disease and its associated
and coronary artery disease who is mechanism of action.
receiving aspirin therapy A. Antacids—block secretion o f acid
C. Patient on day #3 o f mechanical at the proton pump.
ventilation for pneumonia B. H2rcccptor antagonists— inhibit
D. Patient on day # l after histamine receptor o f parietal cells.
cholecystectomy C. Misoprostol— protects ulcer
E. Patient on day #2 afteT fromacid exposure by forming a
appendectomy who is receiving low- sticky gel.
dose heparin therapy D. Protonpurapinhibitor—buffer
gastric acid.
223. The use of proton pump E. Sucralfate—prostaglandin analog
inhibitors (PPls) can result in that increases bicarbonate production.
A. Vitamin C deficiency
227. A 45-year-old m au presents B Chest x-ray
with interm ittent difficulty in C. Upper G1 endoscopy
swallowing for the last 4 months. D. Barium meal
This is associated with severe E Ultrasound
retrosternal pain and
regurgitation. W hat is the roost 231. All of the follow ing statements
im portant investigation in this about Helicobacter pylori (11.
case? pylori) are true except
A. Banuin swallow A. H. pylori does not invade the
B. Chest X-ray gastric or duodenal epithelium
C. CT of the chest B. IT pylori stimulates gastric acid
D. Endoscopy secretion
E. Iron studies C. Eradication of H. pylori prevents
adenocarcinoma of the stomach
228. A 29-year-old w om an is D. H. pylon produces a urease that
concerned about recurrent mouth splits urea into ammonia and C02
ulcers every few months, they arc E. Triple-drug therapy for eradication
small, grey, shallow ulcers that of IT pylori is more effective than
disappear spontaneously. She has dual therapy
no other gastrointestinal sym ptom s
or weight loss. W bat is the most 232. Which of the following is a
likely cause? risk factor for Helicobacter pylori
A Behcet disease infection?
B Crohn’s disease A. excess exposure to antibiotics
C Herpes simplex B. female sex
D Idiopathic aphthous ulcers C. alpha 1-antitrypsin deficiency
E Oral carcinoma D. low socioeconomic indicators
E. proton pump inhibitor therapy
229. W hich of the following is the
most common cause of duodenal 233. Which of the following is NOT
ulcers? considered to be a risk factor for
A. NSAIDs peptic ulcer disease?
B. Helicobacter pylori A. Pregnancy.
C. Alcohol abuse B. Male sex.
D. Chronic corticosteroid therapy C. Head injury.
E. Zollinger-EUison syndrome D. Severe bums.
H. Steroids.
230. Vou see a 48-year-old man
presents with a 3-month history of 234. A 47-year-old woman presents
heartburn after meals which not with dysphagia. Which of the
relieved by antacids and PPIs. The following is not an obstructive
most ap p ro p riate investigation for cause of dysphagia?
diagnosing a hiatus hernia is: A. Pharyngeal carcinoma
A. Computer tomography (CT) scan B. Oesophageal wob
C Retrosternal goitre D. The area of involvement is
D. Peptic stricture localized to the colon and
E. Achalasia rectosigmoid area.
E. Aminosalicylic acid compounds
235. A 29-year-old male presents are not effective in the treatment of
with symptoms of severe gastro- ulcerative colitis
oesophageal reflux. Which one of
the following is most useful in 238. A histology report for a
assessing the role of surgery? specimen taken from the colon of a
A. Cardiac sphincter manometry 24-year-old woman reports ^patchy
B. Gastric emptying study areas of transmural noo-easeating
C. Intragastric pH monitoring off granulomatous inflammation”.
therapy What disease process does this
D. Oesophageal motility study suggest?
E. Oesophageal pH monitoring on A. Colonic adenocarcinoma
therapy B Crohn's disease
C Irritable bowel syndrome
Inflammatory Bowel Diseases D. Tuberculosis
E. Ulcerative colitis
236. Crohn’s disease is associated
with which of the following? 239. Which of the following
A. Inflammation limited to the gastroenterological conditions
superficial layer of the bowel wall would give rise to finger clubbing?
B. The affinity to involve the A. Hepatocellular carcinoma
rectosigmoid junction B. Crohn’s disease
C. Decreased risk of colon cancer C. Irritable bowel syndrome
D Continuous mucosal areas of D. Pancreatic carcinoma
ulceration that affect the anus
E. Fistula formation 240. A patient has been diagnosed
with primary sclerosing cholangitis
237. Which of the following (PSC). PSC is associated with
statements is true regarding which of the following diseases?
ulcerative colitis? A Thyroid disease
A. There is transmural involvement B. Systemic sclerosis
o f the bowel wall. C Rheumatoid arthritis
B. There arc skipped areas of D Ulcerative colitis
inflammation that may have a E. Irritable bowel syndrome
cobblestone appearance on
colonoscopy 241. On visualization of the rectum
C. There is a smaller risk of by sigmoidoscopy, the mucosa
developing intestinal cancer in appears inflamed aud friable. A
comparison with Crohn’s disease. rectal biopsy shows mucosal ulcers
with inflammatory infiltrate, crypt
abscesses with goblet cell depletion.
What is the most likely diagnosis? 246. A 29-ycar-old man presents
A. Crohn's disease with anaemia, bleeding tendency,
D. Pseudomembranous colitis diarrhea and abdominal pain.
C. Irritable bowel syndrome Examination reveals a palpable
D. Ulcerative colitis mass in the right lower quadrant
E. No diagnosis - the report is and anal skin tugs. What isthc most
inconclusive likely underlying condition?
A Chronic pancreatitis
242. Which of the following extra- B. Cocliac disease
intestinal signs is not seen in C Crohn's disease
ulcerative colitis? D Intestinal lymphoma
A. Finger clubbing E. Ulcerative colitis
B. Erythema nodosum
C. Iritis Malabsorption
D. Sacroiliitis
E. Granuloma annulare 247. Which of the following tests is
recommended to be used to screen
243. The most acutely devastating when celiac sprue is suspected?
complication of ulcerative colitis is A Shilling's test
probably: B. Anhgiiadin IgA and IgG
A. Massive bleeding. antibodies
B. Mesenteric ischemia. C. IgA tissue transglutaminase
C. Protein-calorie wasting syndrome. antibodies (tTG)
D. Malabsorption and electrolyte D. Withdrawal o f lactose from the
abnormalities. diet to monitor for improvement o f
E. Toxic megacolon. symptoms
E. Scotch tape test
244. Which of the following may
complicate Crohn’s disease? 248. Which of the following is a
A. Haemochromatosis recognised complication of coeliac
B. Rheumatoid arthritis disease?
C. Venous thrombosis A. Fistulae
D. Atrial fibrillation B. Intestinal lymphoma
E. Acanthosis nigricans C. Primary sclerosing cholangitis
D. Toxic megacolon
245. Which of the following is not a E. Uveitis
recognized gastrointestinal
complication of Crohn’s disease? 249. An 18-year-old female is newly
A. Perianal abscess diagnosed with cocliac disease.
B. Colo vesical fistula Which of the following is the most
C. Anal fissure suitable initial management?
D. Small bowel stricture A. Exclusion o f gluten from the diet
E. Diverticulosis B. £xclusiqp o f lactose from diet
I'. Immunosuppressive agents steatorrhea, lie undergoes sm all
D Non-slcroidal unii-inflammatones bowel biopsies. The biopsy report
E Stool-forming drugs such as reveals normal small bowei
loperamide mucosa. Which of the following is
the most likely diagnosis?
250. ralient is at risk of developing A. Tropical sprue
vitamin R12 deficiency as a result B. Postgastrectomy steatorrhea
of a small bowel resection. Which C Whipple’s disease
part of the small bowel Is D. Nontropical sprue
responsible for the absorption of E Abetalipoproteincmia
vitamin B12?
A. Jejunum 254. A 38-year-old man presents
B. Proximal ileum with intensely itchy elbows and
C. Duodenum knees. There were past history of
D. Terminal ileum episodes of malabsorption relieved
E. None ol'the above by a gluten-free diet. The most
likely cause of hb itch is:
251. A 47-ycar-old woman has A. Atypical eczem a
chronic diarrhea and bloating. B Psoriasis-
Associated symptoms include C. Dermatitis herpetiformis.
lethargy and sveight loss. CBC D Scabies.
reveals Hb of 9 3 gm/dL and MCV E. Polycythaemia rubra vera.
70 fL. Which of the following
investigations would be helpful in 255. Regarding abnormalities of
the patient’s diagnosis? stool colour:
A. Anti-mitochondrial antibodies A Melacna can be caused b> a
B. Anti-smooth muscle antibodies bleeding sigm oid colon cancer
C. Anti-tissue transglutaminase B Pale stools may be caused by
antibodies haemolytic jaundice
D. Anti-nuclear antibodies C. As much as 10 mL o f blood is
E. Anti-neutrophil cytoplasmic required to produce the appearance o f
antibodies rndaena
D Lithium therapy causes green
252. Which of the following is most stoob
likely to cause protein-losing E Pale stools may be a sign of
enteropathy? malabsorption
A. Scleroderma
B. Amyloidosis 256. A 58-year-old man complains
C Disaccharidase deficiency of tiredness, fever, weight loss,
D. Ischemic colitis arthralgia and diarrhoea. -Jejunal
E Menetricr’s disease biopsy reveals flattened mucosa
containing periodic acid-8cbiff
253. A 53-year-old niau has weight (PAS) positive macrophages. What
loss, chronic diarrhea, and is the iqost likely diagnosis?
A Coeiiac’s disease D. 1 nmethopnm-Milfamcihoxj/v'lc
B Tuberculosis (TMP-SMX)
C I Topical sprue E. Doxy eye line
D. Parasitic infection
E Whipple's disease 261. A 32-y car-old woman reports
frequent bouts of constipation
257. Osteoporosis is fairly common alternating w ith diarrhea. She
in patients with IBD. Which of the frequently experiences abdom inal
following is not a risk factor for discomfort, w hich is relieved with
osteoporosis? bowel movements. Stress tends to
A. Corticosteroids aggravate h er symptoms. The most
B. Vitamin D deficiency appropriate treatm ent includes
C. Increased physical activity A. Steroid enemas
D. Active inflammation of the bowel B. Mesalammc enemas
E. CaJcium malabsorption C. Peppermint oil
D. Metoclopramide
D ia rrh ea E. None of the above

258. Which of the following 262. What arc/is the long-term side
organisms is responsible for the effects from taking omeprazole as
development of an antacid regularly?
pseudomembranous colitis? A. Hypocalcemia
A. Escherichia coli B. Vitamin B12 deficiency
B. Clostridium difficile C. Clostridium difficile colitis
C. Pseudomonas aeruginosa D. Having a child with birth defects
D. Methicillin-rcsistant (if taken in the first trimester)
Staphylococcus aureus E. Colon cancer
E. Enterococcus faecalis
263. A 24-year-old woman presents
259. Which of the following by dark and tarry black stool. She
medications is used for the has been having episodes of
treatment of pseudomembranous diarrhea, which have resolved with
colitis? the use of Pepto-Bismol. She is very
A. Metronidazole good. The most likely cause of her
B. Amphotericin B dark stools is
C Ketoconazole A. Upper GI bleeding source
D. Acyclovir B. Lower GI bleeding source
E. Intravenous vancomycin C. Rectal outlet bleeding
D. Bismuth ingestion
260. The most effective drug for the E. None of the above
treatment of traveler’s diarrhea is
A. Metronidazole 264. Diverticulosis is a condition
B. Tetracycline associated with
C. Ciprofloxacin A. Increased risk o f colon cancer
13. Herniations o f the bowel mucosa D Primary sclerosing cholangitis
and submucosa through the muscular (PSC)
layers of the bowel wall E. Ulcerative colitis (UC)
C. Inflammatory bowel disease
D. A 90% risk of developing 269. Which of the following is most
diverticulitis correct about neuroendocrine
E. Predominantly the proximal colon tumors of the pancreas?
A. Nearly all the tumors are
265. The most common cause of associated with MEN 1 syndrome
acute diarrheal disease is: B. Other features of MEN 1
A. Food intolerance. syndrome include
B. Toxic contaminants. phcochromocytoma
C. Bacterial disease. C. The most common peptide
D. Parasitic contaminants. expressed is pancreatic polypeptide
E. Viral illness. that causes a watery diarrhea
D. Nonfunctioning islet cell tumors
266. Which of the following is a account for at least 50% ot the tumors
feature of irritable bowel E. Neuroendocrine tumors associated
syndrome? with MEN 1 tend to be solitary and
A. Explosive diarrhoea arc rarely malignant
B. Bloating
C. Rectal bleeding 270. Which one of the following
D. Weight loss represents an optimal screening
E. Steatorrhoea strategy for colorectal cancer?
A. Colonoscopy every 5 years
267. Which of the following is the B. Computed tomographic (CT)
commonest cause of traveller’s colonography every 10 years
diarrhoea? C. High-scnsitivity fecal occult blood
A. E. Coli test (FOBT) every 2 years
B. Entamoeba Histolytica D Sigmoidoscopy every 5 years with
C. Giardia Lamblia high-scnsitivity FOBT every 3 years
D. Shigella Flexneri E. Sigmoidoscopy every 5 years
E. Yersinia cnterocolitica
271. Major risk factors for colon
Gastrointestinal Tumors cancer include all of the following
except
268. Which one of the following A History o f breast cancer
conditions is NOT associated with B Asian descent
an increased risk of colorectal C Inflammatory bowel disease
cancer? D Peutz-Jeghers syndrome
A. Familial adenomatous polyposis E Pnor villous polyps
(FAP)
B. Osier-Weber-Rendu syndrome 272. A63-year-old man has stools
C. Peutz-Jegher syndrome positive for occult blood. Which of
(he following is the most likely C. Amylase is the most sensitive and
location of bowel cancer? specific test for the detection of acute
A. cecum pancreatitis
B sigmoid D. Enteral feedings (distal to the
C transverse colon ligament of Treitz) can be beneficial
D. appendix after 48 hours for severe cases
E. ascending colon E. Nausea and vomiting arc rarely
present.
Pancreatic Diseases
276. You see a 54-) car-old woman
273. A 65-year-o!d man is admitted presents with a week's history of
to the intensive care unit with jaundice and right upper quadrant
severe acute pancreatitis. Which of abdominal pain. Associated
the following is most predictive of symptoms include dark urine and
severity? pale stools. There is no history of
A. His age weight loss and the patient does not
B. Gallstone pancreatitis as a cause consume alcohol. Laboratory
C Persistent organ failure finding show: a bilirubin of 7
D. Pulmonary infiltrates mg/dL, ALT of 40 iu/L, AST 50
E. C-reacdve protein level greater iu/L and ALP of 350 iu/L. The most
than 150 mg/dL at 48 hours likely' diagnosis is:
A. Gallstones
274. A 55-year-old patient with a B. Viral hepatitis
history' of hypertriglyceridemia and C. Alcoholic hepatitis
severe abdominal pain with D. Carcinoma of the head of the
vomiting over the previous 6 hours. pancreas
Laboratory' findings show: • WBC: E Autoimmune hepatitis
20,000 • Glucose: 295 mg/dL •
AST: 333 1U/ L • LDH: 375 RI/L. 277. Features commonly associated
The most likely diagnosis is with the clinical presentation of
A. Acute cholecystitis acute pancreatitis are:
B. Acute pancreatitis A. Jaundice.
C. Hepatitis B. Pseudocyst formation.
D. IM C. Hypercalcaemia.
E. Diabetic ketoacidosis D. Bowel necrosis.
E. Raised albumin.
275. Which of the following
statements regarding acute Miscellaneous
pancreatitis is true?
A. All patients should receive 278. A 45-ycar-old man has known
nasogastric suction to maintain strict cirrhosis, diabetes, and complains
bowel rest. of multiple joint pain. Examination
B. Anticholinergics are useful in the shows a bronze discoloration of the
treatment of acute pancreatitis. < skin - and testicular atrophy.
Laboratory values show a scrum cavity Dpi, soles o f the feet, and
iron of 500 fig /dL, scrum ferritin dorsum o f the hands
of 2,000 ng/mL, and a transferrin D The condition is associated with
saturation of 80%. The most likely inflammatory bowel disease
diagnosis is E The condition is identified by
A Alcoholism elevation in carcinoem bryomc
B. Hemochromatosis antigen levels.
C. Wilson’s disease
D. Gilbert’s disease 280. A 47*ycar-old man presents
E. Hepatitis C com plaining of w eight gain, on
examination there is an abdominal
279. Which of the following distension with a fluid thrill. Which
statements about Peutz-Jeghers of following is not a cause o f ascites
syndrome is true? secondary to venous hypertension?
A. The condition is sex linked and A Congestive heart failure
usually skips a gencration- B Cirrhosis
B The condition involves the C Constrictive pericarditis
development o f multiple polyps in the D Budd-Chiari syndrome
stomach and the small and large E. Nephrotic syndrome
intestine that commonly show
malignant change
C. There is associated
hypeipigmentalion around the oral
4 NEPHROLOGY

C lo n u 'r u lo p a th y C. Complete blood count (CBC)


D. Antistreptolysin O titer
281. A 49-y car-old man with
untreated hepatitis C infection 285. A 38-ycar-old man presents
develops persistent proteinuria. with fatigue, swelling of the eyelids
Which of the folluw’ing diagnoses is and legs, and frothy m ine. Ilis
not typically associated with bloods ap p e ar normal except for an
associated with hepatitis C? albumin level of 2.6 g/dl. You
A. Membranous nephropathy suspect nephrotic syndrome. How
B. Focal segmental would you confirm the diagnosis?
glomerulosclerosis A. Proteinuria on urine dipstick
C. Membranoproliferativc B. >1 g o f protein lost in the urine
glomerulonephritis over 24 hours
D. Cryoglobulinemia C. >1 g o f protein lost in the urine
E Polyarteritis nodosa over 72 hours
D. >3.5 g o f protein lost in the urine
282. Which of the following is not a over 24 hours
manifestation of lithium E. >3.5 g o f protein lost in the urine
nephrotoxicity? over 72 hours
A. Diabetes insipidus
B. Renal tubular acidosis 286. A 21-ycar-old man presents
C. Chronic interstitial nephritis with painless haematuria in the last
D. Hypocalcemia 3 days. He suffers from type 1
E. Nephrotic syndrome diabetes and she is very healthy.
The patient has recently recovered
283. G oodpasture’s syudrome is from a mild throat infection. Urine
associated w ith dipstick analysis reveals blood and
A. Osteoporosis and renal lilhiasis protein in the urine. The most
B. Pathologic fractures and thyroiditis likely diagnosis is:
C. Hepatitis and recurrent cystitis A. Henoch-Schonlein purpura
D. Pulmonary hemorrhage and B. Benign prostate hypertrophy
glomerulonephritis C. IgA nephropathy
E. Pica and angioedema D. Diabetic nephropathy
E. Urinary tract infection (UT1)
284. Which of the following blood |
tests may be helpful in determining 287. A 17-ycar-oId patient present
a rector strep infection in a pa dent with periorbital oedema (3 days
that has a possible ago) and fadgne since a throat
p^ststrepoxx-ceal complication? ■ infecdon 3 weeks ago with nausea.
A urine dipstick is positive for
protein and blood while serum
creatinine and area are mildly week ago. Which of the following
deranged. The most likely diagnosis diagnoses b the most likely?
is: A. Minimal change
A Nephrotic syndrome glomerulonephritis.
B. Nephritic syndrome B. Post-streptococcal
C Renal failure glomerulonephritis.
D. Glomerulonephritis C. Henoch-Sch6nlem purpura
E. Von Grawitz tumour D. Berger’s disease (ls>A
nephropathy).
288. A 19-year-old man is recently E. Rapidly progressive
diagnosed with type 1 diabetes. glomerulonephritis.
Which is most appropriate
investigation suitable for early 291. Which of the following is
signs of renal impairment? NOT typically seen in nephrotic
A. Blood pressure syndrome?
B. Microalbuminuria A. Proteinuria.
C. Serum creatinine B. Hypoalbuimnaeoua.
D. Serum electrolytes C. O edem a
E. Urine dipstick for glucose D. Hypercholcsterolacmia.
E. Haematuria
289. 16 A 51-year-old woman
presents with episodes of 292. Which of the follow ing is NOT
haemoptysis over a 2-week period. a true cause of haematuria?
There are also a history o f weight A. Glomerulonephritis.
loss a feeling of tiredness and B. Ureteric stone.
malaise. Blood tests taken reveal an C. Schistosomiasis.
ESR o f 70mm/hour, creatinine of D. Rifampicin.
3.5mg/dL and haemoglobin of E. Malaria.
10Jg/dL. Which of the following
investigations b most likely to be 293. A 46-year-old woman develops
positive? nephrotic syndrom e and b
A. c-ANCA. aw aiting further tests to estabUsh
B. Anti-glomerular basement the underlying aetiology. In which
membrane. circumstance w onld corticosteroids
C. Anti-SCL 70. be most effective in reversing the
D. Anti-mitochondrial antibodies. nephrotic syndrome?
E. Rheumatoid factor. A. Membranous nephropathy
B. Minimal change disease
290 A 10-year-old boy is brought to C. Primary amyloidosis
hospital complaining o f passing , D Renal vein thrombosis
very dark smoky urine and his fact i E. M cscczai IeA disease
has become more swoDen over the I
Last couple of days. He tells you Rena! Failure
(hat he had a had boat o f 'flu' a * '
2*>4. Vmuii.t that iv >ceit in pa(icii(> 297. Analysis of (lie urine from a
with chronic i vital disease is patient with acute renal failure
ouult> caused Iw insufficient shows pigmented granular easts.
\ lu'U MOIY' What »s the cause of the patient’s
H N u.inunBl-' acute renal failure?
C' Rcmu lewis A Acute glomerulonephritis
p I nthiopoictm lewis B Acute interstitial nephritis
1' hol.itc Mores C \eute tubular necrosis
P Prerenal a.vtemu
295. A BUN:ercatininc level greater i\ RfuhdoimoUsis
than 20 is associated with t
\ LVh>Jration 298- What is the most common
B. Renal stones complication of hemodialysis?
C Bladder outlet obstruction A Air embolism
P fh p avalccm ia B. Hyperphosphatemia
E. Renal artery stenosis C Hypocalcemia
D. Hypoglycemia
296. Which of the following E. Hypotension
statements about the
pathophysiology of acute renal 299. Which of the follow ing is NOT
failure is TRUE? an indication for emergent
A. Damage to the nephrons from dialysis?
cytokine mediated alteration of nitric A. Blood urea nitrogen (BUN)
oxide has been implicated in the greater than 1OOrag/dL
pathophysiology of intrinsic renal B. Lithium poisoning
failure and prerenal failure, but not C. Severe metabolic acidosis
postrenal failure. D. Uremic encephalopathy with
B. Intrinsic renal failure occurs when seizures
renal perfusion is decreased leading E. Uremic pericarditis
to ischemia o f renal parenchyma.
C. Patients with chronic hypertension 300. Which of the following is not a
require lower blood pressures to potential cause of obstructive renal
develop intrinsic renal failure when impairment?
compared with patients without A. Benign prostatic hypertrophy
chronic hypertension. B. Recurrent kidney stones
D. Tubular and glomerular function C. Retroperitoneal fibrosis
are not maintained in prerenal failure. D. Schistosomiasis
E. Use of radiocontrast agents and E. Systemic sclerosis✓
aminoglycosides during a time o f
decreased renal perfusion does not 301. A 74-year-old tnau with a
lead to further renal injur)’ in most history of moderate congestive
cases. cardiac failure is admitted with
acute renal failure. His potassium
v level is noted to be 5.5 mmol/L.
Which of hi* following medications E. Renal parenchymal unc acid
may be contributing to the raised crystals
scrum potassium level?
A Amlodipinc 305. A 54-ycar-old man with
B Bendroflumethiaridc chronic renal failure (CRF) is seen
C. Iiirosemidc with a normocjtic anaemia (Hb ‘>_5
D Simvastatin g/dl). Which of the following is
E. Spironolactone NOT associated with CRF?
A Anorexia
302. Which of the following Is NOT B Nausea.
an indication for haemodialysis in C. Restless legs.
patients with acute renal failure? D. Hypokahemia.
A. Acidosis with a pH <7 .2 E. Hyperpbosphataemia
B. Hypertension >220 mmHg systolic
or 160 mmHg diastolic 306. Which of the following states is
C. Persistent potassium of >7 mmol/L NOT an indication for renal
D. Refractory pulmonary oedema replacement therapy (dialysis)?
E. Uraemic pericarditis A Refractory metabolic acidosis
B. Oliguria.
303. A 53-year-old woman with C Severe hypcrknlaemia
chronic renal failure develops D Uraemic symptoms.
hyperphosphatemia and E Drug ingestion.
hypocalcemia. Which of the
following findings b most likely 307. Which of the following b NOT
associated with this electrolyte routinely considered as part o f a
dbturbance? renal screen in the investigation of
A. lethargy new-onset renal failure?
B. neuromuscular irritability A. Complement.
C. anorexia B. Renal ultrasound.
D. tachyarrhythmias C Caeruloplasmin and serum copper
E. hyperkalemia D. Anti-ncutrophil cytoplasmic
antibodies.
304. A 63-ycar-old man with an 8- E Bence- Jones protein
ycar history of recurrent severe
arthritis in his brge toes has an 308. Oliguria m ore likely to be due
elevated creatinine level. Which of to prerenal failure than intrinsic
the following mechanisms is the renal failure if:
most likely expbnation for his A. Urine free o f red blood cells or
renal impairment? casts
A. GN B. Urine plasm a urea ratio <3
B. Vascular injury C. Urine osmolality <350 mOsm/l
C. Uric acid kidney stones D. In the presence o f hypertension,
D. Distal tubular atrophy raised JVP and good
■*periphcralcirculation
E Urinar) soduun >lOmM B. Urmc pH that is normal when
plasma bicarbonate levels are normal
309. Which of the following is a C. Chronic metabolic alkalosis
recognised cause of acute renal D. Plasma bicarbonate levels that arc
failure? easily restored with supplementation
A. Bums E Hyperkalemia
B Dermatoinyocvtis
C Duchcnne muscular dystrophy 313. A patient invoked in a serious
D Penicillin therapy motor vehicle accident has a
E Alport's Syndrome positive dipstick for
hemoglobinuria, but erythrocytes
310. A 53-year-old man with HIV are not noted on microscopic
suffers massive intraabdominal examination. The most likely
hemorrhage and then recovered by diagnosis is
operative interference. One day A Myocardial contusion
after the operation, he becomes B Rhalxiomyolysis
oliguric with mildly elevated urea C Intravascular hemolysis
aud creatinine. After 1 week, he D. RenaJ contusion
becomes polym ic with a GFR o f 30. E. Laceration of the spleen
The most likely diagnosis is:
A. Haemolytic-uraemic syndrome 3 N . A 66-year-old woman ou
B. Acute tubular necrosis cnalapriJ a month ago for her heart
C. SIADH failure. She is felling okay, but feels
D. HIV nephropathy that her ankles arc swollen. Her
E. Acute renal failure scrum creatinine level is 2.6 mg/dL
and potassium level is 5 8 mEq/L.
Miscellaneous Her baseline values were normal 2
months ago. Which one of the
311. Which o f the following following is a side effect of ACE
treatments would be least likely to inhibitors that is the most likely
reduce the risk of recurrent stone cause of these changes in renal
formation in a patient with known function?
calcium oxalate nephrolithiasis? A. Toxicity to the proximal renal
A. Increasing fluid intake tubules
B. Decreasing sodium intake B. Impaired uutoregulation of
C. Increasing calcium intake glomerular blood flow
D. Decreasing calcium intake C Microangiopathic arteriolar
E. Decreasing oxalate intake thrombosis
D. Rhabdomyolysis
312. Type 11 rcual tubular acidosis E. Interstitial nephritis
(RTA) is associated with
A. The proximal tubules having 315. In which of the following
decreased ability to absorb circumstances should angiotensin-
bicarbonate converting enzyme (ACE)
inhibitors be avoided where C Loop diuretic^ and potassium-
possible? sparing diuretics
A Glomerulonephritis D. Ia>op diuretics and thiazides
13 Lupus nephritis F Thiazides and acetazolamide
C Renal artery stenosis F. Thiazides and potassium-sparing
D Systemic sclerosis with renal diuretics
involvement
E All of the above 319. Which of the following is NOT
a recognised cause of acute
316. A 43-year-old man has just luhularnecrosis?
been diagnosed with polycystic A. Rhabdomyolysis
kidney disease. Which of the B. Paracetamol poisoning
following is not an extrarenal C. Hypovolaemia
complication of polycystic kidney D. Hypertension
disease? E Corticosteroid therapy
A. Bladder diverticuli
B. Cerebral aneurysms 320. A 16-ycar-old boy presents
C. Hepatic cysts with fatigue and a one w eek history
D. Mitral valve prolapse of low-grade fever, frequency; and
E. Pancreatic cysts arthralgia. A urine dipstick shows
trace proteins, while a blood test
317. A 55-year-old woman with 10- shows rabed eosinophils. T he most
year history of type 2 diabetes likely diagnosb b:
treated with glibenclamide. Her A. Acute tubulointerstitial nephritis
blood pressure b 148/93 with new B Renal failure
onset proteinuria. A renal biopsy C Diabetes mellitus
shows the presence of Kimmclstiel- D Urinary Tract Infection
Wibon lesions. The most E Reactive arthritis
appropriate management b:
A. Increase oral hypoglycncmic 321. A 58-year-old hypertensive
dosage man presents with pitting oedema
B. ACE II antagonists of hb ankles. Blood results show
C. Start cholesterol lowering therapy low albumin and there was
D. Start ACE inhibitors albuminuria. The most appropriate
E. Start renal dialysis initial treatment is:
A High protein diet
318. Monitoriug acid-base status is B. Diuretics
very important in individuals w ith C. Prophylactic anticoagulation
kidney pathology. Which of the D ACE inhibitor
following diuretics causes E Bed rest
metabolic alkalosis?
A. Acetazolainide and potassium- 322. A 55-year-old woman with a
sparing diuretics 10-year history of DM-type2
B. Loop diuretics and acetazolamidt; treated with glibenclamide present
with nov onset proteinuria. Her A Analgesic nephropathy
blood pressure is 148/93, with B. Renal infarction
elevated lipid levels, S e a t e d C Hypcruricacmic nephropathy
haemoglobin of 5J>% and fasting D. Acute tubulointerstitial nephritis
glucose of 110 tng/dL. A renal F Chroruc renal failure
biopsy shows the presence of
Klmnielsliel-Wilson lesions. The 324. A 21-year-old man complains
most appropriate management is: his urine has turned a faint red in
A. Increase oral hypoglyeaemic the last week. l l a \ e no other
j osage medical problems except for
B. ACE II antagonists sensorineural deafness diagnosed
C. Start cholesterol lowering therapy when he was young. On
D. Start ACE inhibitors examination, you notice retinal
E. Start renal dialysis flecks and urine dipstick confirms
protein and blood. The most likely
323. A 64-year-old man is diagnosis is:
undergoing treatment for A Alport's syndrome
polycythaemia vera with B. Benign familial hacmaruria
chemotherapy. Shortly, the patient C. Wolfram syndrome
becomes lethargic, feels unwell and D. IgA nephropathy
suffers weight loss. After 2 weeks, E. Down's syndrome
the patient becomes oliguric,
complains of bilateral flank pain
and becomes oedeinatous. The most
likely diagnosis is:
1 CARDIOVASCULAR DISEASE
Hypertension & Hypotension
I. T he uniw-er h F. The initial evaluation of a patient with hypertension should
include a thorough history and a number o f tests, including the following
Routine tests
• CBC • Chemistry panel, including fasting, glucose, potassium ,
creatinine, and BUN • Cholesterol panel (total cholesterol and HDL/LDL
cholesterol)
• 1 2-lead ECG ‘ Urinalysis
O ptional tests
• Creatinine clearance • 24-hour urinary' protein * Uric acid • Glycosylated Mb
• TSH • Limited echocardiography • Chest x-ray
Hypertension exists when the diastolic blood pressure is consistently m easured >90
mm Hg and a systolic blood pressure remains >140 mm Hg. Initial m anagem ent
should consist o f sodium restriction, limitation of alcohol consumption, and a
regular exercise program. Overweight patients should be counseled to lose weight.
Other tests to assess kidney function, cardiac perform ance, or endocrine
abnormalities ( e .g , pheochromocytoma) arc usually unnecessary. It is strongly
recommended that physicians intervene with lifestyle modifications that can
prevent or delay the onset of hypertension in pauents.

2. T he answer is A. Thiazide diuretics (e g., chlorthalidone, hydrochlorothiazide)


lowered mortality and morbidity from stroke, heart attack, and heart failure m ore
than [J-blockers. ACE inhibitors (e.g., lisinopril) and CCBs (e.g.,am lodipine)
reduced mortality and morbidity as much as thiazide diuretics, but the evidence is
less robust Because the use of thiazide diuretics is supported by a strong body o f
evidence and no other class o f antihypertensive medications has been show n to be
better at improving outcomes, they are the first-line drugs for most patients w ith
hypertension. Doxazosin is on a-blocker and is not indicated for first-line
treatm ent Losartan is an angiotensin-receptor blocker (A RB), which should be
reserved for patients who cannot tolerate ACE inhibitors. Unfortunately, the current
evidence does not support using ^-blockers as first-line therapy for hypertension.
Diuretics remain the preferred first-step drug and an important part o f any
multidrug regimen for the treatment o f hypertension. A CE inhibitors and CCBs
appear to be as effective as thiazide diuretics, but the evidence is not as strong.

3. T he answer is E. Systolic hypertension is a condition that usually affects the


elderly. The condition is defined as a systolic Wood pressure >140 mm Hg and
diastolic pressure <90 mm Hg. The cause is the loss o f elasticity o f the arteries that
occurs with aging. Other causes include thyrotoxicosis, arteriovenous fistulas, or
aortic regurgitation. Untreated systolic hypertension can lead to an increased risk
lor stroke and CVD It is mportant ^ sk factor in the development of coronary heart
disease (CUD) In other words, a systolic blood pressure o f 160 mm llg with a
diastolic blood pressure of 85 mra Hg posed a greater risk, for CVD than a systolic
blood pressure of 135 to 140 mm Hg and a diastolic blood pressure of ^5 mm Hg
Isolated systolic hypertension, defined as a blood pressure o f >140 nun Hg systolic
and <90 mm Hg diastolic, occurs in more than 30% ofwvnien older than 65 year*
and in more than 20% of men of the same age. Treatment involves lifestyle
changes ( e g , exercise, sodium restriction, weight loss) and the use of hypertensive
medications. In most instances, low-dose diuretic therapy should be used as initial
antihypertensivc therapy in the elderly. A longacling dih>drop>ndinc CCB may be
used as an alternative thcrap> in cldcrlv patients with isolated svstolic
hypertension. Diabetics benefit from ACL' inhibitors. In some cases, svstohe
hypertension may be more difficult to control than essential hvpatensKVi.

4. The answer b B. ACE inhibitors (e.g , captopriL caalapril. lisinopnl. ramipnD


function as afterload reducers by inhibiting the rcuiwmgiotctian-aldostei\.''ne
system. An ACE inhibitor or ARB is clearly preferred as initial therapy in any
hypertensive diabetic patient who has moderately mexeased albuminuria or
severely increased albuminuria to slow renal disease progression. However, most
experts begin treating hypertension with an ACE inhibitor or .ARB in diabetics
even in the absence o f proteinuria. The mechanism of action o f ACE inhibitors
evolves the blockage o f angiotensin I to angiotensin LL resulting in a decrease in
- 'Steronc production, which leads to increased soiuim and water excretion.
K;. lodynamk: effects include decreased peripheral resistance, increased renal
blcod flow, and minimal effects on cardiac output mvI glomerular filtration rate.
Adverse effects include headaches, nausea, skin rashes, nonproductive
T.tativc cough (10% to 20% of parents), acute renal failure in patients with renal
cry stenosis, and angioneurotic edema. ACE inhibitors are generally not
ociaied with depression, sedation, fatigue, or impotence. They can be useful in
■serving renal function in patients with diabetes. Those patients with preexisting
. j ! insufficiency or renal artery stenosis require close monitoring o f renal
ction when ACE inhibitors are administered. Patients with CHF, diabetes,
pheral vascular disease, history o f recent MI, hyperlipidemia, and renal
efficiency are good candidates for ACE inhibitors.

hc answ er b A. If intermittent claudication is the only symptom, the extremity


appear normal, but the pulses are reduced or absent. The level of arterial
usion and the location o f intermittent claudication closely correlate (e.g.,
i ac disease frequently causes claudication in the buttocks, hips, and calves,
ic femoral pulses are reduced or absent). In men, impotence is common and
do ids on the location and extent of occlusion.
A ough arterial occlusion in the extremities can usually be diagnosed clinically,
noninvasive tests confirm the diagnosis and are useful in follow-up. Invasive tests
car. it ument the location and extent o f disease if angioplasty, local fibrinolytic
iherr; v. or surgical bypass is contemplated. Doppler ultrasonography is most
widely used ArUiiul stenosis and occlusion can be detected using a velocity
detector (Doppler probe) A colored signal shows the direction of flow (color
Doppler) I lie simplest method for estimating blood flow to the lower extremities
is to compare systolic BP ut the level of the ankle with brachial systolic pressure
(ankle-brachial indices). During this procedure, a blood pressure cuff is applied to
the ankle, inflated above brachial systolic pressure, and deflated slowly. Ankle
systolic blood pressure can be obtained accurately with a Doppler probe placed
over the dorsalis pedis or posterior tibial arteries. This blood pressure at rest
normally is >90% o f the brachial systolic pressure; with mild arterial insufficiency,
it is 70% to 90%; with moderate iruufficicncy, 50% to 70%; and with severe
insufficiency, ^50%

6. The answ er is A. 0-Blockers are used as first-line therapy for the treatm ent o f
uncomplicated hypertension. These agents decrease the heart rate and cardiac
output, pi-Adrenergic receptors are located in the cardiac muscle, w hereas 02-
adrcnergic receptors are located in the bronchia] musculature. Adverse effects
include exacerbation of bronchoconstnction in asthmatics because o f 02-receptor
blockade, bradycardia, left ventricular failure, nasal congestion, nightm ares,
Raynaud’s phenomenon, fatigue, depression, cold extremities, and im potence. 0-
Blockers are also associated with elevated triglycerides and decreased HDL
cholesterol; however, there is not enough effect on lipids to discourage their use in
select cases. These agents arc contraindicated in patients with poorly controlled
diabetes, second-or third-degree heart block, or moderate-to-severe asthm a;
however, these agents have been shown to improve survival after MI and in select
patients with CHF. These agents have also been used for the treatment o f “stage
fright” and as migraine prophylaxis 0-Blockers should not be discontinued
abruptly because o f the risk of rebound hypertension.

7. The answ er is C. Orthostatic hypotension is defined as a decrease in blood


pressure o f at least 20 mm Hg (systolic) and at least 10 mm Hg (diastolic) that
occurs when moving from a supine to an upright position. Also, orthostatic
tachycardia is defined as an increase in heart rate >27 bpm or to a level >108 bpm.
Measurements o f blood pressure and pulse rate should be taken after the individual
has been in an upright position for 3 minutes. Causes o f orthostatic hypotension
include volume depletion, medications ( e .g , tricyclic antidepressants,
antihypertensivc agents), and autonomic dysfunction (as seen in diabetic patients).
Elderly patients are at increased risk, and syncope may resu lt Treatm ent involves
volume replacem ent discontinuation o f offending pharmacologic medications, and
slow positional changes.

8. The answ er is D. Causes for secondary hypertension are numerous and include
the use o f oral contraceptives, excessive alcohol consumption, disorders o f the
renal parenchyma associated with malfunction o f the renin-aldosierone system,
Cushing’s syndrotpe, gheochromocytoma, primary aldosteronism,
hyperthyroidism, myxedema, renal vascular disease, and coarctation of the aoita. In
many cases, blood pressure may be difficult to control. Physical examination may
reveal abdominal bruits, suggestive o f renovascular hypertension, or other findings
suggestive o f contributing disease. Adequate treatment is necessary to prevent the
long-term detrim ental effects of hypertension. Abrupt discontinuation o f (l-blockcrs
is associated with rebound hypertension.

9. The answ er is D. Stiffened blood vessels also haw an impact tVr blood pressure
determination m later life. Systolic blood pressure rises throughout life in Western
populations, whereas diastolic pressure peaks and plateaus in middle age and later
life. “Normal" blood pressure has been defined by detemuning the cardiovascular
risk associated with a given blood pressure. The presence o f an isolated rise in the
systolic pressure without a diastolic rise (isolated systolic hypertension) is fairly
unique to older patients and, unlike younger patients, docs not necessarily imply
anemia, thyrotoxicosis, or aortic insufficiency, which can cause a bounding pulse
and wide pulse pressure in the young.

10. The answ er is E. Extermination o f orthostatic hypotension should be routinely


performed in geriatne patients. Although a number o f factors, such as declining
harorcccptor j*:nxitivitY, diminished arterial compliance, increased venous
tortuosity, decreased rectal sodium conservation. and diminished plasma volume,
could comhinc to cause a drop in orthostatic blood pressure among older patients,
there is no clear evideocc that the pressure drops solely as a function o f age.
Howes cr, a hlood pressure drop when changing f r o m the supine to the upright
position is com mon among geriatric patients (possibly as many as 30°o of
unsclcvtcd patients may experience a 20 min Hg or more drop in sysiolic pressure)
Diseases and medications that causae the problem ire common offenders.

11. The an sw er is B. Hydrochlcrthiazide does not reduce insulin resistance. In


susceptible individuals, hetz can precipitate type 11 diabetes mellitus, albeit
reversible upon discontinuance- Hetz is a distal loop diuretic, which is a quite
serviceable antibypertensivr. tending to function thereby in people who liavc a
tendency to retain salt and whose hypertension may respond to dietary salt
restriction. Hetz docs cause excretion of sodium and potassium, the latter to
pathological proportions on occasion. Hetz causes a slight elevation o f serum
calcium and ia Unit respect tends to preserve bone calcium, for example, in ihose
llu t might be sifciccplible to osteoporosis.

12. The answ er is C. A calcium channel blocker (C) is first-line therapy in a


female who intends to/or has become pregnant, us well as in patients who are of
Atrican-Ciiribbcan descent (with an ACE inhibitor). In a normal patient following
lifestyle therapy ACE inhibitors (A) arc first line, calcium channel blockers (C) are
second line followed by a combination o f diuretics (D) and calcium channel
blockers 13-blockers $E) are used if despite the aforementioned therapy target
blood pressure is nol met If there arc any contraindications or poor compliance
with ACT. inhibitors, angiotensin II receptor blockers (D) are used instead.

13. The answ er b (E). Captopnl is an inhibitor o f angiotensin converting enzyme


and impairs production of angiotensin II, whtch is a potent vasoconstrictor.
Through removal of the feedback mechanism, renin secretion is increased.
There arc additional antihypertensive effects from reduction o f bradykinin
degradation and stimulation of vasodilating prostaglandin production. Converting
enzyme inhibitors such as captopril can be added to beta-adrenergic blockade
therapy to achieve additional antihypertensive effects. Captopril is contraindicated
in patients with bilateral renal artery stenosis, since a reduction in systemic artenal
pressure may lead to progressive renal hypoperfusion.

14. The answ er is (D). Liddle's syndrome is typically asscoiated with


hypokalaemic hypertension and low renin andaldosterone concentrations -the so
called pscudo-hyperaldosteronism. Barter's syndrome isassociated with
hypokaiacmia though hypertension is not a feature. Intype IV RTA, there isa
hyporeninaemic hypoaldosteronism, which m ay also be produced withdiabetic
nephropathy. Hence hypeikalaemia is more typical.

15. The answ er b (C). The presence of fever, facial rash and arthralgia suggest a
diagnosis o f drug-induced SLE, with hydrailazine being a well recognised cause.
Gout may beprecipitated by bendrofluazide and it also causes a photosensitivity
rash but the two diagnoses together with a fever would be remote

Coronary A rtery Diseases

16. The answ er b (A). It is sometimes said thni questions longer than 2 lines are
usually false ... but not in this case. The posterior descending artery is m ost ofteo
(85%) a branch o f the right coronary artery. The sinus node artery is a branch o f the
right coronary artery in 60% o f cases. The AV node is supplied from the posterior
descending coronary artery. The left main stem is much shorted than 4 cm!

17. The an sw er b B. Streptokinase is a thrombolytic agent adm inistered during


Ml. The majority of patients with an acute ST elevation myocardial infarction
(STEM1) have a complete occlusion of a coronary artery due to thrombus. Thus,
thrombolytic therapy has been important treatment option. However, primary
percutaneous coronary intervention (PCI) is the preferred approach for reperfusion
lor most patients with STENll as there are better outcomes (lower mortality and
less recurrent ischemia) with tewer complications such as intracranial hemorrhage
that occurs with thrombolytic use. However, such treatment remains an important
reperfusion strategy in locations with limited availability o f timely PCI.
Streptokinase is associated with a 1% to 2% rale o f allergic reactions consisting of
skin rashes and fqyer and hypotension occurs in another 10%.
rhe criteria for consideration o f thrombolytics include chest pain (consistent with
cardiogenic pain) for at least 30 minutes’ duration and ECG changes that show at
least 1 to 2 mm o f ST elevation in two adjacent prccordial leads. Medication should
be given within 12 hours for maximal benefit. Although extremely variable
depending on the source, the following is a list o f absolute contraindications to
thrombolytics:
• History o f intracranial hemorrhage
• Other strokes or cerebrovascular events within 1 \ car
• Known intracranial neoplasm
• Unclear mental status
• Active gastrointestinal (Gl) bleeding
• Aortic dissection
• Acute pericarditis
Relative contraindications include
• Recent surgery (writhin 3 weeks)
• Prolonged (>10 minutes) cardiopulmonary resuscitation
• Recent vascular puncture in a noncorapressible region (<2 weeks)
Prior stroke (nonhcmorrhagic)
• Uncontrolled hypertension defined as systolic blood pressure (SBP) > 180 mm
Hg or diastolic blood pressure (DBP) >110 mm Hg
Major surgery (3 months or less)
• Pregnancy
• Bleeding d ms thesis
• Active peptic ulcer disease
Minor hemorrhage, menstruation, and diabetic retinopathy arc not contraindications
to fibrinolytic therapy.*•

18. The answ er is A. Risk factors for MI include:


• Hypertension
• Hyperlipidemia— particularly high total cholesterol, high LDL cholesterol, and
low HDL cholesterol
• Cigarette smoking
• Diabetes m dlitus
• Obesity (increased weight for height)
• Male gender
• family history o f CAD
• Sedentary lifestyle
• Type A personality
• Increased age
• Postmenopausal
• Homocystinemia

19. The answ er is A. Dressler’s syndrome, or postmyocardial infarction syndrome,


occurs seyeral days to several weeks after Ml. The condition is characterized by
chest pain, fever, pericarditis with a pericardial friction rub, pcncardial effusion,
pleurisy, pleural effusions, and multiple joint pain The cause is thought to be an
autoimmune response to the damaged myocardial tissue and pericardium The
difference between Dressier’s syndrome and recurrent MI is difficult to determine,
however, in Drcsslcr’s syndrome, there is minimal or no increase in cardiac
enzymes. Treatment includes the use of aspirin, NSAIDs, and, in some eases,
corticosteroids.

20. T he answ er is A. (J-Blockcrs reduce mortality during both acute and long-term
management o f MI. Administration of intravenous P-blockers within 12 to 24 hours
of infarction, followed by oral therapy, has been found to significantly reduce the
mortality rate within the first week of infarction. The most marked reduction occurs
in die first 2 days after infarction. Initiation o f (J-blocker therapy within days to
weeks alter infarction and continuation of therapy has been shown to reduce total
mortality, nonfatal MI, and sudden death. This has been shown regardless of the
patient’s age or sex, infarct location, and initial heart rate, or the presence or
absence o f ventricular arrhythmias. The greatest benefit occurs in high-risk
patichts, including the elderly and those with large anterior infarctions,
arrhythmias, or left ventricular dysfunction.

21. T he answ er is C. The natural progression o f ECG changes seen with MI


include peaked hyperacute I-waves to ST-segment elevation, to Q-wave
development, and to 1 -wave inversion. In antcroseptal infarction, ECG changes arc
usually noted in leads VI through V3. Q waves indicate a transmural infarcL

22. T he answ er is C. Guidelines have been published to help treat patients with
hyperlipidemia. Total cholesterol levels should be kept <200 mg per dL, with HDL
cholesterol (good cholesterol) >40 rag per dL. Further recom m endations arc
divided into those patients with CHD and those without CHD. CH D equivalents
include peripheral arterial disease, AAA, symptomatic carotid arterial disease, and
diabetes meliitus. For patients without CHD and fewer than rwo risk factors, LDL
cholesterol should be kept <160 mg per dL. For those with two o r m ore risk factors
and no CAD, the goal is an LDL level <130 m g per dL. For patients who have
CAD, the new recommendations give a goal for LDL cholesterol <100 mg per dL
with <70 mg per dL as being optimal. Triglyceride levels are not as strongly
associated with CAD but should be kept <150 mg per dL. Risk factors for CHD
include the following:
• Age: men > 45 years; women > 55 years or with premature m enopause without
estrogen replacement * Family history o f premature CH D in first-degree
relative • Smoking • Hypertension • HDL cholesterol <35 mg per dL
• Diabetes • Obesity • History of cerebral or peripheral vascular disease
• A negative risk factor includes an HDL cholesterol level > 60 mg per dL
23. The answ er is B. Angina pectoris is typically described as substern ai chest
pain or pressure that may radiate to the neck, jaw, or left arm. Patients usually also
experience shortness of breath, dizziness, nausea, and vomiting with diaphoresis.
Symptoms are usually precipitated by physical exertion or stress and arc relieved
with rest Episodes usually last 2 to 10 minutes and rarely last >30 minutes.
Atypical presentations include epigastric pain, indigestion, right-arm pain, light-
headedce&s, nausea, or shortness of breath. These occurring alone arc referred to as
anginal equivalents. There are several types of angma:
» Stable Intensity, character, and frequency of episodes are predictable, angina
occurs in response to a known amount of exercise or stress.
♦ Unstable: Intensity, frequency, and duration are different and unpredictable; pain
is precipitated by a lesser amount of exercise or the angina is longer in duration.
Angina at rest or new-onset angina is unstable.
« Variant: Pain that may occur at rest and is secondary to spasm of the coronary
arteries is variant angina.
Typically, the pain is relieved with the administration of sublingual nitroglycerin
ECG may show T-wave inversion or ST-segment depression, but in many cases is
normal and should not be discounted if normal. Exercise stress testing can be used
to determine coronary insufficiency. Treatment of angina is accomplished with the
use of nitrates, (3-blockers, and CCBs.

24. The answ er is E. The term acute coronary syndrome refers to a range of
thrombotic CADs, including unstable angina and both ST-segment elevation and
nou-ST-segment elevation ML Symptoms of acute coronary syndrome include
chest pain, referred pain, nausea, vomiting, dyspnea, diaphoresis, and light­
headedness. Pain may be referred to the arms, the jaw, the neck, the back, or even
the abdomen. Pain radiating to the shoulder, left arm, or both arms increases the
likelihood o f acute coronary syndrome. Typical angina is described as pain that is
substemaL occurs on exertion, and is relieved with rest. Diagnosis utilizes an ECG
and a review' for signs and symptoms of cardiac ischemia. In acute coronary
syndrome, common electrocardiographic abnormalities include T-wave tenting or
inversion, ST segment elevation or depression (including J-poinl elevation in
multiple leads), and pathologic Q waves Most high-risk patients should be
hospitalized. Intermediate-risk patients should undergo further evaluation, often in
a chest pain unit. Many low-risk patients can be discharged with appropriate
follow-jp.
Troponins are regulatory proteins found i n skeletal and cardiac muscle. Three
subuarj have been identified: troponin 1 fFni;, troponin T fTnT), and troponin C
'TnC;. The skeletal and cardiac s a b - f o r m s f o r TnC are similar but Tnl and TnT are
chsthset. According to the American College of Cardilcgy guidelines, any elevated
measure of troponin above the 99* percentile upper reference limit in the
appropriate clinicai setting b defined as an ML
Troponin T and 1 have similar sensitivity arid specificity for the detection of
myocardial injury Unlike troponin f levels, rxoponin T levels may be elevated in
patients with reml disease, polymyositis, or dermatomyositis. The cardiac
troponins may remain elevated up to 2 weeks after symptom onset, which makes
them useful as late markers of recent acute Ml. An elevated troponin T or I level is
helpful in identifying patients at increased risk for death or the development ot
acute Ml Increased risk is related quantitatively to the scrum troponin level.

25. T he answer is B. Exertional angina (chest pain) is the most common


manifestation of myocardial ischemia in young and middle-age persons. Because of
their more sedentary lifestyle or possibly a difference in pathophysiology, this may
not be true in elderly patients. Instead of exertional chest pain, ischemia may be
more commonly manifested as dyspnea in elderly patients. Other elderly patients
with coronary artery disease (CAD) may be completely asymptomatic, although
silent ischemia may be demonstrated by stress testing or Holter monitoring.

26. T he answer is E. Transient hypotension is clinically the most important


complication of nitroglycerin use, although headache is a very common side effect
that usually responds to acetaminophen. Nitroglycerin should be avoided or used
with extreme caution in preload-dependent conditions such as aortic stenosis,
HCM, and right ventricular infarct. Nitroglycerin has caused methemoglobinemia
in very rare instances. Nitroprussidc can cause thiocyanate toxicity, especially
when used in patients with renal insufficiency.

27. The answer is B. The anterior wall o f the left ventricle and the inter-ventricular
septum. The left anterior descending artery normally supplies the anterior wall o f
the left ventricle and the inter-ventricular septum. It arises from the left coronary
artery, as does the circumflex artery, which supplies the posterior and lateral sides
of the left ventricle. The right coronary artery usually supplies the sino-atrial node,
atrio-ventricular node, right ventricle and inferior part o f the left ventricle-

28. T he answer is C . Acute coronary syndrome is a spectrum of cardiac


ischaemia-infarction determined by the presence o f two out o f three factors: chest
pain, ECG changes and cardiac enzyme rise. Depending on these results, p»rirftK
will fall into one of the following categories: unstable angina. NSTEMI or STEML
Inverted T w a v e s (A) and ST depression (B) are signs o f ischaemia. ST elevation.
Q waves and raised troponin are indicative of infarctioo. Initially. ‘ST devaLion' or
‘non ST elevation’ ECG changes arc used to stratify each patient's risk as the
results o f blood tests for troponin levels (E) (which should be carried out 12 hours
after the pain started) are not known and Q waves hav e not ?imr to develop ST
elevation ( C ) is a very good predictor o f imminent infarction (positive troponin).
However, if tins patient is treated quickly enough with thrombolyses oc primary
PCI, infarction can be avoided. A patient with STEM! who goes on to have
negative troponin is termed to have had an 'aborted M l’. Q waves (D» (indicating
full-thickness MT) take time to develop, so *Q wave’ or ‘con Q w ave’ Mi is a
diagnosis given op discharge.

DO
---------------- , — 1-------
ECG: ST elevation
+VC -vc
Troponin +ve STEM1 NSTEM1
-ve Aborted Ml Unstable

29. The answer Is (A), a) True, b) False. This is more consistent with a strained
muscle, c) False. This is a normal examination finding. J) False. This raises the
possibility of myocardial infarction, e) False. This is more suspicious o!
costochondritis.

30. T he answ er is (B) a) False, b) True, e) Fabc. Fhis presents with $F elevation
in all leads, d) False. ST elevation m multiple leads is always significant e) False

31. T he answ er is (C). Myoglobin levels are often elevated in patients with renal
failure because of decreased clearance. A single troponin measured on CD
presentation of a patient has limited utility in excluding an acute Ml. Cardiac
troponins have no ability' to exclude unstable angina without myocardial infarction
because cell injury is required to elevate the troponin and because of the tune delay
associated with the rise in levels. Total CPK. levels are not as specific as cardiac
troponins for identifying myocardial cell injury. CPK-MB fraction is associated
with false-positive test results, and may occur in certain clinical conditions such as
pericarditis and myocarditis.

32. T he answ er b (B). Nitroglycerin provides benefit to patients with an acute


coronary syndromes by decreasing myocardial preload and to a lesser extent
afterload. Nitroglycerin increase venous capacitance leading to venous pooling, a
decrease in preload and myocardial oxygen demand. Coronary vasodilation may
improve blood flow to ischemic myocardium.

33. T he answ er b (B). Qopidogrel prevents platelet aggregation through


antagonism o f the ADP receptor. It has been shown to reduce mortality’ from stroke
and 1HD in primary preventionstudies

A nythin ias

34. The answ er b C. Adenosine, digoxin, and calcium-channel antagonists act by


blocking conduction through the atrioventricular (AV) node, which may increase
the ventricular rate paradoxically, initiating ventricular fibrillation. These agents
should be avoided in Wolff-Parkinson-White syndrome. Procainamide is usually
the treatment of choice in these situations, although amiodarone may also be used.

35. The answ er b A. A prolonged QT interval is a common entity associated with


sudden arrhythmia death syndrome. Arrhythmias may be induced in normal hearts
by medications; electrolyte abnormalities (e g., hypokalemia, hypomagnesemia);
myocarditis, and endocrine, CNS, or nutritional disorders. These arrhythmias arc
associated with prolongation of the QT interval. A group of inherited gene
mutations has been identified associated with cardiac ion channels that cause long
QT syndrome and carry an increased risk for sudden death. The average age ot
persons who die o f long QT syndrome is 32 years; men are more commonly
affected. In addition to a prolonged QT interval, which occurs in some but not all
persons with long QT syndrome, another characteristic LCG abnormality is the so-
called Brugada sign (an upward deflection of the terminal portion of the QRS
complex). Most cardiac events arc precipitated by vigorous exercise or emotional
stress, but they also can occur during sleep. Torsades de p o i n t e s and ventricular
fibrillation ore the usual fatal arrhythmias. Long QT syndrome should be suspected
in patients with recurrent syncope during exertion and those with family histories
of sudden, unexpected death. Not all persons with long QT syndrome have warning
symptoms or identifiable EGG abnormalities, and they may present with sudden
death. P-Blockers, potassium supplements, and implantable defibrillators have been
used for treatment o f long QT syndrome. Identifying the specific gene mutation m
a given patient with long QT syndrome can help guide prophylactic therapy.

36. The answer is C. Treatments for stable patients with supraventricular


tachycardia (also referred to as regular narrow QRS tachycardias ) include vagal
maneuvers such as Valsalva’s maneuver, coughing; activation o f gag reflex; carotid
sinus massage; and placing an ice bag on the face or swallowing ice-cold water,
which can be extremely effective. Unilateral carotid sinus massage, one o f the m ore
common methods used, should be given at the angle o f the ja w on one side for 3 to
5 seconds. Patients with a history o f carotid artery disease are at increased risk for
the dislodgment o f plaque, which may lead to stroke. Adenosine (Adenocard) and
verapamil (Isoptin) administered [V arc also effective if the previously m entioned
measures fail to succeed. Adenosine is preferred because o f its rapid onset o f action
and short half-life. For unstable patients, low-energy electrical cardioversion is the
treatment of choice. Most patients troubled by this arrhythmia arc candidates for
radiofrequency ablation.

37. T he answer is B. The use of a permanent pacemaker is indicated if the patient


suffers from symptomatic bradyarrhythmias, asymptomatic Mobitz U AV block,
and complete heart block. First-degree AV block, thought to be a relatively benign
arrhythmia, can be associated with severe symptoms that may benefit from
permanent pacing. Type l second-degree AV block does not usually require
permanent pacing because progression to a higher degree A V block is not com m on
Permanent pacing is known to improve survival in patients with complete heart
block, especially if they have had syncope. Single and dual chamber pacemakers
are available and can be used depending on the patient’s diagnosis. Typically,
pacemakers are monitored on a regular basis by the patient’s cardiologist with
telephonic monitoring.
38 The answer is A~ Supraventricular tachycardia ^SVl^ is characterized by a
rap, 1 regular rhythm with a narrow QRS complex and abnormal P waves The
hean rate is usually 100 to 100 bpen. Some patients may be asymptomatic; others
may experience chest pain, palpitations, and shortness of breath. Hemodynainically
unstable patterns with SVT require immediate treatment with electrical
synchronized cardioversion. For those patients who are stable, \-aga) stimulation
can be attempted with carotid massage tbut not in patients with previous
cerebrovascular accidents or carotid bruits), Valsalva's maneuver, activation of the
gag reflex, or placing a cold ice bag on the face. If these measures are unsuccessful,
medication including adenosine, verapamil, dilhazem, or a 0-bkvkcr, can be used
Untreated SVT may lead to heart failure.

39. T he answer is A. Atrial flutter is a regular, rapid cardiac rhythm characterized


by an ectopic focus that gives rise to atrial rates from 2S0 to 350 impulses per
minute. Usually, impulses are only transmitted to the ventricles every second, third,
or fourth impulse. The heart rate is usually approximately 150 bpm. In many cases,
atrial flutter is difficult to distinguish from sinus tachycardia; however, carotid
ir.us massage or V alsalva's maneuver may help distinguish the characteristic
^.nvtooth) flutter waves seen with atrial flutter ECG shows flutter waves best in
th inferior leads 11, 111, aVF, and in V I. RR interval may be regular, reflecting o
f:\ed-ratio AV block (2:1, 3:1), or may be variable, reflecting a Wenckebach
,‘riodicity. Limited data suggest that the risk of thromboembolism, although
:allcr than with atrial fibrillation, is increased, suggesting anticoagulalion should
- considered. Treatment consists of verapamil, diltiazem, 0-blockers, or digoxin,
hich slows conduction through the AV node. Electric cardioversion (low energy)
indicated for patients who are unstable and show signs o f CHF (e.g., pulmonary
-es. hepatojugular reflux, distended neck veins).*•

. T he answer is B. PVCs arc abnormal ventricular beats that arc characterized


•vide QRS complexes, which are usually not preceded by P waves. In patients
h normal hearts, PVCs usually disappear with exercise. If the patient remains
iptomatic and there is no organic heart disease, no further treatment is
isary. If PVCs are frequent, electrolyte abnormalities and heart disease should
excluded. Patients with frequent, repetitive, or multiform PVCs and underlying
n disease arc at increased risk for sudden death because o f cardiac arrhythmia
nicularly ventricular fibrillation). Without underlying cardiac disease, bigeminy
b trigeminy are considered benign rhythms. Treatment o f PVCs is controversial
> - hould be reserved for symptomatic patients. If MVP, hypertrophic obstructive
.ardiomyopathy, prolonged QT interval, LVH, or CAD is present, a trial of P
blockers can be used. Types 1A (quinidine, procainamide) and IB (lidocaine,
rrcxiletine) antiarrhythmic agents may be used; however, they are associated with a
high incidence of side effects and can make the arrhythmias worse. Type 1C agents
fflecaimde, propafenone) should not be used because o f their potential for
increased mortality rates. Elimination o f exogenous catecholamines,
sympathomimetic amines, alcohol, and caffeine may decrease symptoms. In
general, anti arrhythmic drug therapy is rarely necessary.

41. The answer is D, The A T provides clinicians with information regarding ngbt
atrial pressures and filling. It mainly consists of five wave forms:
1 a wave - representing azriaJ systole;
2 c wave - representing closure o f the tricuspid valve (this wave is act usually
visible);
3 x descent - representing a tall in atnaJ pressure during ventricular systole,
4 v wave - representing axnal filling against a closed tricuspid valve;
5 y descent - representing the opening o f the tricuspid valve.
In atrial fibrillation, the ‘a waves' are absent (D) due to dysfunctional atnal systole.
A raised A T with normal waveform pattern (A) is usually seen in fluid overload
and right heart failure. Large v waves (B) axe usually seen in patients with tricuspid
regurgitation. Cannon ‘a waves’ (C) are seen m patients with complete heart block,
single chamber ventricular pacing, ventricular arrhythmias and ventricular ectopics.
Large ‘a waves’ (E) can be seen in pulmonary hypertension and pulmonary
stenosis.

42. The answer is (E). a) False. Stroke and heart failure are possible consequences,
b) False. Thyrotoxicosis is a well-recognized cause, c) False, b-blockcrs arc
generally effective, d) False. Digoxin can provide rate control but does not alter the
rhythm, e) True.

43. The answer is D. The PR interval is usually measured from the start o f the P-
wave to the start o f the QRS and the normal range lies within O.J2-0.2s (i.e. 120-
200 ms). In first degree heart block, the PR interval is prolonged, greater than 0.2 s
(200 ms) (D). Shortened PR interval (i.c <120 s or <0.12 s) (E) results from fast
AV conduction, usually down an accessory pathway seen in W olff- Parkinson-
White syndrome.

44. The answer is (C) a) False. Atrial fibrillation is characteristically irregularly


irregular, b) False, c) True, d) False. Mobitz heart block types I and II have
occasional missed beats, e) False.

45. The answ er is (D) a) False. Pre-excitation o f the ventricles causes runs o f
tachycardia, b) False. This is a variant o f ventricular tachycardia, c) False. Tills is
an atrial tachyarrhythmia, d) False. A ventricular tachyarrhythmia c) True. This is
also known as Mobitz type I second-degree heart block.

46. The answ er is (B). Serum potassium is important to monitor in patients on


digoxin as hypokalaemia can potentiate the effects o f digoxin leading to toxicity.
47. The answer is (C). Adenosine is a naturally occurring purine nucleoside with a
pharmacological half-life of < 2 seconds Its principal role is in the diagnosis and
management of paroxysmal supraventricular tachycardia. Cautions include atnal
fibrillation or flutter caused by accessory pathways and contraindications include
second- or third-degree heart block and sick sinus syndrome.
Before administering adenosine to a patient it is important to tell them that they
may experience a strange sensation that may lead them to feel like they are about to
die. This sensation may last for a few seconds.
Atenolol is a beta-blocker and has limited use in an acute setting whilst atorvastatin
has a role in reduction of cholesterol in a chronic capacity. Amlodipinc is a calcium
channel blocker and amiodarone has a role in refractory ventricular fibrillation or
atrial flutter.

48. The answer is (D). Torsades de pointes is a form o f polymorphic ventricular


tachycardia. Most episodes are associated with an acquired form o f QT
prolongation. Many o f these cases can be treated medically with intravenous
magnesium sulfate. Class LA and 1C antiarrhythmic agents arc contraindicated, they
may worsen the dysrhythmia by further prolonging ventricular repolarization.

49. T he answer is (E). This patient is presenting with atrial fibrillation with WPW.
All AV nodal blocking agents arc contraindicated in this case, including beta-
blockers, calcium channel blockers, digoxin, and adenosine. Use o f these agents
may accelerate conduction through the accessory pathway and increase the
ventricular response, leading to ventricular fibrillation. The medication of choice is
procainamide.

50. The answer is (E). Verapamil may cause fatal hypotension in VT (negative
inotopic and peripheral vasodilatation effects). Flccainidc should be used with
caution because it may produce an incessant form of VT, which is difficult to
control. Adenosine is used to differentiate SVT with bundle branch block from VT.

51. The answer is (B). Betablockcrs arc the mainstay o f treatment in long QT
syndrome. 'Hie most commonly used drugs arc propranolol and nadolol but
metoprolol and atenolol are also used. Implantable Cardioverter-Defibrillators arc
the most effective treatment in high risk cases The others drugs may produce a
prolongation of the QT intervaJcxacerbating risk of polymorphic VT and Torsades
dc poinles.

V alvular heart diseases

52. I be anjw er is I t . Aortic stenosis ( A S ) o c c u r , when there n obstruction in the


blood flow through the aortic valve. I ypically, there i z a pressure gradient >10 mm
Ifg acrr>v, the obstruction. The cause*, for aortic stenosis include previous
rheumatic fever with associated damage to the valves, excessive calcification of the
valves leading lo narrowing, or congenital causes (c g., bifid aortic valve) Men arc
mdre'commonly affected than women. Cardiac output is usually maintained until
the stenosis is severe. Once the condition becomes severe, symptoms may include
the classic triad (t.e., angina, dyspnea, and syncope), especially with physical
exertion. The classic symptoms of angina, exertional syncope, and dyspnea
generally follow an extended latent period during which the patient is
asymptomatic. The survival o f patients with AS is nearly normal until the onset of
symptoms when survival rates decrease sharply. After the onset o f symptoms,
average survival is typically 5 years or less Although the rate of progression of AS
is variable and difficult to predict, approximately 75% of patients with AS die
within 3 years after the onset of symptoms if the aortic valve is not replaced. Some
patients with severe AS remain asymptomatic, whereas others w ith moderate
stenosis have symptoms attributable to the condition. The normal aortic valve area
averages 2.5 cm2, and there should normally be no gradient. A valve area o f <0.8
cm2 or a gradient o f >50 mm Hg represents critical stenosis capable o f causing
symptoms or death. Seveje cases can result in sudden death. Physical findings
include a harsh systolic-ejection murmur found at the left sternal border that
radiates to the carotids, a palpable left ventricular heave, and delayed carotid pulse
upstroke. Some patients may present with findings o f CHF. Echocardiography is
recommended in patients with classic symptoms o f aortic stenosis accom panied by
a systolic murmur and in asymptomatic patients with a grade 3/6 or louder systolic
murmur. Severe cases should be referred for possible valve replacem ent. Those
patients with severe aortic stenosis should avoid strenuous activity.*•

53. The answ er is A. Some murmurs are specific to certain cardiac conditions:
• Austin Flint murmur is associated with severe chronic aortic, regurgitation and

may be middiastolic or prcsystolic. The murmur occurs when there is backflow of


blood from the aorta into the lefi ventricle and flow into the left ventricle from the
left atrium. The regurgitant stream often prevents the full opening o f the mitral
valve, thus obstructing flow into the ventricle.
• Still's murmur affects children and is described as a hum m ing or musical-
sounding systolic murmur that is loudest at the left sternal border. It is a benign
murmur The murmur is usually heard in children 3 to 7 years o f age and
disappears before the onset o f puberty.
• Physiologic Si murmur affects approximately 33% o f children younger than 16
yean who have a physiologic S3 heart sound that disappears before 30 years o f age
fhe sound is best beard with the patient in the left lateral decubitus position, with
the bell o f the nethoicopc over the point o f maximal impulse. I he sound is usually
a low-frequency thud that occurs in early diastole.

54. The answ er b E. MVP (v,ystolic-chck syndrome, Barlow's syndrome, and


floppy valve .yndrome) r> usually asymptomatic but rnay cause cbesl pa.fi,
palpitations, anxiety, d/spnea, or fatigue The condition is common and associated
with myxomainii. !nn formation o f'h e val/c leaflet MVP usoallv affect', health>.
young (15 to 30 years of age), thin women. MVP is determined by the detection of
a midsystolic click that is followed by a late systolic murmur and becomes louder
with Valsalva’s maneuver. A high pitched late systolic crescendo-dccrcscendo
murmur heard best at the apex may also be present. Presence of both murmur and
dick is not necessary tor the diagnosis. In patients with MVP, cardiac arrhythmias,
including PVCs. paroxysmal supraventricular tachycardia, and ventricular
tachycardia, may cause palpitations and may need treatment (usually with |3-
hlockers). Rarely, MVP may progress to mitral insufficiency because of rupture of
the chordae lendineae and may require valve replacement.

55, The answer is B. The prevalence of aortic regurgitulion increases with age
Unlike aortic valve stenosis, aortic valvular insufficiency is rarely caused by
degenerative aortic valve disease. Acute aortic valvular insufficiency may be due to
infective endocarditis, aortic dissection, trauma, or rupture o f the sinus of Valsalva
Chronic aortic insufficiency may be caused by aortic root disease secondary to
systemic hypertension, syphilitic aortitis, cystic medial necrosis, ankylosing
spondylitis, rheumatoid arthritis, Reiter’s disease, systemic lupus erythematosus,
IihlcTS-Danlos syndrome, and pseudoxanthoma closticum. Chronic aortic
insufficiency can be caused by valve leaflet disease, including rheumatic heart
disease, congenital heart disease, rheumatoid arthritis, ankylosing spondylitis, or
myxomatous degeneration. Symptoms of aortic valvular insufficiency are the same
in older persons as they arc in younger ones. Usually, the main symptoms arc
related to heart failure, with exertional dyspnea and weakness being common
symptoms. In some elderly patients, symptoms of dyspnea and palpitations may be
more common at rest than with exertion. Nocturnal angina pectoris, often
accompanied by flushing, diaphoresis, and palpitations, may occur, this is thought
to be related to the slowing o f the heart rate and the drop of arterial diastolic-
pressure. The classic findings of a high-pitched, blowing diastolic murmur and a
wide pulse pressure with an abruptly rising and collapsing pulse should make the
diagnosis of aortic valvular insufficiency easily recognized in elderly patients.

56. The answer is C. Acute mitral regurgitation due to chordae lendineae rupture
produces a holosystolic apical murmur, which radiates to the axilla but may be
difficult to hear due to rales. Papillary muscle rupture also causes acute mitral
regurgitation, but the apical systolic murmur begins with the first heart sound and
ends before the second. Aortic insufficiency produces a soft diastolic murmur heard
best at the right base. Acute tricuspid insufficiency causes a soft blowing
holosystolic murmur, which increases with inspiration and is heard best along the
left parasternal border. Acute ventral septal defect produces a loud”holosystolic left
parasternal murmur with a palpable thrill that tapers in intensity as intraventricular
pressures equilibrate.

57. The answer is C. Mitral stenosis occurs as a result o f a chronic autoimmune


attack upon the mitral valves. This leads to scar tissue formation, which ultimately
tethers the leaflets and prevents them from opening fully The autoimmune attack is
usually triggered by rheumatic fever (group A streptococcus) antibodies cross-
reacting with endocardial tissue. Mitral stenosis leads to reduced cardiac output and
hence peripheral cyanosis. The apex beat is described as “tapping” because of the
increased force in closing the mitral valve. The opening snap in diastole correlates
to foxed opening of the valve and may be followed by a mid-diastolic murmur.
Mitral stenosis leads to left atrial enlargement, and this is related to a bifid P-wavc
(P milraie) on the ECG. The P-wavc is bifid due to the presence o f a larger P-wavc
(left atrium) superimposed on a smaller one (right atrium). Hypertrophic
obstructive caidiomvQpathv leads to a double impulse npex beat.

58. T he answ er is B. A murmur heard loudest on inspiration (A) points to a rigbt-


sided valve lesion. The right intercostal space midclavicular line (C) is the
anatomical landmark for the aortic valve The mitral area is over the apex. A
murmur louder with the patient in the left lateral position (B) (as opposed to
leaning forward) is associated with mitral lesions. If heard, you should determine
whether the murmur radiates to the axilla. Corrigan’s sign (D) (visibly exaggerated
pulsating carotids) ls one o f the many signs of a hyperdynamic circulation
associated with aortic regurgitation (including dc Mussets, Traubes, Quinkca,
Durozicz and a whole hoM o f other:). A na/rov/ pulse pressure (Is) is a sign o f
aortic 3tcnosts.

T h e answ er b £ . S L L can cauac myocarditis, pericarditis and endocarditis.


5 9 .

Whilst the latter manifestation can cause valvular vegetations, they are rarely
symptomatic and are more likely to be diagnosed post mortem. C arnes o f aortic
regurgitation, which presents primarily as dyspnoea, include idiopathic aortic root
dilatation, syphilitic aortitis, aortic dissection and rheumatic fever. It is also
associated with ankylosing spondylosis and Marfan syndrome. In addition to a
collapsing pulse, aortic regurgitation also presents with an early diastolic murmur,
a mid-diastolic rumble (Austin Flint murmur), de M usset’s sign (synchronous head
nodding), Quincke’s sign (pulsing nail bed), I raube's sign (“pistol shot” sound at
the femoral artery as its walls slacken and then crack taught like a sail in the wind)
and a laterally displaced apex bcaL

Heart Failure

60. T he answ er h> B. There is no agreed-upon first-line test for the diagnosis of
heart failure and no simple method of measuring the adequacy o f cardiac output in
relation to normal levels o f activity. Heart failure usually is diagnosed in persons
with known heart disease who present with nonspecific symptoms (e g .,
breathlessness, ankle swelling) and signs (e g., basal lung crackles). To confirm
clinically suspected heart failure, physicians rely on surrogate measures o f cardiac
function such as left ventricular ejection fraction. However, it is clear that a large
proportion of patients with heart failure, particularly older patients and women.
haw presence! systolic function (i.e., diastolic heart failure). The best w.i> to
diagnose and treat these patients is unclear. BNP increases when cardiac myocytes
arc strained, therefore, BNP is an effective method for detecting heart failure* with
or without systolic dysfunction.

61. The answ er is D. Many medications used to treat heart failure (e g , diuretics
such as spironolactone [Aldactone], ACE inhibitors, angiotensin 11-rcccptor
blockers) reduce BNP concentrations. Therefore, many patients with chronic stable
heart failure will have BNP levels in the normal diagnostic range (i.e., BNP level <
100 pg/mL f 100 ng/L]). However, digoxin and some P-blockcrs appear to increase
natriuretic peptide concentrations. Exercise causes a short-term increase in RNP
levels, although only small changes are detectable one hour after exercise. No
circadian variation has been reported when BNP is measured every three hours for
24 hours, and there is less hourly variation with BNP than with ANP.

62- The answ er is B. Diastolic heart failure is a major contributor o f morbidity and
mortality. The condition is defined as symptoms o f heart failure in a patient with a
normal left ventricular function. It is characterized by a stiff left ventricle with
decreased compliance and impaired relaxation, which leads to increased end-
diastolic pressure. Signs and symptoms are similar to those of heart failure with
systolic dysfunction. The diagnosis of diastolic heart failure is made with
transthoracic echocardiography. Treatment of diastolic heart failure should include
normalizing blood pressure, promoting regression o f left ventricular hypertrophy
(LVH), avoiding tachycardia, treating symptoms o f congestion, and maintaining
normal atrial contraction when possible. Diuretic therapy is the mainstay of
treatment for preventing pulmonary congestion, while ^-blockers appear to be
useful in preventing tachycardia and thereby prolonging left ventricular diastolic
filling time. ACE inhibitors and ARBs may be beneficial in patients with diastolic
dysfunction, especially those with hypertension. In order to reduce heart rate,
blockers or nondihydropyridine CCB are recommended. In order to eliminate fluid
retention, diuretics are used with caution. ACE inhibitors are used to control blood
pressure, suppress LVH, and improve relaxation. High doses o f ARB are
recommended to reduce the risk o f hospitalization.

63. The answ er is C. The causes o f CHF are numerous and include CAD (most
common), dilated cardiomyopathy arising from toxins such as alcohol and
doxorubicin, idiopathic causes, infection, and collagen vascular disorders. Other
causes include hypertension, cardiac arrhythmias, cardiac valvular disorders,
hypertrophic cardiomyopathy, and restrictive cardiomyopathies (caused by
disorders such as amyloidosis, hemochromatosis, and sarcoidosis). CHF occurs
when there is a decrease in cardiac contractility, which leads to decreased cardiac
output that does not keep up with the body’s physiologic demands. Initially, this
inability to keep up with physiologic demands may only be seen with exercise.
However, as the disease progresses, signs may also occur at a resting state.
Symptoms include shortness of breath; paroxysmal nocturnal dyspnea, which
awakens the patient and causes severe shortness of breath and diaphoresis, causing
the patient to sit up for prolonged periods; orthopnea, and peripheral swelling.
Signs include jugular venous distention more titan 4 cm elevated from the sternal
angle with the patient’s head elevated al a 45-degree angle, hepatomegaly,
hepatojugular reflux, S3 heart sound, peripheral edema, and pulmonary rales).
Treatment consists o f a low-sodium diet, diuretics, ACE inhibitors (ramipril,
enalapnl. lisinoprif) or angiotensin ll-reccptor blockers (losartan, tandesarian) it
unable to tolerate ACE inhibitors, P-blockcTS (carvcdilol, long-acting inctoprolol
succinate), digoxin, and other afterload-rcducing medications (e g , hydralazine,
isosorbide dinitratc). CCBs, especially verapamil, should be avoided because o f
their negative inotropic effect on the heart; however, they are useful in cases ot
CHF caused by diastolic dysfunction and hypertrophic cardiomyopathy.*•

64. T he answer is D. Medications for the treatment of CHF include the following:
• Diuretics. Thiazide diuretics have been showm to be useful in decreasing fluid
overload in patients with mild CHF by inhibiting sodium chloride reabsorption at
the distal tubule; however, they are not usually effective in patients with advanced
symptomatology. In moderate and severe cases, the loop diuretics ( e .g ,
furosemide, bumetanide) arc indicated; these agents inhibit solute resorption in the
loop of Henle. Spironolactone (a potassium-sparing diuretic) can also be used m
the treatment o f CHF. Electrolytes should be monitored because of changes in
serum potassium, as well as in sodium, magnesium, and calcium.
• ACE inhibitors (captopril, enalapril, lisinopril, ramipril). These medications serve
as preload and afterload reducers by blocking (1) the production o f angiotensin fl, a
potent vasoconstrictor, and (2) the release o f aldosterone. A CE inhibitors are
effective in the treatment o f CHF and have been shown to increase survival in
affected patients. Electrolytes should be monitored because o f the possibility o f
hyperkalemia and renal insufficiency (especially in patients w ith renal artery
stenosis). ACE inhibitors have also been shown to be beneficial in promoting renal
blood flow in diabetes.
• Angiotensin II-receptor blockers (losartan, valsartan, candesartan) have sim ilar
effects to those o f ACE inhibitors, although conclusive trials have not been
reported regarding equal effectiveness
• Digoxin. This medication has been shown to be effective in severe CHF and in
CI1F complicated by atrial fibrillation. Its mechanism o f action involves the
energy-dependent sodium-potassium pump, leading to increased intracellular
calcium and a positive inotropic effect. F id d ly patients and those taking other
medication (e.g., quinidinc, amiodarone) are at increased nsk for toxicity and need
close follow-up with monitoring of digoxin levels Potassium levels should also be
monitored closely; hypokalemia can precipitate arrhythmias in patients taking
digoxin.
• ^-Blockers (carvcdilol, metoprololol) can reduce mortality in select patients—
especially in patients with idiopathic dilated cardiomyopathy With slower heart
celts, diastolic fjacr.on improves. Veuncuiar filling unprcw-i xvJ the ejection
frac&on may improve over 6 10 12 months, giving nsc to improved exercise
capacity’ Randomized control trials have shown significant reduction m ail-cause
mortality and cardiac events in patients taking carvedilol with miMiv >>mptomat.v
CHF and ar. ejection fraction < to 35*k
• Vasodilators (e.g., hydralazine, isosorisde uinitraic) can be used if patients arc
unable to tolerate ACE inhibitors. They work by decreasing preload ns a result o:
vasodilation.

65- The answ er is D- The use o f cardiac glycosides, such as digcxin, is not
disputed .and is generally recommended if there is a supraventricular arrhythmia
present, such as atrial fibrillation in the presence o f CHF. Digoxin is also used in
cases o f CHF in which the heart is dilated and the systolic function ls significantly
impaired. In patients with normal systolic function but with decreased ventricular
compliance (diastolic dysfunction) that gives rise to CHF. the use o f digoxin is not
recommended. Digoxin is recommended for symptomatic patients with stage C or
D heart failure. Digoxin should be used only as a second-line therapy for
controlling the heart rates o f patients with atrial fibrillation associated with heart
failure and is not recommended for the treatment o f diastolic heart failure.
Digoxin should also not be used in patients with idiopathic, hypertrophic, subaordc
stenosis; the medication is also usually withheld in patients with an acute Ml and
CHF, unless diuretics and vasodilators fed to improve cardiac failure. Unless there
is a specific contraindication, all patients with CHF and systolic dysfunction should
take both an ACE inhibitor and a ^-b locker, and if volume overloaded, a diuretic as
well. An ARB is recommended for patients who cannot tolerate an ACE inhibitor.
Addition o f an aldosterone antagonist can be beneficial for patients with
symptomatic heart failure or for patients with left ventricular dysfunction after an
Ml. A combination o f hydralazine and isosorbide dinitrate added to standard
therapy has been effective in African American patients with class HI to IV heart
failure. Digoxin can decrease symptoms and lower the rate of hospitalization for
heart failure, but does not decrease mortality. There is no evidence that any drug
improves clinical outcomes in patients with heart failure with preserved systolic
function.

66. The an sw er is E. Nonsteroidal anti-inflammatory agents inhibit the effects of


ACE inhibitors and diuretics and can worsen cardiac and renal function, fhe
emergency physician (EP) should review the medication lists o f heart failure
patients and also avoid prescribing nonsteroidal anti-inflammatory drugs (NSAlDs)
when such patients present for injuries or chronic pain. The main risk of
acetaminophen, at excessive levels, is bepalotoxicity. Stable heart failure patients
arc discharged on ACE inhibitors and beta-blockers because these medications
have been conclusively shown to decrease mortality. Angiotensin receptor blockers
have fewer side effects than ACE inhibitors but can still cause cough or
angioedema.
67. The anrwer b C. The specificities of orthopnea and d y ^ r ^ i for r ^ n failure
ajc. respectively, 88°/*, ard 50%. RaJcs have a predictive accuracy of 7tyv»
Peripheral edema and weight gain are more likely to occur in normo'ciLsive acute
decompensated bcarl failure, which develops over days or weeks Hypertensive
hean failure, which occurs over 48 hours or less, is the m od common acute
presentation. Hypotensive acute hean failure is the least common presentation of
acute decompensated hean failure

68. T he answer b B. Hean failure is a disease of chrome, insidious onset that can
first present with shortness of breath, persistent cough or insomnia. The heart fails
to maintain adequate cardiac output for a variety of reasons; the harder it tries, the
faster it fails. Occasionally, the left ventneie will weaken to the point where it
cannot empty efficiently. The subsequent raised end diastolic volume causes a pre-
stretchcd ventricle, leading to a more forceful contraction (Starling's law). This
alternating strong-weak pattern can be seen in the ECG as pulsus altem ans
(varying amplitudes o f the R-wave). A column o f blood builds up proximal to the
left ventricle, in the lungs, causing pulmonary oedema. This creates afterload upon
the right ventricle, which can lead to tricuspid regurgitation. The bottleneck o f
blood can extend further, through the right side o f the heart and hence A T is raised
(the heart is unable to pull all the blood from the jugular vein through the lungs and
the left ventricle). Furthermore, close to the right side o f the heart, the inferior vena
cava remains full o f blood, leading to oedema in the liver (hepatomegaly). A nkle
veins swell with blood as the heart fails to suck the blood vertically against gravity.
The increased hydrostatic pressure results in fluid extravasation into tissues. The
push o f a finger is enough to displace this tissue fluid, hence this ankle oedem a is
pitting. Causes o f non-pitting oedema include lyraphoedcma and pretibial
myx oedema.

69. T he answer Is (B) a) False Beri-bcri (vitamin B1 deficiency) is, however, a


cause, b) True, c) False. This leads to outflow obstruction, d) False This impairs
ventricular filling, e) False.

70. T he answer Is (B). A patient at rest with inild right-sided heart failure will
most likely have a normal arterial blood pressure and cardiac output I (is cardiac
reserve, however, is diminished. Central venous pressure (and right atrial pressure )
will be elevated since the right ventricle cannot generate a normal cardiac output at
normal filling pressures. Since cardiac output at rest is normal, thcie is no reason
for left atrial filling pressure to be elevated

71. T he answer b (A). Digitalis lues been used lor yean to treat ctuoiuc C'llf
Digitalis enhances contractility, icduccs the heart’s sympallkrtjc roixjii.se* and
controls the ventricular response (impioving diastolic filling lime) t<* atrud
im llaiion. Paroxysmal atrial tachycardia ar.d accelerated junctional rhythms ire
common arrhythmias seen m digitalis toxicity.

“2. The answ er is (EV Spironolactone is a well recognised cause of


^TUtvomasrii

73. The answ er is (Eh The pharmacokinetics o f digoxin are complex and best
explained by a txvo compartment model The loading dose is mainly dependent on
the \ olumc ol Distribution ot a drug but this patient has moderate renal failure
The loading dose is calculated (using various models) by Liking into account age,
creatinine clearance, body surface area etc. Volume of distribution becomes
important particularly when body weight is -10kg or less. On balance it is the renal
failure that is the most important factor in this patient in determining the loading
dose Digoxin is cleared by the kidneys so the maintenance dose would require
adjustment in renal failure.

Rheumatic Fever & Infective Endocarditis

74. The answ er is E. Antibiotic prophylaxis (amoxicillin 2 g orally 30 to 60


minutes before the procedure) is recommended for patients who have high risk
cardiac conditions, which include
• prosthetic cardiac valve
• history o f infective endocarditis
• unrepaired cyanotic congenital heart disease; or a completely repaired congenital
heart defect with prosthetic material, during the first 6 months after the procedure;
or a repaired congenital heart defect with residual defects at the site or adjacent to
the site o f a prosthetic patch or prosthetic device (which inhibits cndothelialization)
• cardiac transplantation recipients with cardiac valvular disease.
The guidelines suggest preventive treatment for high-risk cardiac patients, not for
all dental procedures but only for those that involve manipulation o f gingival tissue
(around bone and teeth) or the periapical region o f teeth (tip of the tooth root). The
guidelines do not recommend antibiotics for routine anesthetic injections through
non-infected tissue, placement or adjustment o f orthodontic appliances, shedding of
baby teeth, or bleeding from trauma to the lips or inside of the mouth.
Further the American Heart Association (AHA) guideline no longer consider any
G1 (colonoscopy or esophagogastroduodenoscopy) or genitourinary procedures
high risk and therefore do not recommend routine use of endocarditis prophylaxis
even in jyatient with the highest risk cardiac conditions. Vaginal or cesarean
delivery is not an indication for routine antibiotic prophylaxis.

75. The answ er is E. The Modified Duke criteria have two major criteria and
seven minor criteria. To have a definite diagnosis of endocarditis, a patient needs
two major criteria, one major and three minor criteria, or five minor enteria
positive. A patient with one major and one minor, or three minor, may be said to
have possible endocarditis.
The major criteria arc:
• Blood culture findings (either typical organism from two cultures, positive blcod
cultures >12 hours apart, or three or more positive cultures taken over more than I
hour)
* Endocardial involvement (either vegetations on echocardiogram or a new
valvular regurgitation)
The minor criteria arc
X Predisposing cardiac/valvular abnormality (e.g. metal prosthetic valve)
X Intravenous drug use
X Pyrexia greater than or equal to 38°C
X Vasculitic phenomenon (c.g petechiae, splinter haemorrhages, Osier nodes)
X Embolic phenomenon (eg. cerebral abscesses)
X Blood cultures positive but not matching major criteria
X Echocardiographic findings suggestive but not matching major criteria
Comparing our patients, A has three minor criteria, B has one major criterion (the
murmur sounds like mitral regurgitation) and one minor criterion (Strep, viridans
accounts for 30-40% o f endocarditis, but there is only one culture positive here), C
has three minor criteria, D has one major criteria (blood cultures) and the murmur
sounds like aortic stenosis that meets no criteria, and E has one major and three
minor criteria. Diagnosis using the Modified Duke criteria is important to guide
prognosis and management, which often involves a long regimen o f multiple
antibiotics (usually over 4-6 weeks).

76. The answ er is D The diagnosis here is subacute bacterial endocarditis,


probably due to Streptococcus viridans. The definitive diagnosis is by blood culture
(D) although echocardiography (B) will show vegetations on affected heart valves.
Although the lesions described are vasculitic (as are the painless Janeway lesions
and the Roth spots in the retina), in this case they are due to antigen-antibody
complexes triggered by infection. The issue o f routine prophylaxis for patients with
valvular disease prior to dental procedures is controversial; in the UK, it is no
longer recommended.

77. The answer is C. Transoesophagcal echocardiography (C) is the investigation


ot choice since it provides high resolution images, as well as the option o f
alternative views. Simple auscultation (A) is appropriate to detect the structural
breakdown o f the valve. Transthoracic echocardiography (B) is the initial
investigation and is ideal as it is non-invasive, however, definitive investigation is
made difficult by the scattering o f US signals the mechanical valve. A chest x-ray
(D) and ECG (E) would be useful for detecting gross anomalies to heart function
such as failure, but are not sensitive enough to detect vegetative damage.
78. T he answ er is (B) a) False. Surgery may be required in more severe eases, h)
True. Dirty needles are a well-recogruzcd cause, c) False Intravenous antibiotic0,
are always indicated in the first instance, d) False. Diabetes tnclliius is not a nsk
factor, c) False. This is a severe disease which requires aggressive treatment.

79. T he answ er is (E). Erythema nodosum has no association with infective


endocarditis. Retinal haemorrhages are seen in the form of Roth spots Vasculitis
accounts for Osier’s nodes, splinter haemorrhages. Roth spots and Jancw.iv lesions

80. The answ er is (B). If the causative organism is not known empirical Uicrapy is
as follows:
• Intravenous benzylpenicillin and gentamycin unless staphylococcal inlcction is
suspected when vancomycin is substituted for penicillin.
* Vancomycin and gentamycin if patient is allergic to penicillin.

81. T he answ er is (C). Polyarthritis together with erythema marginatum,


Sydenham's chorea, carditis andsubcutaneous nodules constitute the major criteria
associayed with Rheumatic fever. Minor criteria include raised ESR. Arthralgia,
pyrexia and a prolonged PRinterva!. Migratory-erythema is associated with a
glucagonoma

82. The answ er is (E). Findings that would be typical include increased cardiac
output, increased pulse pressure, a decresendo murmur and a low diastolic blood
pressure. Vasoconstriction not dilatsrioo is typically fouxal

Cardiom yopathy

83. The an sw er b B. Hypertrophic obstructive cardiomyopadiy is an autosomal-


dominant transmitted disorder th»r is f tararf<TT7»^ by an enlarged cardiac septum,
which obstructs blood flow pom the left ventricle. Symptoms include dizziness,
light-headedness, palpitations, chest pen, dyspnea, or syncope with physical
exertion. The most important cotx^ilicaDGc o f the is sudden death The
condition has an annual incidence of 4% to 6% in children, and 2% o f adults are
affected. Signs inrli*y pulses rhar are bifid and brisk in upstroke and a sy stolic -
ejection type migrmw positioned along the leri sternal border, winch becomes
louder with movement* rhai decrease verves return (afterioad), such as standing or
Valsalva’s maneuver. Movements that increase venous return (afterioadj, such as
squaring. reduce the murmur. ECG usually shows evidence of L VH and septal Q
waves in the ja c n l Diagnosis is by echocarcbogratP- Fa&Aiat
treaaacan if accomplished with the use of ^-bicckers- Duopyraoude (Norpace> may
be jsed as an ahematr'i e Verapamil has alsn oeea used frequently oo an cajxric
basis especially m acral LLbnllalyxi but in effect is unpredictable, arid acme
'xzrs.&fTJirz.c ccflapse described ir. psjznzi with s;Jsetasta.’ <a jc. t . c

7C
t a
<1i.i s IoIk* dysfunction. D ioxin should U; avoided Bccair^r of rbc: rc.k for
dcatfi, extreme physical exertion should be avoided.

84. The a n m e r is A. A pericardial knocic, an early diastolic sound, that can he


heard at the apex 60-120 milliseconds after the second heart sound but before an
S3 is associated with constrictive pericarditis but is clirucaiiy very challenging to
differentiate fiom an S3. Both conditions share the signs of JVD and JCussmaui,
which is an increase in venous pressure with inspiration. The dip and plateau filling
pattern, also known as the square root sign, n seen m both conditions. A ma/Ved
decrease Answers: 39 through 5J 35 followed by a rapid rise and plateau in
carlydiastolic ventricular pressure is seen in hemodynamic studies, such as Doppler
echocardiography or cardiac catheterization. CT and MRJs of the heart can
diffcreoliaie constrictive pericarditis from restrictive cardiomyopathy. Pericardial
thickening is seen with constrictive pericarditis, but not with restrictive
cardiomyopathy. Constrictive pericarditis can be treated surgically. Restrictive
cardiomyopathy is symptomatically managed with diuretics and ACE inhibitors
and medical therapies specific to the underlying etiology, for example,
corticosteroids for sarcoidosis and chelation for hemachromatosis.

85. The answer is D. Most eases o f restrictive cardiomyopathy are idiopathic,


often with an autosomal-dominant familial pattern. The remainder o f cau ses are
systemic disorders such as those listed earlier, as well as progressive system ic
sclerosis (scleroderma), and hypereosinophilic syndrome. All etiologies affect the
heart muscle, either by scarring o r infiltration, and cause stiffness th at affect
diastolic filling and raises enddiastolic pressures. Systolic LV function is normal.
The EP should consider restrictive cardiomyopathy in a patient presenting with
signs of CHF but without cardiomegaly or systolic dysfunction.

86, The answer is (C) a) False, b) False, c) True. This is a cause o f sudden d^arh
particularly in young people, d) False. This is a normal finding in young people.
e) False.

87. The answer is (B). Hypertrophic cardiomyopathy is defined


unexplained, asymmetrical or concentric hypertrophy o f the urriibtrri left \cn in c;e.
There is also hypertrophy of the right ventricle, h may be inherited as an
autosomal-dominant condition, but al least half o f cases may be the result o f
sporadic mutation and therefore the patient may be jarw are o f the coodiUon.
Symptoms mav include'
• Angina
• Dyspnoea
• Palprjdjoe:
• Syncope
• Sudden deal:.
Clinical vigns include
• Jerky pulse
• JVP: large a waves, indicating right ventricular flow obstruction
• Double impulse at apex
• Loud fourth heart sound due to the left ventricular hypertrophy
• Third heart sound
• Late systolic murmur
It is unlikely that this episode has been caused by a pulmonary embolus as she is a
fit and well lady with no obvious risk factors. A pneumonia or pneumothorax
would have caused some symptoms and arc unlikely to have resulted in a cardiac
arrest.

Pericarditis

88. The answer is (A). The pulmonary' trunk lies posterior to the aorta, rhe
ascending aorta lies completely within the pericardium, as does the pulmonary’
trunk. The left atrium is the most posterior chamber of the heart, the right atrium is
just anterior and to the right of the left atrium. The left atrial appendage is not
readily seen on transthoracic echocardiography and requires transoesophageal
echocardiography.

89. The answer is A The answer is (A) (KussmauTs sign). Third bean sounds (B)
and focsih heart sounds (Q are associated with bean failure, but not pericardial
riscasrs Lung signs are less ilkeaY than ‘right sided’ ones, such as ascites ana
peripheral oedema. The first and second heart soends are usually reduced as the
pericardia; wsl: is thickened and sc cn i cansmissice: is reduced.

9Q. The answer is (a). True. A rapid increase in bean size sod a gJobcbr shadow-
are features, ri False. .An ECG is required, c) False. Clinical history and ECG are
esser.ria.1 cri False. ECG is Sagcosric. e'i False. Bacterial endocarditis is not seen on
chest X-ray .

91. T he answer b (A), liecfificarica o f cardiac tamponade relies upon ‘Beck’s


triad1: hypotension, jugular vein distention and muffled heart sounds. This results
from fluid accumulation in tbe pericardial sac that dampens the transmission of
sounds through tbe chest wall. Identification of tbe quiet heart sounds can be
difficult just using a stethoscope, which is why a hospital setting is useful for
further investigations. Bradycardia is not associated with cardiac tamponade, in
fact, tachycardia is often seen as there is reduced filling time in the cardiac cycle
due to reduced capacity o f tbe chambers.

A ortic Aneury sm

92. T he answer is A. Cigarette smokers are five times more likely than
nonsmokers to develop an AAA. The risk is associated with the number of 42 years
the patient has smoked and declines with cessation. Diabetes mellitus is protective,
decreasing the risk o f AAA by half. Women tend to develop AAA in their sixties,
10 years later than men. Whites are at greater risk than African Americans
Hypertension is less o f a risk factor than cigarette smoking.

93. The answer is C. Aortic abdominal aneurysm is defined as dilated aorta with a
diameter at least ' 1.5 times the diameter measured at the level of renal arteries.
AAAs result from a weakening in the wall of the aorta. Most cases occur inferior to
the renal arteries and are asymptomatic; however, back pain or abdominal pain may
precede rupture. Most aneurysms are the result o f atherosclerotic disease that
results in weakening o f the vessel. Strong evidence suggests a genetic susceptibility
to AAAs. Patients with these aneurysms have a 20% chance of having a first-
degree relative with the same condition. Male siblings are at particular risk.
Approximately 75% of AAAs are asymptomatic and are detected during routine
physical examination or during an unrelated radiologic or surgical procedure.
Symptoms of an AAA may result from expansion or rapture of the aneurysm,
pressure on adjacent structures, embolization, or thrombosis. The most commonly
reported symptom is any type o f abdominal, flank, or bock pain. Pressure on
adjacent viscera may result in compression of the bowel. Patients may present with
early satiety and, occasionally, nausea and vomiting. Rarely, ureteral com pression
may result in a partial ureteral obstruction. Thrombus and atheromatous material,
which line nearly all AAAs, may occasionally result in distal arterial em bolization
and, rarely, aneurysm thrombosis. The abrupt onset o f severe, constant pain in the
abdomen, flank, or back, unrelieved by positional changes, is charactensoc of
expansion or rapture o f the aneurysm.
Physical examination often reveals a pulsating aNJommaJ. mass. Obesity.
uncooperarivexKSS, ascites, tortuosity o f the aorta, and excessive lumbar lordosis
are cooftions that may make diagnosis by palpation difficult- Examination of the
abdominal aorta is facilitated by having the parient lie on the examinarioa cable
with the knees slightly flexed. The aorta is palpated during exhalation- A pulsatile
abdommaJ mass left of midhr*»— between the xiphoid process and the umbilicus—
is highly suggestiv e o f an AAV
Diagnosis is matte with ultrasound or CT examination. B-nxxie ultrasound is the
screening method o f choice for asympcomaac AAAs. U is available in most
hospitals, is inexpensiv e, docs not require ionizing radiation, reveals details o f the
vessel wall and associated atherosclerotic plaques, and allows accurate
measurement o f the aneurysm in longitudinal and transverse dimensions.
Typically, aneurysms >5.5 to 6 cm are treated surgically, whereas smaller
aneurysms are observed for any changes. If they grow 1 cm/year, larger surgery is
recommended. Endovascular repair is safer, results in shorter hospital stays and
quicker recovery, and translates into significant cost savings when compared with
conventional surgery. The operative mortality rale is usually <5%. The mortality
rate of patterns vvith aneurysms >6 cm is approximately 50% in 1 year, patients
with aneurysms between 4 and 6 cm have a mortality rate o f 25% in 1 year
94. The answ er is D The most likely diagnosis, and the one that must be most
urgent!} excluded, is an aortic dissection (D). The location does not indicate a
pneumothorax (A), the symptoms and ECG arc against an N1I (B) and the pain
seems too severe for a pulmonary embolus (C), or pain o f musculoskeletal origin
fE). Cbesi x-ray may show widening o f the aorta, and CT and MRJ scans may be
diagnostic If confirmed, BP reduction and dampening o f the aortic systolic wave
by beta-blockade is indicated and urgent surgical intervention should be
considered

95. The answ er is (D) a) False, b) False. This is an idiopathic cause o f digital
ischaemia, c) False. Budd-Chiari syndrome occurs following occlusion of (he
hepatic vein, d) True. Tertiary syphilis may lead to aneurysms of the ascending
aorta and aortic arch, c) False. The commonest cause o f subarachnoid haemorrhage
is a berry aneurysm.

96. The answ er i s (D). Whilst measuring the blood pressure in all limbs raises the
suspicion o f aortic dissection, the most diagnostic investigation is a CT chest.
Other diagnostic investigations include MRI and a trans-ocsopliagcal
echocardiogram. There arc no diagnostic ECG changes associated with dissecting
aortic aneurysms and liver function tests are of no use in confirming the diagnosis.

Pulmonary Em bolism and Anticoagulation

97. The answ er is D. A PE is a thrombus that lodges in the pulmonary vasculature


and may give rise to a pulmonary infarction. In most cases, the thrombus forms in
the leg or pelvic veins. The most dangerous thrombi form in the iliofemoral vein.
Other causes o f emboli include tax emboli after fractures and amniodc fluid emboli,
which arc rare. Risk factors for PE include malignancy, hereditary impaired
coagulanozi estrogen therapy, obesity, CHE, orthopaedic or pelvic surgery, and
rroioaged anesthesia. Signs and symptoms include tachypnea, cough, hemoptysis,
chess pa— fever, and cyanosis severe cases! Diagnosis is based c c the clinical
hdeesy *od surpcvrdvc tescs, induem g a \enhliixva-perfus:on scan and pulmonary
-irsersogrzrr. .Arterial Need tescs show hypoxia fP 02 <60 cun HgX and ECG
arc nocspxcific fT-wave dbtxxmalincs in the peecordial leads and sinus
tachycardia •. In addihoo. right-axis deviance. an S1-Q5-T3 partem, and a right
bundle branch N ock may be observed. Further testing may include venous studies
o f the lower extremities to look for thrombus; however, >20? o of patients may have
no evidence el venous embolism. Chest radiographs are usually normal; however, a
homogenous, wedge-shaped density' based in the pleura and pointing to the hilurn
(Hampton's hump) is highly suggestive o f PE. Treatment involves anticoagulation
for 3 to 6 months with oral warfarin or thrombolytic therapy and embolectomy if
the patient has hypotension and continuing hypoxemia while receiving high
fractions of inspired oxygen. Thrombolytic therapy is not indicated for the routine
treatment o f patients with PE. A stepwise approach to the diagnosis o f pulmonary
embolus consists o f a ventilation perfusion scan. A higb-probabtlity vcntilaiion-
perfusion scan provides sufficient evidence for the initiation of treatment for PE_
Likewise, a normal scan should be considered sufficient to exclude PE.
Unfortunately, 50% to 70% o f scans 2re indeterminate (low or intermediate
probability) If the results show a high probability o f PE, treatment with
anticoagulants is indicated. If the ventilation-perfusion scan is normal, treatm ent is
not indicated. If the lung scan shows intermediale or low probability o f pulmonary
embolus, a nomnvasive leg test (ultrasound) for proximal DVT should be obtained
If the leg test is positive, then treatment is indicated If the leg test is negative and
suspicion is high, then a pulmonary angiogram or another ultrasound and d-dinier
test can be repeated in 5 to 7 days. If the test is negative, the risk for pulmonary
embolus is low. A spiral CT of the chest can also be used for rapid diagnosis.

98. The an sw er is C. Warfarin is classified as an anticoagulant Its m echanism of


action is the inhibition of vitamin K-dcpendcnl clotting factors (i.e , factors II, VII,
IX, and X). T he medication is used in stroke prophylaxis for patients w ith prior
neurologic events, atrial fibrillation, mechanical heart valves, or previous DVT or
PE. Warfarin interacts with many medications, so concomitant use with other drugs
should be monitored carefully. Most complications arc related to bleeding;
however, other side effects, including nausea, vomiting, fever, burning o f the feet,
and rashes, may occur. The most common complication unrelated to excessive
bleeding is skin necrosis, which usually occurs within the first week o f therapy.
Some cases may be severe enough to require surgical debridement or even
amputation. Patients treated with wart an n should have their PTs and INRs
followed to ensure proper levels o f anil coagulation.

99. The an sw er is E. PE is usually a consequence o f DVT and is associated with


greater mortality. MDCT angiography is the diagnostic test o f choice w hen the
technology is available and appropriate for the p atien t MDCT is a form o f CT
scan, where the two dimensional array o f detector elem ents replaces the linear
array ot detector elements used in typical conventional and be heal C T scanners.
The development of MDCT has resulted in the development of high-resolution C l'
applications such as CT angiography and CT cvkutoocopy It is w arranted in
patients who may have a PE and a jwsitive d-dimcr assay result, or m patients who
have a high pretest protabtbty o f PE. regardless of d-dxroer assay result
For patients with contraindications to CT. including contrast allergy, renal disease,
and pregnane>. vennLicxva-jxTtusion scanning is the preferred imaging mcsialirv
tor evaluation ot j.»ssible PE. Pulmonary angiography is needed onlv when the
clinical suspicion tor PE remains high, even when less invasive stud) results are
negative. In unstable emergent cases highly *iep:c:ous for PL, echo-cardiography
may be used to evaluate tor right ventricular dysfunction, which ts indicative o f but
not diagnostic for PE in patients with a low pretest probability of L>VT o* PE. a
negative result from a higb-scnsitivity d uimct assay is sufficient to cxchnlc venous
thromboembolism

SO
100. The answer is D. Heparin is an anticoagulant used to prevent thrombosis.
Heparin works by binding to and activating antithrombin 111, an extremely potent
anticoagulant that prevents thrombin generation and fibrin formation The drug is
administered intravenously ar.d subcutaneously. The major side effect is bleeding.
If needed, protamine sulfate may be administered to rapidly reverse heparin's
anticoagulant effect, in most cases, this measure is unnecessary and the
discontinuation of heparin is adequate. Other complications include hepann-
induced thrombocytopenia, which occurs in 10% of patients taking the medication.
The thrombocytopenia can actually lead to a paradoxical arterial thrombosis, which
can be life threatening. Discontinuation of the medication usually reverses the
thrombocytopenia. When administering intravenous heparin, the P IT should be
monitored. Any increase in heparin dose is usually detected 4 hours later (as noted
with a prolonged PTT) and vice versa with decreased doses o f heparin. The goal
tor anticoagulation is usually 1.5 to 2.0 times the normal value, but may depend on
the individual case. Patients should not take aspirin while taking heparin;
intramuscular injections should also be avoided. Chronic use o f heparin may
increase the risk of osteoporosis. LMW heparin is now* available and is used for
anticoagulation. PTT and thrombin times are minimally affected by typical
therapeutic doses. Therefore, laboratory monitoring is not required.

101. The answer is B. Enoxaparin was the first LMW heparin approved for the
treatment o f DVT in a dosage o f 1 mg kg twice daily or 13 rag once daily. LMW
heparin offers distinct advantages over uafracrionatcd heparin:
It can be administered subcutaneously once or twice daily, it has a longer biologic
half-life, dosing is fixed, and laboratory monitoring is not required. In addition,
thrombocytopenia appears to be less likely. In patients with DVT, subcutaneous
administrarion of hqtsrin is as effective as continuous infusion o f unfracoocacd
heparin in preventing complicarioas and reducing the risk of recurrence. Outpatient
management o f DVT using LMW hepsrin far short-term anticoagulation until
warfarin is at a therapeutic level is considered safe and cost-effective. Candidates
far outpatient therapy must be hcmodynamically stable, without renal failure, and
not 21 high risk tor bleeding. Additionally, they must have an appropriate
supportive home environment and be capable o f daily monitoring until the INR is
therapeutic- LMW heparin is typically given in combination with warfarin for 4 to
5 cays. Simultaneous initiation o f warfarin and imfracriocaied heparin or LMW
heparin has not been associated with adverse outcomes. Dalteparin (FragminX
another LMW heparin, is approved only for prophylaxis of DVT. The FDA has
also approved the use o f tinzaparin (hmohep) for the treatment o f DVT.

Miscellaneous

102. The answer is A- Superior vena cava syndrome results from compression of
the superior vena cava by a neoplastic process (90% of cases) and less commonly
by inflammatory states. Other causes include benign tumors, aortic aneurysm,
thyroid enlargement, thrombosis of a central venous l,ne, and fibrosing
mcdiustinilis. Lung cancer, particularly small cell and squamous cell type, is the
most common associated malignancy. The condition causes the obstruction of
venous drainage to the heart and leads to dilation o f collateral veins of the upper
chest and neck. Signs include plethora and swelling of the face, neck, and upper
torso. Edema of the conjunctiva, shortness o f breath in a supine position, and CNS
disturbances, including headache, dizziness, stupor, and syncope, may be seen.
Acute development o f symptoms indicates a poor prognosis. The diagnosis o f
superior vena cava syndrome is essentially a clinical one. Chest x-rays may show
widening o f the mediastinum, particularly on the right, but the best confirmatory
test is CT. The MRi has no advantages over CT. The one potentially life-
threatening complication o f a superior mediastinal mass is tracheal obstruction.
Treatment includes steroids, chemotherapy, and radiation to the tumor. Although
the most common cause o f this syndrome is metastasized carcinoma o f the lung,
other less common infectious causes include tuberculosis, histoplasmosis, and
constrictive pericarditis.

103. The answer is D. Cardiogenic shock is mostly commonly due to extensive Ml


with suppressed myocardial contractility. Pump failure is the underlying factor in
most causes of cardiogenic shock. Hypoperfusion, with or without hypotension, is
the unifying feature o f cardiogenic shock, regardless o f etiology. During an AMI,
several mechanical complications can precipitate cardiogenic shock, including
acute myocardial regurgitation due to papillary muscle rupture, ventricular septal
defect (VSDX and free-wall rupture. Mechanical complications cause one-fourth o f
the cardiogenic shock following AML Right ventricular infarction can also cause
cardiogenic shock due to loss o f preload. Cardiac contractility can also be severely
depressed due to sepsis, myocarditis, myocardial contusion, and cardkanyopatby.
Mechanical obstruction to forward blood flow can also lead to cardiogenic shock,
including aortic stenosis. HCM, mitral stenosis, left atrial m yxom a, and pericardial
tamponade. Regurgitation o f LV output due to chordal rupture or aortic
insufficiency can cause profound cardiogenic shock.

104. The answer b B, Coarctation of the aorta is accurately diagnosed by magnetic


resonance angiography. .An ECG does not normally show' any diagnostic changes
Collateral vessels may be visible around the scapular Upper limb pulses are
stronger than lower limb pulses and this can be an indicator o f coarctation is the
absence o f peripheral vascular disease (unlikely in a 14-year-old) but only if the
brachial artery assessed is proximal to the coarctation The coarctation may reside
between the right brachiocephalic artery’ and the left subclavian artery leading to
radial pulse asymmetry, or distal to them both leading to radio-femoral delay. A
reduced ankle brachial pressure index may also be present.

105. The answer b B. Dopamine has varying dose-related effects due to a l-. J3»-
and dopaminergic actisity Low dose effects (3 : doses < 2 pg. kg'min) are
predominantly renal There is increased renal, cerebral, coronary ;uk1 mesenteric
blood flow with vasodilation because of agonistic action on dopamine receptors m
these vascular beds. Intermediate dose effects (at doses 2-10 pgk g m m ) are
predominantly cardiac, pi-agonist activity results in increased cardiac contractility
and heart rate The increased cardiac output and increased dopamincigic activity
results in increased mesenteric perfusion. Slight al-adrenergic activity may lead tv>
a degree of peripheral vasoconstriction.
High dose effects (at doses 10-20 pg/kg/min) arc vasoconstrictive and carvliac a I ■
Adrenergic activity causes marked peripheral vasocc*nstrictkvn and a rise in blood
pressure. There can also be renal and mesenteric vasoconstriction At verv high
doses ( > 20 pg/kg/min) the overriding adrenergic activity may cause extreme
vasoconstriction, which can suppress dopaminergic neul vasodilation and
deleterlously affect the renal and peripheral circulation

106, The answer is (B) a) False, b) True, This is a primary cardiac tumour, usually
in the left atrium. Other cardiac causes include infective endocarditis and
congenital cyanotic heart disease, c) False, d) False, e) False.

107. The answer is (B). A PDA is a defect between the pulmonary’ artery and the
aorta. The ductus arteriosus normally doses within the first 4S hours of life. In
premature babies it may remain open for longer, sometimes up to 3 months. If it
remains patent longer than this it is unlikely to close spontaneously. A persistently
patent ductus is a common congenital b est lesion, occurring cither singly or in
combination with other defects. Girls are more likdy to be affected by PDA, and
those affected by congenital rubella syndrome axe more likdy to suffer a PDA. All
other options are correct

108. The answ er is (E>. Adrenergic agents are divided into pure alpha-agents
(phenylephrine), mixed alpha- and beta-agents (epinephrine, norepinephrine), and
pure beta- or primarily beta-agonists (isoproterenol, dobutamine, dopamine). The
alpha-receptors are found primarily in blood vessels, where alpha stimulation
causes vasoconstriction. Beta-agonists work primarily on the heart and promote
increased heart rate, increased contractility, and increased myocardial oxygen
consumption. Bcta2 receptors are in smooch muscle o f the bronchi, blood vessels,
and uterus.

2 PULMONARY MEDICINE

Normal Physiology and Function

109. Answer is: B 450mL. Tidal volume is approximately 6 mL/kg. Therefore in a


70 kg male this is likely to be around 420 mL, making B the closest and most
suitable answer.
110. Answer l.i: C 6L. I olaJ lung capacity is approximately 80 mL/kg. Therefore
in a 70 kg male this should be 5600 ml., making C the closest answer.

111. Answer Is: C* 4.5 I.. Vital capacity is approximately 60-70 mL/kg. Therefore
in a 70 kg male this is between 4300 and 4900 mL, making C is the closest answer

112. Answer b : C' ISOmL Anatomical dead space is the volume of gas in the
conducting airways which is not involved in gas exchange. It is approximately 150
ml..

m . ANSWER: B. FEV) 10 L (33% o f predicted), FVC 3 0 L (75% o f


predicted), FEV I/FVC 0 33 (44% o f predicted). Let us offer a simple strategy for
interpreting spirometry results. After you have checked to see if the test has been
properly performed, with o smooth graphic display (voluroe-timcplot) and at least 6
seconds o f forced expiratory time, look at the FEVI/FVC ratio. If this ratio is
reduced (as in examples A and B), there is an obstructive pattern. A reduced ratio is
typically defined as a value below the 95% confidence interval (not given in these
examples). The Global Initiative for Chronic Obstructive Lung Disease (GOLD)
guidelines attempt to simplify this definition by using a cut-off for a reduced ratio
as <0.7. This cut-off is generally accurate except for all but the oldest o f our
patients, in whom an FEVI/FVC ratio <0.7 may still be normaL (The FEVI/FVC
ratio declines with age due to loss o f lung elastic recoil during the normal aging
process.) If the FEV 1/FVC ratio is normal or increased, there is no obstruction.
The pattern in Answer C suggests severe restriction; Answer D indicates normal
spirometry; and in Answer E the results suggest a pattern o f moderate restriction.
Restriction is best confirmed with full measurement o f lung volumes, including
total lung capacity, functional residual capacity, and residual volume, as measured
by the helium-dilution technique or plethysmography.

114. The answer is C. With a pulmonary embolism, there is increased dead space
ventilation with a relatively higher proportion o f ventilation to perfusion (i.e., high
V/Q ratio). An area o f the lung is receiving oxygen, but due to thrombotic
obstruction o f pulmonary vessel, there is an area o f under perfusion. Pneumonia
represents a situation where inflammatory fluid due to infection impairs ventilation
while maintaining adequate perfusion (low V/Q). Pulmonary hemorrhage is similar
in that blood is present in airspace w hile maintaining adequate perfusion (low
V/Q). ARDS is an extreme case o f airway inflammation that compromises
oxygenation and sometimes ventilation while maintaining perfusion. Cyanide
toxicity represents a cellular inability to upload oxygen due to cyanide.
115. The answer is B. The oxygen haemoglobin dissociation curve represents the
relationship between the partial pressure o f oxygen and the oxygen saturation. The
affinity o f haemoglobin for oxygen increases as further m olecules o f oxygen are
bound. This results in a sigmoid-shaped curve until no further oxygen can be
hound. When this curve is shifted to the left, this represents a higher affinity o f
haemoglobin for oxygen at that given pressure. Conversely, when this curve is
shifted to the right this represents lower affinity and consequently oxygen is
released to the tissues more readily.
The curve is shifted
Figure 1.5 The oxygen
to the right by haemoglobin dosodetfon curve.
acidosis (decreased 090, diphotphogtycerxe.

pH), increased
temperature,
increased 2,3-
diphosphoglycerate
and raised C02. This
is analogous to
exercising muscles
and means that more
oxygen is made
available to the
tissues. At extreme
altitude the oxygen
haemoglobin
dissocation curve shifts to the left because there is much less C 02 in the blood (see
the figure).

COPD & Asthma & Respiratory Failure

116. Answer: B. Fluticasone A common side effect o f inhaled corticosteroids is


hoarse voice or dysphonia (a change in voice quality). The precise pathophysiology
is uncertain, but it likely is the consequence o f deposition o f corticosteroid onto the
larynx, with consequent irritative effect, mucosal thinning, or muscular atrophy.
The dysphonia tends to be intermittent and fully reversible with cessation o f
medication. It cannot be prevented by rinsing the posterior pharynx or
hypopharynx after each use of the medication. Sometimes this unpleasant side
effect can be ameliorated by addition of a valved holding chamber (“spacer”) to the
metered-dose inhaler being used to deliver the inhaled steroid^or changing the
delivery system (e.g., from a dry-powder inhaler to a metered-dose inhaler with
spacer). The inhaled steroid ciclesonide (Alvesco) is released as a pro-drug that is
activated by esterases along the bronchial mucosa. It is said to have fewer
oropharyngeal (and possibly laryngeal) side effects as a result o f this unique
mechanism o f activation. Although any inhaled medication can potentially cause
occasional hoarseness, none o f the other choices (salmeterol, tiotropium, albuterol,
or ipratropium) com m only causes this problem.

117. Answer: E . Sputum Gram stain and culture are not routinely necessary to
guide the choice o f antibiotics.
The bacterial pathogens causing exacerbations o f COPD are most often S.
pneumoniae, H influenzae, and M. catarrhalis, bacteria that can be treated with a
variety o f antibiotics chosen empirically (including macrolides, cephalosporins,
am oxicillin/clavulanate, trimethoprim-sulfamethoxazole, doxycycline, and
quinolones). Empiric treatment o f exacerbations o f COPD without need for sputum
Gram stain and culture has proven effective in numerous clinical trials. Although
m any flares o f COPD are triggered by events other than bacterial infection,
om ission o f antibiotics (Answer D ) when there is evidence for increased cough,
sputum purulence, and/or fever results in less frequent resolution and more frequent
deterioration than treatment with a short course o f antibiotics. Oral steroids given
for 2 weeks result in identical outcomes (in terms o f resolution o f the exacerbation
and risk o f recurrence) compared to 8 weeks (Answer A ), and recent evidence
suggests that 5 days o f therapy may suffice in many cases. Oral steroids are equally
effective compared with intravenous steroids in the treatment o f COPD
exacerbations with no greater incidence o f gastrointestinal side effects (Answer B).
N o n invasive mechanical ventilation, such as with bilevel positive airway pressure,
can reduce the need for intubation with mechanical ventilation in severe
exacerbations o f COPD (Answer C)

118. Answer Is: D . pH 7.27 P a02 7.2 kPa PaC02 6.9 kPa Bicarbonate 34.8
mmol/1 COPD (chronic obstructive pulmonary disease) has become the m ost
popular term to describe patients with spectrum o f chronic bronchitis and
emphysema. Pulmonary function tests commonly show the follow ing results:
Increased residual volume and total lung capacity Reduced vital capacity, FEV1,
peak expiratory flow rate, and FEVliFVC ratio. Respiratory failure is defined as a
Pa02<8kPa (60mmHg), and is divided into type I and type II. In type I respiratory
failure, PaC02<6.5kPa. Pa02 is low (hypoxaemic) but PaC 02 may be norm al or
low . In type II respiratory failure, the PaC02>6.5kPa and the P a02 is low . The
significance o f this classification is that in type II respiratory failure the patient
may have developed tolerance to increased levels o f PaC 02: in other w ords, the
drive for respiration no longer relies on hypercapnic drive (high P a C 0 2 ), but
instead on a hypoxic drive (low P a02). Therefore if the patient is given a high
concentration o f oxygen therapy, the hypoxic drive for ventilation may decrease.

119. The answ er is (E). Respiratory failure is a syndrome in w hich the respiratory
system fails in one or both o f its gas exchange functions: oxygenation and carbon
dioxide elimination. Respiratory failure may be classified as hypoxaem ic or
hypercapnic and m ay be either acute or chronic.
Hypoxacmic respiratory failure (type 1) is characterised by a low PaOJ with .t
normal or low PaC02. This is the most common form of respiratory failure* and it
can be associated with virtually all acute diseases of the lung, winch generally
involve fluid filling or collapse of ulvcolur units. Some examples of tyj'c I
respiratory failure urc cardiogenic or non-cnrdiogcnic pulmonary oedema,
pneumonia, acute pulmonary embolus and acute exacerbation of umIuim
Hypercupnic respiratory failure (type II) is cluraclm vrd hy a low P al)' and n Inpli
PuC02. Hie pll depends on the level of lm.arbou.itc, winch, in turn, i' dependent
on the duration ol hypercapnia ( omnn>n causes include drag ovndoM*.
neuromuscular disease, chest wall abnormalities and w e r e aiiwiv disoider. (c p
asthma, chronic obstructive pulmonary disease ((.’OPD) and ihroim pulinoii.iiy
emboli)

I2U. The mi*wer i« l>. (.’OPI) is a disease proiev* involving progrevave iduoim
uirllow ohfttniction because of chrome broncluhv crnphy'4-ma. or hmh (luoniv
bronchitis i* dclincd clinically as cxccv.ivr cough anil s|xiium production on most
days for at IcaaI I months during at least 2 cwnrcutive year. Lrnphyvetnu i.\
characterized hy chronic dyspnea resulting limn the destruction of lung tissue and
tfic enlargement .of uir spaces Asthma, which features airflow obstruction, airway
inflammation, and increased airway responsiveness to various sumuli. may be
distinguished from COPD by reversibility o f pulmonary function deficits. Acute
exacerbations of COPl) arc treated with oxygen (in hypoxemic patients), inhaled
02 agonists, inhaled anticholinergics, antibiotics, and systemic corticosteroids.
Theophylline may be considered in patients who do not respond to other
bronchodilators. Antibiotic therapy is directed at tbe most common pathogens,
including Streptococcus pneumoniae, Haemophilus influenzae, and Moraxella
catarrhalis. MiJd-to-moderalc exacerbations o f COPD are usually treated with
broad-spectrum antibiotics such as doxy eye line, trimethoprimsulfamethoxazolc,
and amoxicillin-clavulanate potassium. Treatment with extended-spectrum
penicillins, fluoroquinolones, third-generation cephalosporins, or aminoglycosides
may be considered in patients with severe exacerbations. The management o f
chronic stable COPD includes smoking cessation and oxygen therapy. Inhaled 02
agonists, inhaled anticholinergics, and systemic corticosteroids are also used in
patients with chronic stable disease. Inhaled corticosteroids decrease airway
reactivity and can reduce the use o f health care services for management o f
respiratory symptoms. Avoiding acute exacerbations helps to reduce longterm
complications. Long-term oxygen therapy, regular monitoring o f pulmonary
function, and referral for pulmonary rehabilitation are often .utilized and can
improve the quality o f life and reduce hospitalizations.
Influenza and pneumococcal vaccines should be administered. Selected patients
who do not respond to standard therapies may benefit from lungrcduction surgery.

121. The answer is C. Exercise-induced asthma occurs mainly in patients already


diagnosed with asthma. Wheezing usually begins shortly after the initiation o f
exercise mid can he debilitating mid limit participation Shoil acting 02 *jj/ouc:ts
( | S to .10 minutes before exercise) arc recommended first-line agents for
pharmacologic treatment, although Icukotncne rcurptor antagonists or inhaled
corticosteroid* with or without long-acting p2 agonists may be needed in refractory
cones, If symptoms persist despite treatment, alternative diag/vnca such as cardiac
or other pulmonary etiologies, vocal cord dyKfimction, or anxiety should be
considered I he regular use o f inlialcd corlicmlcroiiLn is best in the setting of
asthmatic symjrlnm.s occurring in addition to exercise. f >tlier preventive measure.
include a slow warm-up and the avoidance ol very warm and very cold condition:*

122. The answer in IJ. AAthma is a revennblc obstructivu lung disorder dint is
cnaractcri/ed by rc v.1i ve airways. I he u/nd ition is thought to be inherited;
however, some individuals may be affected without a family history. Many facton
may precipitate an attack, including infections, smoke, cold weather, exercise, toxic
furncs, and stress Symptoms include wheezing, shortness o f breath, tachypnea,
tough, and tightness or pressure in the chest The mainstay o f acute treatment
(rescue therapy) is an inhalant form o f a 02-adrcoergjc agonist, such as albuterol.
Inhaled corticosteroids and salmctcrol (a long-acting 02-adreneTgic agonist) are
used in chronic therapy. For patients who have more severe asthmatic attacks, short
courses o f oral corticosteroids may be necessary, particularly with upper
respiratory infections. Theophylline, once readily prescribed, is used less
frequently, and its benefits are controversial. Cromolyn sodium, a mast-cell
stabilizer, ipratropium bromide, an anticholinergic medication; and the leukotnenc
modifiers can be used for chronic asthmatic conditions. Pulmonary function tests in
patients affected with asthma usually show a normal or decreased vital capacity,
decreased forced expiratory volum e in l second, increased residual volume,
increased total lung capacity, and a positive response to inhaled bronchodilators. In
children, rescue 02-adrenergic agonists are the treatment o f choice for mild
intermittent asthma.

123. The answ er b E. Medications used in the treatment o f asthma ore divided
into long-term control medications that are taken regularly and quick-relief (rescue)
medications that are taken as needed to relieve bronchoconstriction rapidly.
Long-term control medications include anti-inflammatory agents (corticosteroids
and leukotriene modifiers) and long-acting bronchodilators.
Quick-relief medications include short-acting 02 agonists, anticholinergics, and
systemic corticosteroids. Patients whose symptoms are mild and infrequent can use
a short-acting bronchodilator as needed for relief o f symptoms. Patients with more
frequent cough, wheeze, chest tightness or shortness o f breath should begin
treatment with u long-term controller medication. Low daily doses o f an uihalcd
corticosteroid suppress airway inflammation and reduce the risk o f exacerbations.
For patients who remain symptomatic despite compliance with inhaled
corticosteroid treatment and good inhulationai technique, addition o f a Jong-acting
02-agonist is recommended. Patients with more severe disease may need higher
doses of inhaled corticosteroids.
Corticosteroids remain the most potent and effective anti-intlammalory agents
available for the management o f asthma. An inhaled corticosteroid is the most
effective long-term treatment for control of the disease in treating all types of
persistent asthma in patients of all ages. For long-term use, inhaled steroids are
preferred over oral steroids because the inhaled agents have fewer systemic side
effects. However, oral steroids are the most effective drugs available for
exacerbations o f asthma incompletely responsive to bronchodilators. Even when an
acute exacerbation responds to bronchodilators, addition o f a short course of an
oral steroid can decrease symptoms and may prevent a relapse. For asthma
exacerbations, daily systemic steroids are generally required for only a few days,
without a taper.
Leukotriene modifiers (montelukast and zafirlukast) air less effective alternatives
for those who cannot tolerate low-dose inhaled steroids. Leukotriene modifiers arc
also generally less effective than an inhaled iongacting {3 agonist. Both o f these
classes are not recommended for treatment o f acute asthma symptoms.
Salmeterol (Serevent) is a king-acting (32 agonisL Its mechanism of action and side
effect profile are simitar to those o f other (32 agonists. Unlike the short-acting
agents, salmeterol is not intended for use as a quickrdief agent
Theophylline, once the mainstay o f asthma treatment is now' considered a second-
or third-line agent because of its adverse effect profile and potential interactions
with many drugs. Furthermore, serum theophylline levels have to be monitored
during treatment In addition to its well-known broncbodilator effects, theophylline
has anti-inflammatory activity. Currently, theophylline therapy is generally
reserved for use in patients who exhibit nocturnal asthma symptoms that are not
controlled with high-dose anti-inflammatory medications.
Zafirlukast (Accolate) and zileuton (Zyflo) antagonize the action of leukotrienes at
their receptor (zafirlukast) or inhibit the lipoxygenase pathway (zileuton). Both
drugs are approved for the management o f chronic asthma in adults and in children
older than 12 years. Zafirlukast and zileuton have numerous drug interactions.
Short-acting inhaled (32 agonists are the agents o f choice for relieving
bronchospasms and preventing exercise-induced bronchospasms. Selective 02
agonists, including albuterol, bitolterol (Tomalate), metaproterenol (Aiupent),
pirbuterol (Maxair), and tcrbutaJine (Brethaire), are preferred to nonsclective 0
agonists such as epinephrine, ephedrine, and isoproterenol (Isuprel) because the
selective agents have fewer cardiovascular side effects and a longer duration of
action. Inhaled 02 agonists have a rapid onset of action. Short-term systemic
corticosteroid therapy is useful for gaining initial control o f asthma and for treating
moderate-to-severe asthma exacerbations. The intravenous administration of
systemic corticosteroids offers no advantage over oral administration when Gi
absorption is not impaired. Ipratropium is a quaternary atropine derivative that
inhibits vagalmediated bronchoconstriction. It may be useful as an adjunct to

on
inhaled 02 agonists in patients who have severe asthma exacerbations or who
cannot tolerate 02 agonists.

124. The answer is E. The respiratory causes o f clubbing include broochogeruc


carcinoma (C), empyema (A), mesothelioma (B), cystic fibrosis (D ), lung ahsccss,
fibrosing alveolitis and bronchiectasis. In patients with CO PD, the signs that ma>
be seen in the hands are carbon dioxide retention tremor, peripheral cyanosis and
tar staining in the fingertips.

125. The answer b D). Polycythaemia is commonly found in COPD pauents and
other respiratory diseases where gts transfer is reduced. Chronic hypoxia
stimulates the production o f erythropoietin from the kidneys, which acts on the
bone marrow' to augment the oxygen carrying capacity of the blood. Usually this is
the most important derangement that is found in the full blood count in the absence
of an exacerbarioa.

126. The answer is B). The panexit h is an acute respiratory alkalosis with
associated hypoxia. This is consistent with an acute asthmatic attack. A normal or
rising CO2 is an ominous sign indicative o f a life threatening attack and the need to
consider ventilatory support. Patients with hypcrventiladon syndrome do show a
respiratory alkalosis but this is not associated with hypoxia.

127. Answer is (A). The diagnosis is suggested by the presence o f widespread


emphysema on chest CT scan in a oonsmoker. The presence o f widespread
(panlobular) emphysema in a oonsmoker should raise the possibility o f alpha-1
antitrypsin deficiency; other suggestive findings include COPD that develops in 2
cigarette smoker at an unusually young age (e g ., before age 50) or a strong family
history o f emphysema. However, many patients with hom ozygous alpha-1
antitrypsin deficiency do not have these “classic" characteristics, and som e experts
recommend widespread testing for alpha-1 antitrypsin deficiency among patients
with COPD. Testing for alpha-1 antitrypsin begins with measurement o f serum
alpha-1 antitrypsin level, a routine blood test done in the chemistry laborator
Patients who are homozygous for alpha-] antitrypsin deficiency w ill have a very
low blood level o f the protein, on the order o f 15% o f normal. This finding reflects
the fact that most patients with alpha-1 antitrypsin deficiency make an alpha-]
antitrypsin protein that is transported ineffectively out o f the liver, rather than
making no alpha-1 antitrypsin protein (the homozygous null/null genotype is rare).
Further testing o f the patient with a very low serum alpha-1 antitrypsin level can be
performed by protein electrophoresis or genetic analysis to determine the specific
genetic abnormality.

128. The answer is E. Cor pulmonale is defined as right heart failure caused by
chronic pulmonary hypertension. Patients usually present with dyspnoea,
fatiguc/syncope Signs include cyanosis, tachycardia, raised JVP, right ventricular
beavi, loud P2 - passystolic m ura^r, early diastolic (Graham Sicvl munnurL
hepatomegaly and oedema. Causes can be categorized iz:o. (1) Lung disease
severt dycric aflbna, COPD (DL bronchiectasis. pulmonary fibrosis (A \ lung
rrsccfiocu (>i Pulnsocary vascular disease: pulmonary emboli. pulmonary
vasculitis primary ;ximoaarv hypertension (B \ acute respiratory distress
syndrome, stcjde-ceil disease, parasite iofesatxxi: (3) Thoracic cage aboomsahts
kyphosis, scoliosis, dwracoplasjN- (d) Neuromuscular disease: mvasthenn ctjm>
•C \ >>’.ionj\elitis. raotar neurone cLsease; (53 I lypov cr.uiudon sleep apnoca.
enlarged adenoids in children aid cerrbccv oscular disease Therefore. from the list
it rs obvious that multiple sclerosis is not one of the many causes o f cor pulmonale.

Respiratory Infections

12$. A nsw er B. Serum immunoglobulin G. Many cases o f bronchiectasis are


•idiopathic,” presumably the result o f a poor necrotizing infection that caused
irreversible damage to the airway walls. In other msuuxxs. however, an underlying
predisposition can be identified. .Among the potential conditions that may
predispose to the development o f bronchiectasis are hypogammaglobulinemia and
primary ciliary dyskinesia (also called unmodle cilia syndrome). Diagnosis o f the
former may result in gammaglobulin replacement therapy, which would be
expected to slow the progression o f the disease. Diagnosis of the latter (primary
ciliary dyskinesia) by electron microscopic examination of nasal or bronchial
epithelial tissue and associated ciliary uhrastructune (Answer D) does not lead to
any therapeutic options that would alter treatment at this time. Pulmonary
vasculitides, such as granulomatosis with polyangiitis (Wegener’s syndrome) or
eosinophilic granulomatosis with polyangiitis (Churg-Strauss syndrome), do not
typically cause bronchiectasis, making assay for antinuclear cytoplasmic antibodies
(ANCA) associated with vasculitis unhelpful (Answer A). Celiac disease (Answer
C) is associated with idiopathic pulmonary hemosiderosis (called Lane-Hamilton
disease) but not bronchiectasis. The etiology o f bronchiectasis cannot be discerned
by detailed characterization o f its physiologic consequences on pulmonary function
testing (Answer E).

130. The answ er is A In young adults, causes for pneumonia include


Mycoplasma, Chlamydia pneumoniae, influenza, adenovirus, Pneumocystis carinii
(in immunocompromised patients), and other community-acquired organisms
including Streptococcus, Haemophilus, and, occasionally, Legionella. Pneumonia
in adults with no underlying disease is usually caused by S pneumoniae,
representing more than 50% o f commumlyacquircd pneumonias that require
hospitalization Oliver causes in this patient group include H. influenzae,
Legionella, Mycoplasma (more commonly seen in young adults), and influenza
viruvri If the patient is older titan 00 yean and lias other significant medical
prohb-rn* f? g , diabetes, COPD, heart disease, alcoltolism;, the most common
pathogens iie.fud*: the previously rnrnU'/ned organisms as well as Klebsiella,
I ntcrohactcnace.sc, Chlamydia. and S aureus, For patier.ts with aspiration ox
txistvonu.il infections, the causative organisms include the previously mentioned
organisms and the gram-negative organisms uncludxg Pseudomonas) and
anaerobes

131. The anvwer is A. Cougiung is pun of the body’s infection protective sy stem
and helps remove panicles and material from the airway. In some eases, the patient
may experience a chronic cough that can be attributed to a number ot different
problems, including po&tnasal dnp (most common cause), gastroesophageal reflux,
and bronchoeonstriction as seen in coughvariant asthma patients. Other common
associated conditions include the use of ACE inhibitors, chronic bronchitis seen in
smokers, and bronchiectasis Treatment involves eliminating the underlying cause.
Treatment o f conditions, such as asthma and COPD, may involve the use oi
bronchodilators 0 agonist or thcophyliincX cromolyn sodium, and inhaled steroids,
the treatment o f postnasal drip may involve the use o f antihistamines and topical
nasal steroids. Patients should be informed that it may take 8 to 12 weeks before
their cough improves when using inhaled steroids. Treatment o f gastroesophageal
reflux involves the use o f antacids, H2-rcceptor blockers, and PPIs. Eliminating a
cough caused by ACE inhibitors usually takes several days before improvement is
seen. Unfortunately, it is more difficult to treat patients with chronic bronchitis.
The use o f antibiotics with the absence o f supporting symptoms suggestive o f
infection is not useful and should be avoided. M ucolytics have not been shown to
be beneficial.

132. The answer is C. ARDS is characterized by respiratory distress that may be


caused by different insults. Most cases o f Ai^DS in adults are associated with
pulmonary sepsis (46%) or oonpulroonary sep sis (33%). Risk factors include those
causing direct lung injury (e.g., pneumonia, inhalation injury, pulmonary
contusion) and those causing indirect lung injury (e.g., non-pulmonary sepsis,
bums, transfusion-related acute lung injury). The mechanism o f injury involves
damage o f capillary endothelial cells and alveolar epithelial cells, which leads to
pulmonary edema with decreased pulmonary com pliance and decreased functional
residual capacity. Symptoms include rapid on set o f dyspnea usually 12 to 48 hours
afler the insult, with wheezing and intercostal retractions. Laboratory tests show
hypoxemia that responds poorly to oxygen administration, requiring frequent
monitoring o f arterial blood gases. Radiographs show diffuse or patchy alveolar
and interstitial infiltrates without cardioxnegaly or pulmonary vascular
redistribution.
When treatment with mechanical ventilation is required, the use o f low volume
ventilation with higher positive end-expiratory pressure values (12 to 18 or more
cm 1120) for the initial mechanical ventilation is recommended. Fluids should be
minimized. Patients arc usually paralyzed w ith pancuronium. Antibiotics are
unnecessary, but nosocomial infections may develop. Most patients with the
:! AiDS io c bjr-c z:
z t i c z z :*e orzin whch is the major ci-se
:: ~ < = i:L McrtLir.- m s z z i ^ 'd l.

133. The answer b D*. rLserccphiliiS influenzae iE) and Streptococcus pneumonia
fC) are organrsns which arc usually responsible for community-acquired
pceumoma. S. aureus (A) and Pseudomonas spp. (B) arc usually found in patients
with hospital-acquired pneumonia. L. pneumophilia (D). along with Chlamydia
spp. and Mycoplasma pneumoniae, arc the atypical pneumonia-causing organisms.
A urinary' antigen test is routinely used for the detection o f Legionella spp.
Serological tests can be used for the detection o f My coplasma and Chlamydia spp.
and also Legionella spp.

134. The answer is C: From the answers, amoxicillin (A), erythromycin (B) and
flucloxacillin (D) are all antibacterial agents and do not target fungal infections.
Therefore the only possible answers arc C and E. Fluconzaole (E), although a
widely used antifungal, is not effective against Aspergillus spp. Amphotericin B.
along with other antifungals such as voriconazole, itraconazole and caspofungin, is
effective in treating Aspergillus infections which can affect the lung in five way's:
(1) Type-1 hypersensitivity reaction causing atopic asthma through inhalation o f
fungal spores; (2) Allergic bronchopulmonary aspergillosis (ABPA) which results
from a type-3 hypersensitivity reaction to A. fumigates; (3) Aspergilloraa
(mycetoma) which is a fungus ball within a pre-existing cavity, often caused by
tuberculosis and sarcoidosis; (4) Invasive aspergillosis which occurs in
immunocompromised patients, SLE, bums and after broad-spectrum antibiotic
therapy; and (5) Hypersensitive pneumonitis.

135. The answer b D: This patient is suffering from peripheral neuropathy


secondary to isoniazid (D) administration. Lsoniazid is associated with depletion of
vitamin B6 (pyridoxine) and therefore results in peripheral neuropathy. Other side
effects include rash, deranged liver function (due to the fact that this drug is
metabolized in the liver by acetylation and dehydrazination and converted to
ammonia compounds which can cause hepatitis), CNS effects and sideroblastic
anaemia.
The common side effects o f pyrazinamide (A) (a pro-drug that inhibits growth of
NL tuberculosis) include arthralgia, hepatoxicity, gastrointestinal disturbances,
rash, pruritus and sideroblastic anaemia. Side effects of rifarapicin (B) (a
cytochrome P450 inducer) include hepatoxicity, fever, gastrointestinal
disturbances, rash and can also cause the urine and tears to become an orange-red
colour which is considered a benign side effect. The side effects associated with
ethambutol (C) (a bacteriostatic drug that inhibits the formation o f the ceil wall in
M. tuberculosis) include optic neuritis, red-green colour blindness, peripheral
neuropathy and vertical nystagmus. Although ethambutol could also be a possible
answer here, the clue given in the question points to the patient having stopped
taking has v-ianins *ah.ch are usuaiiv gives ir. conjunction A ii tiOniazaa
trcarmen!.

136. The answer is C: Chronic bronchitis is one of the most common respiratory
diseases due to cigarette smoking. The smoking history and productive cough for at
least years is indicative of chronic bronchitis.

137. The answer is B: Ludwig's angina usually develops from a penodontal or


dental infection and is one of the most common neck space infections. The
condition is usually a rapidly developing, bilateral cellulitis that aflects the
sublingual and submaxillary space, without involvement o f the lymph nodes or
formation o f abscesses. The infection usually rapidly arises from the second and
third mandibular molars as a result o f poor dental hygiene, tooth extraction, or
trauma. Symptoms include edema and erythema of the upper neck (under the chin)
and floor o f the mouth, trismus, drooling, dysphoma, dysphagia, and dyspnea.
Fever, chills, and tachycardia are usually present. Tongue displacement upward
may also occur and threaten the airway. In severe cases, the condition may be fatal.
Treatment includes protection of the airway in severe cases and IV antibiotics (e.g,,
penicillin, widespectrum cephalosporins) in high doses to cover anaerobic
organisms (Bacteroides). Incision and drainage may be required.

138. The answer is D: Bronchiectasis is defined as chronic infection o f the bronchi


and bronchioles leading to permanent dilatation o f these airways. The main
organisms involved in this condition are H. influenzae, S. pneumoniae, S. aureus
and P. aeruginosa. Answers A-C and E are all known causes o f bronchiectasis. The
causes can be divided into: (1) Congenital: cystic fibrosis, Young's syndrome,
primary ciliary dyskinesia, Kanagner’s syndrome; and (2) Acquired: Post-infection
with measles, pertussis, bronchiolitis, pneumonia, TB and HTV. Other acquired
causes include bronchial obstruction secondary to tumours or foreign bodies,
allergic bronchopulmonary aspergillosis (ABPA), hypogammaglobulmaemia,
rheumalois arthritis, ulcerative colitis and idiopathic.

Intestitial Lung Diseases

139. Answer C: Relapsing polychondritis, the rarest o f the collagen-vascular


diseases listed, is associated with airway disease, including tracheomalacia and
subglottic stenosis, but not with diffuse interstitial lung disease. The other choices
are all associated with interstitial lung disease. In addition, scleroderma ls
associated with pulmonary hypertension and with esophageal and sometimes
oropharyngeal dysfunction with an increased risk o f aspiration pneumonia.
Systemic lupus erythematosus can cause pleural effusions, diffuse alveolar
hemorrhage, and “shrinking lung syndrome," possibly due to diaphragmatic
weakness. Mixed connective tissue disease may also cause pulmonary
hypertension.
140. Answer is D: Amiodarone. Pulmonary disease caused by medication is well
recognised. Amiodarone is a frequently prescribed medication and a well known
cause o f pulmonary fibrosis. Amiodarone is a class 111 anti-arryhthmic drug
commonly used in heart failure patients. Its side effects include fatal interstitial
pneumonitis, diffuse alveolar damage and pulmonary fibrosis. Other drugs which
are known to cause pulmonary* fibrosis include busulfan. bleomycin, methysergide
and c>clophospamidc. These drugs are largely used m the treatment of patients
with cancer, rather than the cardiac arena.
Of the other drugs in the list above, the only other ooc which has an important
effect on the respiratory system, is afcaiolol (a beta-Nocker). These agents are
relatrvc 1ycontra-indicated in asthmatics since they can cause broncho-coostricoorL

141. The answer is D: Ichopatiuc pulmonary fibrosis, a form o f interstitial lung


disease, results when there is inflammation ct the lung tissue with resulting
fibrosis. A toxic exposure cr antigenic response is thought to precipitate the
inflammatory process. Affected paaass may report a gradual, increasing shortness
o f breath; a dry cough, and generalized fatigue whh lack o f endurance with
physical exercise- Physical examinafioo shows bfhasflg ary rales, clubbing, and.
occasionally, cyanosis- In advanced disease, chest radiographs show-
honeycombing. and pulmonary fm cio c tests show a restrictive pattern with
reduced vital capacny, diffusing capacity for carbon monoxide, and total lung
volume. In addition, there is a normal or increased forced expiratory' volume in 1
second to forced vital capacity ratio. Arterial blood g2 ses may show mild
hypoxemia, but bypcrcarbia is rare. The patient’s ESR may be increased.
Transthoracic or transbroochial biopsy is usually needed for a definitive diagnosis.
The treatment of idiopathic pulmonary fibrosis is controversial because of a lack of
understanding o f the natural history o f the disease. Only 10% to 15% o f patients
improve with corticosteroid therapy, and 26% o f patients develop serious
complications from the steroid therapy. Indicators of good response to steroid
therapy include young age, female gender, ground-glass lesions on CT scan, and
active inflammation on lung biopsy samples. Azathioprine, cyclophosphamide, and
other cytotoxic drugs have been used as second-line agents or in combination with
steroids as first-line therapy. Although the general prognosis was poor, combined
treatment improved 3-year survival rates. Selected patients with idiopathic
pulmonary fibrosis have been treated with lung transplantation.

142. The answer is C: Idiopathic pulmonary fibrosis is a progressive fibrotic


disorder o f the lungs that occurs most frequently in individuals over 60 years o f
age. To date there is no cure or effective treatment for this disease and response to
steroids is < 10%. The remaining diseases all have a significant response to
corticosteroids although relapse is common in BOOP and sarcoidosis.

J43. The answer ts D: Fibrosing alveolitis associated with rheumatoid arthritis is


Locir^nguishible from cryptogenic fibrosing alveolitis. It presents with progressive
breathlessness ami couglv Signs include linger clubbing, cyanasis ami bilateral cut!
inspiratory crackles Other pulmonary complications ol Rheumatoid arthritis
include pleural cfhisions, empyema. cryptogenic organising pneumonia,
bronchiectasis and pulmonary nodules. The latter arrusually asymptomatic but may
cavitate resulting in haemoptysis and when cveumng with coal workers
pneumoconiosis (Caplans Syndrome) are associated wyJibrcathJcssness.

144. The answer is C: The restrictive lung pattern together with the reduced T!CO
and KCO suggest lung fibrecas

Lung Tumors

145. Answer is E: Small (021 ) cell carcinoma SlADH involves the inappropriate
secretion o f ADH which leads to retention o f waier and hspocuiraaE^a. SLADH
may be catted by many rumours, such as prostate, thymus, pancreas.: and
lymphomas. b o w ser the co st common tuoem rarsr-.c, SLADH axe small cel* *
carcinomas et the hmg. Sralt cell esreznoesas teas cell carcipogu s) orien ongiaste
from APUD cells (amine precursor decarboxylase ccOs- neuroendocrine
cells). There is a high occurence o f paraneoplastic syndromes rssiviaind with this
type o f mmoar, so prcsecubs: can be very varied.

146. The answer b B: I cancer is cites with a perancoplasac


syndrome, which occurs as a result o f cancer and is extripuhaonary. The follow ing
are some common neoplastic syndromes.
• Squamous cell: hypercalcemia
• Small cell: Cushing's syndrome. SlADH with hyponatremia, m yas&emc
syndrome, Eaion-Lambert syndrome, peripheral neuropathy, suhacmr cerebel lar
degeneration
• Large cell: gynecomastia
• Adenocarcinoma: dubbing, thrombophlebitis, marantic endocarditis, periostitis,
or hypertrophic osteoarthropathy
In addition, all o f the previously mentioned lung cancers may be associated with
dermatomyosltis, disseminated intravascular coagulation, eosinophilia,
thrombocytosis, and acanthosis nigricans.

147. The answer is B: Noo-raetastatic paraznalignant manifestations for small cell


carcinoma include inappropriate ADH and ectopic ACTH secretion, Eaion-
Lambert syndrome (70% occurs in association with small cell carcinoma -
autoimmune disorder affecting release o f acetylcholine at neuromuscular junction
causing proximal muscle weakness, fatiguability and muscle wasting. Often, power
is increased initially by exercise - reversed myasthenis effect Weakness and
fatiguability can be improved with guanidine hydrochloride), polymyositis,
dementia, cerebellar syndrome and peripheral neuropathy. Cutaneous lesioos
(dennatomyositis, thrombophlebitis migrans, acanthosis nigricans and erythema
gyratumrepens) arc rare. HPOA and ectopic PTH*re)ated peptide secretion relates
particularly to squamous cell carcinoma.

148, The answer is E: Superior vena cava obstruction (SVCO) interrupts venous
return from the head, arms and thorax to the right atrium resulting in facial
swelling, stndor, cough, breathlessness, hoarseness, headache etc. It was first
described by William Hunter in 1757 in a case of syphilitic aortic aneurysm but
these days the commonest cause is malignancy and in particular bronchial
carcinoma and lymphoma. Treatment is of the underlying condition, chemotherapy
for small cell lung cancer and lymphoma and radiotherapy for non-small cell lung
cancer Sometimes patients require stenting of the SVCO for symptomatic relief
prior to more definitive treatment. Pleural effusions lire a contni-indication to
surgery if they are malignant but effusions in patients with bronchial caremoma
may be reactive.

149. The answer is D: Pancoasf s tumours are defined as tumours arising from the
lung apex cither on the left or right side. As the tumour grows it can compress
structures such ns the brachiocephalic vein, subclavian artery, recutTcnt laryngeal
nerve (causing voice hoarseness (A)), vagus nersv, phrcuic nerve or compression
of the sympathetic ganglion resulting in a group of symptoms known as Homer's
syndrome (miosis (B) - pupil constriction, cnopthalntos - sunken eye, ptosis (E) -
drooping eyelid and ipsilateral anhydrosis (C) - loss of sweating due to
compression of sympathetic supply (thoracic outlet) to the face). Exopthalmos (D)
is defined os the appearance o f protruding eye and is seen in patients with Grave's
disease.

Hemoptysis

150. Answer is C: Normal renal function A CT pulmonary' angiogram (CTPA) is a


diagnostic test employed to image the pulmonary’ arteries, its main use is to
diagnose pulmonary embolism. In order to cany out a CTPA, contrast is
administered intravenously. Radiological contrast media are nephrotoxic. This
effect is dose dependent, and in many patients the effect is mild, transient, fully
reversible and of no clinical significance. The risk of contrast nephropathy is
amplified by the presence of coexisting conditions, which include: Renal
impairment Hypovolaemia Low cardiac output Diabetes mellitus (especially if
treated with metformin) Hyperviscosity (myeloma). For this reason, and in
answering the question above, one MUSI' conduct a renal function test prior to
requesting a CTPA (or other contrast imaging investigation), in order to prevent
contrast nephropathy. An estimated glomerular filtration rate (eGFR) is probably
most useful in most cases. One of the rights bestowed on a newly qualified doctor
is that o f a referrer for imaging investigations. An essential element of this is to
check the preconditions, such as renal function, before requests are made.
151. The answ er is B: Chest pain is common in patients with pulmonary embolism
(PE). When evaluating a patient for possible PE, the presence o f orthopnea
suggests heart failure, fever suggests an infectious process, wheezing suggests
asthma or chronic obstructive pulmonary disease (COPD), and rhonchi suggest
heart failure, interstitial lung disease, or infection. These generali/ulions are
supported by a 2008 study designed to improve the diagnosis o f PE on the basis of
the history, physical examination. EKCi, .uid chest radiograph

152. The answer U II: Hemoptysis is the presence o f blood in the expectorate
fnlmpulmonnry causes include infections (c g . bronchitis, pneumonia, tuberculous,
fungal infections), neoplasm, bronchiectasis, pulmonary embolus, AV
malformations, Goodpasture's disease. vasculitis, Iniuinn, or ilio presence o f a
foreign body. Extrapulmona/y causes include (II bleeding. ('HE with pulmonary
edema, severe initrnl stenosis, rpixcixix, or other conditions (including
disseminated intravascular coagulation) Most cases arc sell-lunilcd mid in p iiic no
additional workup, however, persistent or severe hemoptysis should be evaluated
wit]) sputum collection for (imm's stain and uiltm c, cytology, add-lust bacillus
Stains. CBC, PT, PIT, chest r.idiograph. and flexible bronclioscopy. If a lowri
^respiratory tract source is suspected, ihc patient should undeigo a chest \-n iy lin t,
and if a mass is noted, bronchoscopy should he performed A high-resolution C l
may be helpful in the diagnosis It is important to disttnguisli Ik:tween OI blood
loss (which has a dark red color and acidic pH) and true hemoptysis (which is
typically bright red in color and alkaline).

153. The answer is A: Warfann depletes vitamin R dependent coagulaticm factors


II, VII, IX. X, and proteins C and S. EFP contains all o f these plus several other
coagulation factors. Cryoprecipitate that is derived hom FFP only contains factors
VIII, XIII. fibrinogen, fibroncctm, and son Willcbrand factor. Choice B is incorrect
as checking the patient's complete blood count (CBC) may not fully quantify blood
loss but can serve as a baseline. Choice B is correct for this case o f a patient who is
known to be on warfarin and develops sudden hemoptysis. It is prudent to request
FFP, as this w ill take several minutes to thaw units o f FFP. FFP w ill effectively
replace the coagulation factors that are roost likely inactivated by warfarin. Vitamin
K administration could be used in less severe bleeding as it would allow time for
vitamin K to reactivate the warfarin inactivated factors. Cryoprecipitate does not
contain the factors necessary to reverse the effects o f warfarin. Transfusion of
uncross matched red blood cells is an important step in the management o f patients
with massive bleeding but will not address the underlying medication-induced
coagulopathy. With bolding his medication (warfarin) will not make a significant
impact on his current state and may Deed to be reinforced when patient is more
stable and is ready for discharge.

Qfl
154. I lie nn*wer h I'- Pulmonary lubcrailosn (A), IlfonthieihiMs (II),
A t|« r^illuinu ((') jiikI Wegener's iuiiiuiloniulo?«i?t (I)) cause h:icmoplysis Oilier
causes include:
• bronchogenic carcinoma,
• pnlmomiry abscess;
• liutners lung;
• pulmonary em bolus,
• liooilpuslurc's syndrome.
Asthma ( l ) does not result in patients presenting with haemoptysis

155. The answer is C : Major risk factors lor pulmonary emboli include:
• Immobility.
• Abdominal/pdvic surgery.
• Malignancy labdommaLpelvic. advanced metastatic).
• Lower limb fracture.
• Pregnancy.
• Hip/knec replacement.
• Previous venous thromboembolism.
Minor risk factors include:
• Oral contraceptive pills.
• Hypertension.
• Long-distance travel.

156. The answer is D: Cystic fibrosis is cot a disorder that is seen very often. On
examination o f the chest in a peiient suffering from cystic fibrosis you would
expect to bear coarse inspiratory' crackles due to the production o f thick secretions
and difficulty o f the mucociliary system in clearing these. The result is coarse
crepitations heard on auscultation.

157. The answer is A: In the setting o f chronic inflammation, such as occurs with
bronchiectasis, bronchial arteries locally increase in size and number. These are
susceptible to rupture, especially with coexistent infection or coughing. Bleeding
can be significant because the vessels are under the high pressure o f the systemic
circulation. In contrast, the pulmonary arterial circulation rarely results in bleeding
because o f its low-pressure characteristics and because o f autoregulation to divert
blood flow away from diseased portions o f the lung (hypoxic pulmonary
vasoconstriction). The pulmonary capillaries arc not the source o f bleeding in
bronchiectasis. They are the origin o f bleeding in diffuse alveolar hemorrhage,
which can occur in a variety o f conditions, including collagen vascular diseases.

Pleural Diseases

158. The answer is E: Pneumothorax is the accumulation o f air within the pleural
space The usual cause o f pneumothorax is a penetrating wound such as a stabbing
wound, gumliot wound, or dcccIcration-typc injury (e g., as seen in motor vehicle
accidents). Spontaneous pncumolhorax can also occur and typically affects tall,
thin men or smokers (as a result o f a ruptured bleb)
Clinical findings include decreased breath sounds on the side affected, shortness of
breath, chest pain (most common symptom), cough, distended neck veins, and
hypotension A chest radiograph is usually diagnostic.
Treatment may require immediate intervention but in many cases depends on the
extent of pneumothorax. If pneumothorax involves up to 15% to 20% o f lung
volume, observation is the only treatment necessary. Supplemental oxygen ;s
usually administered, and most cases resolve in 10 days. For larger
pneumothoraces, chest tube placement is necessary. Tension pneumothoraces
require emergent decompression with a large-bore needle placed in the second
intercostal space o f the midclavicular line, followed by chest tube placem ent
Providing 100% 0 2 accelerates the rate o f pleural air absorption.

159. The answ er is C: The fact that there is reduced air entry, dullness to
percussion in the lower and midzoncs of the right lung and reduced chest
expansion, indicates that there is most likely to be a pleural effusion (C) from the
list o f answers above. ‘Stony dullness' is usually used to describe the presence o f a
pleura] effusion but, in clinical practice, distinguishing between dullness and stony
dullness can be quite challenging for even the most experienced clinicians.
Pulmonary embolism (B) does not usually present with any chest signs. Pneumonia
(A and E) and bronchial carcinoma (D) can lead to a secondary pleural effusion,
but during the initial stages w ill present with bronchial breathing over the affected
area o f the lung.

160. The answ er is A: Hyper-resonance coupled with pleuritic chest paw and an
acute onset from the history strongly points to a diagnosis o f pneumothorax.
Pneumothoraces are usually spontaneous in young thin men and tend to occur due
to subpleural bulla rupture. Some other causes include asthma, COPD, TB,
pneumonia, connective tissue disorders (e g, Marfan's syndrome, Ehlers-Danlos
syndrome), trauma, iatrogenic (e.g. pleural aspiration/biopsy, percutaneous liver
biopsy, etc.) There is no history o f cough and, in addition, the patient is afebrile
with no bronchial breathing over the chest wall to suggest pneumonia (B ). With
pleural effusion (C) there would be dullness to percussion and reduced air entry on
the affected side; there would also be a more chronic onset o f symptoms from the
history. Lung fibrosis (D) would not typically present with signs o f hyper­
resonance and furthermore, disease is usually bilateral with air entry Fine
inspiratory crackles are beard on auscultation.
Although pneumothoraces can develop from chest trauma, there is uo history o f
this from the question stem, making a traumatic chest injury (E) unlikely

Miscellaneous
161. Answer is A: Ketrognathia. Persons with a nano wed posterior phar\ngcal
opening are at increased risk for critical nanowing and occlusion of the upper
airway dunng sleep. The posterior pharynx may be nanowed due to a large tongue,
posteriorly extending soft palate, large tonsils, or posteriorly positioned jaw and
tongue (retrognathia). A large neck circumference (greater than 16 inches m
women aod 17 inches in men) conelates with excess fatty deposition in the upper
airway (Answer B). Nasal obstruction, such as with large nasal polyps (Answer C),
can aggravate pre-existing obstructive sleep apnea, but it is not a risk factor for the
presence of obstructive sleep apnea. Short stature in the absence of obesity is not a
risk factor for sleep-disordered breathing (Answer D). Insomnia (Answer L) may
be a manifestation of obstructive sleep apnea syndrome, but it is nonspecific and
not closely associated with the presence of obstructive sleep apnea.

162. Answer D: Motor vehicle accidents. Persons with obstructive sleep apnea
(OSA) have a 7-fold increased risk o f being involved in a motor vehicle accident as
a consequence o f their associated daytime liypersomnolennce. Strong
epidemiologic evidence indicates that OSA is also an independent risk factor for
adverse cardiovascular events. These include heart attacks and strokes, as well as
cardiac arrhythmias and sudden death. In addition, OSA is a cause o f hypertension,
likely as a result o f repetitive sympathetic stimulation associated with episodic
hypoxemia and sudden awakenings and evidence indicates that treatment o f OSA
results in a meaningful lowering o f blood pressure. OSA does not increase the risk
o f aspiration (Answer A), laryngospasm (Answer B), diabetic ketoacidosis
(Answer C), or hypoxemic respiratory failure (Answer E).

163. The answ er is B: Cystic fibrosis is the most common fatal genetic disease in
the United States. It is transmitted as an autosomal-recessive trait The incidence in
the United States is reported to be approximately 1:3,500 in whites and 1:17,000 in
African Americans. Those who arc heterozygous are unaffected. The disorder
involves exocrine glands and affects predominantly the GI and respiratory systems.
Complications include meconium ileus present at birth, chronic cough and
wheezing with copious mucous production, pancreatic insufficiency with possible
development o f insulin-dependent diabetes (up to 8%), retarded growth, infertility,
and chronic obstructive pulmonary disease. The diagnosis is made by the
pilocarpine iontophoresis sweat test. Levels o f sodium and chloride >60 mEq/L arc
usually diagnostic. Survival beyond 30 years of age is occurring more frequently.
Death usually results from pulmonary complications such as infections with S.
aureus, Pseudomonas aeruginosa, and H. influenzae.

164. The answer is C: Sarcoidosis (C) is a multisystemic granulomatous disorder


o f unknown aetiology which commonly affects adults, with an increased
prevalence in the Affo-Caribbean population compared to Caucasians. Usually
discovered during incidental findings on chest x-ray, patients with sarcoidosis (20-
40 per cent) are usually asymptomatic. Acute presentations include erythema
nodosum (painful, erythematous, raised lesions on shin fronts with/ without
arm/thigb involvement) with/without polyarthralgia. 90% o f patients with
pulmonary disease will have abnormal chest x-rays with bilateral hilar
lyraphadenopathy. Other signs on chest x-ray include pulmonary infiltrates or
fibrosis Patients may present with dry cough, progressive dyspnoea, reduced
exercise tolerance and chest pain. In some patients with pulmonary sarcoidosis
(10-20 per cent), symptoms progress leading to a decline in lung function. Some of
the non-pulmonary manifestations of sarcoidosis include lymphadcnopathy,
hepatomegaly, splenomegaly, uveitis, conjunctivitis, lacrimal and paroud gland
enlargement.
Blood tests may reveal a raised ESR, lymphopenia, deranged Ll-Ts, elevated scrum
ACE and raised immunoglobulins. Twenty-four hour urine collections may reveal
hypercalciuria. Tissue biopsy (o f lung, liver, lymph nodes, skin nodules or lacrimal
glands) is usually diagnostic, with histology revealing non-caseating granulomala.
Patients with bilateral hilar lymphadenopathy without systemic manifestations do
not require corticosteroid treatment. Acute presentations usually require bed rest,
NSAIDS and possibly corticosteroid therapy. Corticosteroid treatment is usually
indicated in patients with parenchymal lung disease, uveitis, hypcrcalcaemia,
neurological/cardiac involvement. In severe disease, intravenous corticosteroid
therapy or immune suppressants may be required.

165. The answer b E: Examination skills can be teamed by rote to u large extent
although an experienced candidate will be slick in their approach. Interpretation o f
clinical findings proves to be more of a challenge for students. In this case we urc
provided with three pieces o f clinical information:
i chest expansion unilaterally.
I breath sounds on the affected side.
1 percussion note on the affected side.
These features are all leaning towards a diagnosis o f an extensive collapse or
pneumonectomy. A lobectomy would be unlikely to provide such dramatic signs as
the remainder of the lung normally expands to fill the void left by removal o f a
single lobe. Characteristic findings of a pleural effusion would be l chest
expansion on the affected side, stony dullness to percussion with bronchial
breathing above the effusion. Other scenarios are widely available in clinical
textbooks although there is no substitute for clinical practice for compounding your
knowledge!

3 Gastroeoterology and Disorders


O f The pancreas And Liver

Liver Functions
166. The answ er is B: Gilbert's disease is a persistent, lifelong condition that
involves the deficiency of glucuronyl transferase. It afi'ccts as much as 5% of the
population. There may be a familial component. Patients exhibit a persistent
elevation in indirect (unconjugntcd) bilirubin. Stressful states and lusting may
increase bilirubin levels. Patients do not exhibit symptoms, und thete is no
evidence of hemolysis. Gilbert’s syndrome can be distinguished from h q u tin s bv
nonnaJ liver function tests, absence of urinary bile, and predominantly
uncoiyugated bilirubin fractionation. Hemolysis is differentiated by the absence of
anemia or rcticulocylosis. Liver histology is normal but biopsy is not needed for
the diagnosis. No treatment is required, and no untoward effects are noted Patients
should be reassured that they do not have liver disease.

167. The answer Li D: Prc-hepatic jaundice is caused by any form of hemolysis, or


an inborn error of bilirubin metabolism that overwhelms the liver's ability' to
conjugate bilirubin.
The most common causes of hepatic jaundice arc viral infection, toxins, drugs, and
alcohol. As hepatocytes necrose, the liver's ability to conjugate bilirubin is
impaired, and the levels of both unconjugaied and conjugated bilirubin rise in the
blood.
Post-hepatic jaundice is caused by a physical obstruedoo of conjugated bilirubin
excretion, typically a pancreatic rumor or a gzllsiooe in the common bile duct, or
more uncommonly parasitic infestation or biliary atresia.

168. The answer is E: The synthetic function of the liver is assessed either by the
dotting pathway, as vitamin K dotting factors are absorbed via the liver,
or through serum albumin levels. If the liver is not functioning property, clotting
becomes deranged and the <=*TTTmalbumin c a ken&'aaoos decrease. Serum albumin,
however, can act as an acme p fa v profess sad be reduced in many inflammatory
or infective situations. Alanine transferase sad a-kaline phosphamsc are markers of
thihrr rfairagr

169. The answer b C: Jaundice, also known as icterus, refers to yellow


pigmentation o f the skin, sriexae and tzacosa due to raised plasma bilirubin,
usually visible ax > 2 mg?dL Jaundice can be caeegoryed a x o eding to: ( I ) The site
o f the problem: pee-hepohe (e.g. haemolysis, physiological in neonates,
dyscrythropoesis, gluaconyl rr^W rr^g dcficcocy). kepebc (e.g. viral hepatitis,
CMV, Epstem-Barr virus (EBV), drug r e ared, alcoholic bepahhs, cirrhosis) and
post-hepatic (e-g. galk uw c in the common bde duct, pancreatic cancer,
cbohngioczrcinoma, primary biliary orthosis); or (2) The type of circulating
bilirubin (conjugated or uoccojugated).

170. The answer is B: Laboratory tests for hepatobiliary disease can be divided
into three general categories: (1) markers of acme bepaiocyie injury and death,
including aspartate aminotransferase (AST or SGPT), alanine aminotransferase
(ALl or S(jOI'), und alkaline phosphatase; (2) measures of hepatocyte synthetic
function, including prothrombin time and albumin; and (3) indicators of hepatocyte
catabolic activity, including direct and indirect bilirubin, and ammonia. AST is
nonspecific ns it also found in heart, smooth muscle, kidney, and brain tissues,
AM' is u more specific marker o f hepatocyte injury. Moderate elevations o f LDH
arc seen in all hepatocellular disorders and cirrhosis, but may also become
significantly elevated us a result of hemolysis. .Ammonia levels do not correlate
accurately to clinical status

171, The answer is D: The higher the clastin content of a tissue, the greater its
bilirubin content and the more intense the icteric discoloration. The innermost layer
o f conjunctiva (the subcpithelial lamina propria) and its contiguous, most
superficial aspect o f the sclera (the episclera) ore abundantly endowed with clastin
fibers. The sclera proper contains far less clastin tissue.

172. The answer b A: b - Hcpatocytcs. c - Enzyme inhibitor, d - Enzyme inducer


e - Conjugation is OK but excretion from the hepatocyte into the bile is impaired.
(Gilbert's syndrome - bilirubin can't Go in to the hepatocyte - uncoojugaled
bilinibinaemia. Crigler-Najjar syndrome - bilirubin can't Conjugate - uoconjugatcd
bilinibinaemia. Dubin-Johnson syndrome - bilirubin can't Depart from the
hepatocyte - conjugated bilinibinaemia.)

173. The answer is D: Gilbert's syndrome is inherited in autosomal dominant


fashion and affects 2-5% o f the population UDP gtucuronyl transferase levels are
reduced leading to an unconjugaicd hyperbilirubinacmia. W hilst serum bilirubin
levels are elevated the other LFTs are normal. Jaundice dee pens-after a p a io d o f
fisting or inlcrcurrent illness but bilirubin levels are reduced by enzym e inducers
such as phenobarbitooe. A s uncoojcgated bilirubin is tightly bound to albumin it
cannot cross the glomerulus and so is not found in (he urine. This contrasts with the
bOirubin-ghJCurooide-albumin complex formed in patients with cholestatic
jaundice (and raised conjugated bilirubin levels) where I S o f the complex rs
dxalysable and although most o f the bilirubin is reabsorbed in the proximal tuNde
some bilirubin is detectable in the urine.

Hepatitis

174. Answer is C: He is a carrier tbr hepatitis C Heporitis A (H A V) and hepaht^s


E (H E \1 are hepatitis viruses that do not have a chronic earner state - you cither
survive the infection or w ill die from it If someone is HAV antibody positive then
it means that they have had the infbctioc in the past but have surviv cd i: bis are act
currently infected with in
Hepatitis B (HBV), hepatitis C (HCV) and hepatitis D (H DV) can cause chronic
infection (although HDV can only occur m the presence o f ongoing HBV
infection). HBV infection is diagnosed by looking for the presence o f sjfb e e
aac^en \H3>AgV If HBsAc is positive ±en the patient has HBV infection - if this
o rrcscn: for cvre than o rr.onths the puoer.i is said to have chronic HBV infection.
HBV core antibody (H3cAb) gives an indication o f previous infection or natural
~aTi-aur> In this case, the patient has had HBV infection in the post but has
managed to clear il Individuals who are given HBV vaccination arc given a
recombinant version o f HBsAg and NOT core antibody or core antigen (HBcAb or
HBcAgV. If >ou are HBcAb positive (like this patient), then there is no benefit from
giving the paher.: additional HBV vaccination. Patients vvbo are immune from
HBV. either tunc ugh active or passive immunity, will be HBV surface antibody
(HBsAb) positive

175. The answer b E: HCV-positive patients with one long-term, steady sex
partner do not need to change their sexual practices. They should, however, discuss
the risk (which is low but not absent) with their partner. HCV-positive women do
not need to avoid pregnancy or breast-feeding. Potential, expectant, and new
parents should be advised that about 5 o f every 100 infants bom to HCV-infectcd
women become infected. This infection occurs at the time o f birth, and no
treatment has been shown to prevent the transmission. There is no evidence that the
method o f delivery is related to transmission; therefore, the need for cesarean
section versus .vaginal delivery should not be determined on the baas o f HCV-
infection status. Limited data on breast-feeding indicate that it does not transmit
HCV, although it may be prudent for HCV-positive mothers to abstain from breast­
feeding if their nipples are cracked or bleeding. Infants bom to HCV-positive
women should be tested for HCV infection and, if positive, evaluated for the
presence or development o f chronic liver disease. HCV is not spread by sneezing,
hugging, coughing, food or water, sharing eating utensils or drinking glasses, or
casual contact.

176. The answ er b A: Hepatitis is also known as infectious hepatitis, the causative
agent is an RNA virus. The onset o f clinical symptoms is usually acute, and
children and young adults are usually affected. The transmission is via a fecal-oral
route. The course o f the disease is usually mild, and the prognosis is usually
excellent- There is neither an associated chronic state nor a carrier state. The
rfiagnods is made by the detection o f elevated levels o f IgM antibodies, which
inrfirati- active disease, and IgG antibodies, which indicate previous disease. Most
cases require no special treatment other than supportive care.
Hepatitis B. This DNA viral disease is more severe than hepatitis A and causes
more complications. It is transmitted parenterally and through sexual contact
(especially in sexually active young adults and homosexuals). The symptoms are
often severe and can be devastating to elderly patients or those who are debilitated.
Approximately 10% o f cases become chronic; up to 30% o f affected patients
become carriers o f the virus after they are infected. The detection o f the hepatitis B
surface antigen supports the diagnosis o f acute illness, and values become positive
between 1 and 7 weeks before the symptoms become evident.
Hepatitis C This disease accounts for as many as o f the cases of L-
the US. It is the main indication for liver transplant in the United Stairs when
cirrhosis is present. The disease is transmitted by infected Wood said is commonly
seen in intravenous drug abusers sad those who had blood transfusiors infected
with the virus The disease is usually insidious in its presentation, and the vevenry
is variable- As many as 50% of these patients may develop chronic disease, which
may eventually lead to cirrhosis. Hepatitis E. The transmission is iimjlar to the
hepatitis A virus. The disease is found in India and Southeast Asia, Africa, ar«d
Mexico. Cases in the United Stales arc usually related to travel to tber-e c^lerr.ic
areas. Hepatitis E virus is associated with a high fatality in pregnant women

177. The answer is C: The following are specific tests iced when assenting a
patient infected with HBV:
* Hepatitis B surface antigen (Australian a n tig en T h is test detects the surface
antigen o f the HBV. It is usually detected 1 to 4 months after exposure to the vines
Its presence represents infection with the virus. In approximately 10% of cases, this
test remains positive and no antibodies are formed. This stale denotes the chronic
earner state.
* Hepatitis B antibody. This test detects the presence o f antibodies to the hepatitis B
surface antigens. It usually occurs 5 months after exposure to the virus and persists
for life. Its presence represents past infection and relative immunity to hepatitis B.
It can also be used to check for antibodies after immunization for the HBV.
* Hepatitis B core antibody IgM and IgG. Anti-hepatitis B core antibody IgM is
useful when trying to determine infection with the virus during the “Window
period’' (i.e., the time between the disappearance o f the surface antigen and the
development of the antibody). Its presence indicates a current infection with
hepatitis B. Anti-hepatitis B core antibody IgG indicates a previous hepatitis B
infection, and its presence remains indefinitely.
* Hepatitis B e antigen. The presence o f the e antigen indicates that the blood is
highly infectious. It is associated with more severe cases and the development o f
the chronic carrier state. Its persistence for longer than 8 w eeks indicates that a
chronic carrier state has developed. In 90% o f cases, hepatitis B c antigen-positive
mothers infect their fetuses.
* Hepatitis B antibody to the delta agent. Conversion from the hepatitis B e antigen
to the anti-hepatitis B e indicates a lower infcctivity rate and improvement in the
patient’s liver function status. It usually reflects a benign outcome

178. The answer is C : HCV is the mos: common chronic blood-borne infection.
Identified in 1988 through molecular biologic techniques, HCV is an enveloped
RNA virus that is classified as a separate genus in the Flaviviridae family..HCV is
roost efficiently transmitted through large or repeated percutaneous exposurcs to
blood, such as transfusions or transplants from infected donors (although the blood
supply has been screened for HCV since 1992;, inadvertent contamination of
supplies shared among patients undergoing chronic hcntodialy.L,. or tfrzrn, of

ICC
equipment among injection drug users. Transmission of HCV may also occui
through high-nsk (particularly anal) sex, perinatal exposure, percutaneous
exposures in the health care setting, or exposure to the blood of ait infected
household contact. There is no anti-HCV vaccine, and immune globulin docs not
prevent infection. There is no means to prevent uiothcr-to-child transmission
(estimated to occur 5% of the time), and breast-feeding is allowed for mothers with
chronic HCV.
The incubation period for newly acquired (acute) HCV infection ranges from 2
weeks to 6 months, averaging 6 to 7 weeks. The course of acute HCV is variable:
60% to 70% are asymptomatic, 20% to 30% have jaundice, and 10% to 20% have
nonspecific symptoms such as loss of appetite, fatigue, and abdominal pain, ALT
elevations are typically <800 IU per L and rarely exceed 1,000 1U per L. Most
patients (80%) develop chronic HCV infection, with a typical, chronic polyphasic
fluctuation in ALT between normal and 300 IU per L. No clinical features of the
acute disease or risk factors for infection, including a history o f percutaneous
exposures, have been found to be predictive of chronicity. Since viral replication
can be detected as early as 1 to 2 weeks after exposure, acute HCV is best
diagnosed with a HCV RNA polymerase chain reaction assay. Emergence of the
anti-HCV antibody is expected in 80% o f patients by 3 months and 97% by 6
months and is the recommended test to screen for chronic HCV, which is currently
recommended by the USPSTF for all patients bom between 1945 and 1965, as well
as for those with risk factors for infection. The persistence of HCV viremia beyond
6 months defines chronic infection, whereas clearance o f detectable virus indicates
either self-eradication or treatment success when measured 12 weeks after the end
of treatm ent
The current standard for combination treatment for chronic HCV (pegylated
inferferon plus ribavirin for genotype 2, 3, or 4, plus either protease inhibitor
telaprevir or boceprevir for genotype I infection) is evolving rapidly at the time o f
this writing (July 2013). New HCV polymerase inhibitors and nucleoside +
nucleotide reverse transcriptase inhibitors are expected by 2014 that have the
potential to offer all-oral (i.e., interferon-sparing) treatment for at least genotype 2
and 3 infections, and most likely genotype 1 and 4 infections as well by 2015, with
shorter treatment intervals and considerably fewer side effects and loxicities.
Recommendations on who to treat are similarly evolving rapidly on the basis o f the
patient’s HCV genotype, stage o f liver disease, and preference for treatment versus
deferral until newer treatments are available. Although several noninvasive tests
arc currently available to estimate liver disease stage (such as fibrosurc,
Hepascore, and ultrasound elastography and others), a liver biopsy remains the
gold standard to stage liver disease.

179. T he answ er is A: The immunological tests for hepatitis B can be complicated


and confusing, however they can be very helpful in determining the current
hepatitis status of the patient as well as their previous exposure. Hepatitis B is a
DNA virus that is mainly transmitted via blood products and sexual activity. -
Certain individuals, such as those who work in healthcare, are required to be
vaccinated against hepatitis B. The vaccine contains the hepatitis surface antigen
(HBsAg) against which the immune system produces hepatitis B surface antibody
(HBsAb). It is HBsAb that is measured after vaccination to evaluate the response to
the vaccine. Some individuals have a poor response to the vaccine and require
higher doses of vaccine or booster shots. HBsAb are also found in patients whe
have had hepatitis in the past In this situation the patient will also have developed
antibodies to the hepatitis B core antigen (HBeAg), which is not present in patients
who have been immunised.
Hepatitis B surface antigen (HBsAg) can usually be detected within 4 weeks of
infection and is the earliest marker of hepatitis B infection. It is not found in
patients vaccinated against hepatitis B. If HBsAg is still detected in the scrum 6
months after an acute hepatitis B infection, the patient has become a chronic earner
of the virus. The risk o f developing chronic hepatitis B infection is related to age at
the time of infection. The majority o f infected neonates develop chronic infection
whereas only 5-10% of adults do so. Hepatitis B c antigen (HBeAg) can be present
in both acute and chrome hepatitis B and indicates a high level of infcctivity. The
presence of HBeAg in chronic hepatitis B suggests that the patient is infective and
has an aggressive disease requiring treatment. In addition to establishing
infcctivity, the measurement of HBeAg can be used to assess the efficacy of
treatment with falling levels indicating success. It should be noted that there ore
some mutant strains o f the hepatitis B virus found in Asia and the Middle East that
do not produce HBeAg. Hepatitis B DNA levels can also be used to assess disease
activity with high levels indicating active disease and infcctivity. Hepatitis B core
antibodies (HBcAb) can be present in acute and chronic infection. They arc formed
against the hepatitis B core antigen (HBeAg), which is found within the liver and
therefore cannot be detected using serology. In acute infection it is the second
marker to be detected, after HBsAb, and is usually seen after 4 weeks of infection.
In chronic disease the presence o f high litres of HBcAb, in the absence o f HBeAg,
indicates low infectivity and disease activity. This serological picture is associated
with a better prognosis with fewer individuals progressing to cirrhosis and
hepatocellular carcinoma.

180. The answer is C: Autoimmune hepatitis may present with a fulminant or


subfulminant course with new onset of hepatic encephalopathy and transaminases
greater than 2000. It tends to relapse without maintenance treatment, and responds
well to oral prednisone in a vast majority of cases. However, autoimmune hepatitis
does not tend to present with a cholestatic picture as described in C, unless
associated with overlap syndromes with PBC or cholangiopalhy.

181. The answ er is B: Acetaminophen is known to cause centrilobular necrosis.


This type o f necrosis occurs immediately around the terminal hepatic vein. In
addition to acetaminophen, centrilobular necrosis can be caused by carbon
tetrachloride, broroobenzene, halothanc. and rifampin Diffuse hepatic necrosis has
also been reported with acetaminophen toxicity. Answer A is incorrect. Drugs
causing acute hepatitis include methyldopa, isoniazid, nitrofurantoin, and
phenytoin. Acetaminophen is not associated with acute hepatitis.
Answer C is incorrect. Hepatic fi brosis is commonly associated with drugs that
cause chronic hepatitis and/or hepatocellular injuiy. Some o f the culprits include
ethanol, methotrexate, and amiodarone. Acetaminophen is not associated with fi
brotic liver changes.
Answer D is incorrect. Granuloma formation does not occur with acetaminophen
overdose. Some o f the drugs associated with granuloma formation include
sulfonamides, methyldopa, quinidinc, hydralazine, and allopurinol.
Answer E is incorrect. Microvesicular fatty change does not occur with
acetaminophen overdose. It is usually seen with tetracycline, salicylates, and
ethanol use.

182. T he answer is A: The antibody can be demonstrated in 80-90% o f patients,


usually late in convalescence, and indicates relative or absolute immunity. In
contrast, HBsAg occurs very early and disappears in <6 months. Persistence o f
HBsAg indicates chronic infection. The pattern in this patient is also seen
postvaccination, and perhaps as a consequence o f remote infection.

183. The answer is D: Acetaminophen reliably produces hepatocellular damage


when taken in large doses. Daily doses should not exceed 4 g a day, and accidental
or intentional overdoses o f 10-15 g will result in liver injury. Fatal fulminant
disease is usually seen with ingestions over 25 g.

184. The answer is B: ALP is the most sensitive and specific indicator o f response
to ursodiol therapy, and various cutoff values have been used to indicate a
response. Improvement in aminotransferase levels can occur but is less predictive
o f response than the ALP level. Increases in bilirubin level and prothrombin time
indicate advanced disease and generally do not improve with ursodiol therapy.

Cirrhosis and Portal Hypertension

185. T he answer is C. The incidence o f massive lower G1 bleeding is 20% to 27%


episodes per 100,000 persons annually, with a mortality rate o f 4% to 10%. The
clinical evaluation o f GI bleeding depends on the hemodynamic status o f the
patient and the suspected source o f the bleeding. Patients presenting with upper GI
or m assive lower GI bleeding, postural hypotension, or hemodynamic instability
require inpatient stabilization and evaluation. The immediate response to
significant bleeding is to resuscitate the patient if they are unstable. Hospitalization
is also required in patients who are hemodynamically unstable or elderly and those
who have comorbidities.
GI bleeding suspected from a lower source may be secondary to diverticular #
disease, angiodysplasia, ulcerative colitis, ischem ic colitis, neoplasm , colorectal
polyps, proctitis, a/v malformations, anal and rectal neoplasia, and hemorrhoids.
Acute m assive rectal bleeding frequently arises form an upper G1 source.
Colonoscopy identifies definitive bleeding in more than 70% o f patients. A
technetium -99-tagged RBC scans m ay be helpful in identifying the site o f bleeding
i f the volume is >0.1 to 0.4 mL/min. However, positive findings in this type o f
testing must be verified with an alternative test because o f a relatively high number
o f false-positive results. Angiography may be useful in patients w ith active
bleeding >0.5 mL/min and can identify highly vascular non-bleeding lesions such
as angiodysplasia and neoplasms.

186. The answ er is B: The most common source for GJ bleeding is from a site
proximal to the ligament of Treitz (UG I bleed). Most UGI bleeds are secondary to
peptic ulcer disease. However, in patients with an established lower GI source o f
bleeding, the most common etiology is diverticular disease, followed by colitis,
adenomatous polyps, and malignancy.

187. The answ er is D: Hemorrhoidal bleeding is usually limited, with the blood
being found on the surface o f the stool, on the toilet tissue, or noted at the end o f
defecation, dripping into the toilet bowl. When patients describe the passage o f
blood clots, colonic lesions should be suspected and investigated. Although the
m ost common cause o f rectal bleeding is hemorrhoids, other, more serious causes
should be investigated^

188. The answ er is D: Ascitic fluid is tested for cell count, glucose and protein,
Gram stain, and culture to identify bacterial peritonitis. A total WBC count >
1000/ml or a neutrophil count > 250/m l is diagnostic for spontaneous bacterial
peritonitis. Low glucose or high protein values suggest infection. Gram stains and
culture results can be falsely negative 30-40% o f the time, and empiric antibiotics
should be started in the ED based on clinical suspicion. Culture sensitivity is
increased by using 10 mL o f ascitic fluid per blood culture bottle. Additional
studies o f ascites that can help with inpatient evaluation are cytology, albumin,
LDH, and tumor markers.

189. The answ er is A: Hepatomegaly describes the enlargement o f the liver. It is


detected clinically by palpating the right upper quadrant during inspiration. A s the
patient inspires the liver is displaced inferiorly by the lungs onto the exam iner's
hand. In normal individuals the liver should not be palpated with the exception o f
children and particularly thin patients. The presence o f hepatomegaly is usually
described in terms o f size, e.g. number o f finger-breadths below the costal margin.
The texture o f the liver edge should also be documented and it should be noted
whether it is smooth or craggy. Conditions that cause hepatomegaly w ith a smooth
margin include viral hepatitis, biliary tract obstruction, hepatic vein thrombosis
(Budd-Chiari syndrome), right heart failure and m yeloproliferative disease.
Hepatomegaly with a craggy border is usually associated with hepatic metastdtic

nn
ditcave and p c A y c r j - A i c divcar-e End-stage cirrhosis d o c s not result in hepatomegaly.
ir'er damage often causes hepatomegaly and fairy infiltration, but at, the
damage progresses and fibrosis ensues, replacing the normal tissue architecture, the
liver becomes small and scarred.

190. The answer U E: Sclerosing cholangitis is a condition causing inflammation,


fibrosis and subsequent stricture formation of the bile ducts, leading to cholestasis
and eventual cinhosis. These changes occur in the intra- and cxtrahcpatic bile
ducts. There is a strong association with inflammatory bowel disease, most
commonly ulcerative colitis, with about 80% of patients with sclerosing cholangitis
having coexistent disease (note that although only -5% o f patients with UC have
concomitant sclerosing cholangitis). These patients present with the effects o f
jaundice. Liver biopsy is diagnostic. Treatment is primarily symptomatic.
Ursodeoxycholic acid may delay disease progression and liver transplant is curative
in some cases. Purely extrahepadc disease may be surgically treated.

191. The answer is C: PBC is a rare condition predominantly affecting women in


their 50s. The aetiology is thought to be autoimmune. There is chronic
granulomatous inflammation o f the interlobular bfle ducts causing cholestasis.
Presentation is through the effects o f cholestasis - jaaocfice, pruritts, hepatomegaly
and pain, b may also be picked up early by deranged liver function tests in the
asymptomatic patient. Diagnosis is made by liver biopsy, and anrimitochoadrial
anriberncs may be presenL Management is threefold 1) symptomatic to reduce
rtchrerrw and prevent osteoporosis, 2) specific m ediations to improve absorption
aad 5r.gr j 5) disease-modifying drugs such as onodeorychofcc a d d (which
reduces ccoiestasisjL Recent evidence however q gg-g* that orsodeoxychobc acid
does atz sa p e v s ooracry. Lxrsr sizt be m o p t i o n in some candidates
•at±. ead-fiagy disease

192. The a— er m Er Parjerts t t c s c k are a ride o f derndepmg S5P. which

zsxzZy vcr&TTs anerr generalised pair., v m h g aacics,


vygreir^ lever 2ad tigocL. The most sccescb causative organisms a t Grsst>
aga.~»t b ac-j m ca E sch o cca co5 and JCichsaeila spp., whka ester the
7 »r g r ^ cremation h o a the ^rsegnai hraer. and eoloebe the radac fluid SB ?
ease lead 10 rapid ■•Varr.jirmginB o f Irver civaese caanrg hepasic eacepfcalopadjv
ac*d oeath. The diagnosis c f SB? is cocfem cd by pzracenteses, which involves
t&ir og a «agEpte of ascitic fluid horn the abdomen gang a needle. The aspirated
ascitic fluid is analysed for while cell count, glucose and protein. In addition, the
fluid should be sect to microbiology for culture and Gram staining. If the white cell
count is above 250 ceils/mm} the patten! requires intravenous antibiotics (e g
cefotaxime or ceftriaxone;. Some patients also require human albumin solution to
restore their intravascular fluid volume. Patients who have had a previous episode
of SBP, and patients who are considered to be at high risk o f developing SBP,
should be considered for prophylactic oral antibiotics (e.g. norfloxacin or
ciprofloxacin) The development or spontaneous nocicnai perrorotu correr,.por<i; :c
a poor long-term prognosis.

193. 'file answ er Is B: Dupuyiren c o n t r a c t u r e is a progressive fibroplasia o f the


palmar facia that results in a flexion contracture o f the fingers, ft is m ost common
in the ring and little fingers Risk factors include the male sex, a family history,
diabetes mcllifus, alcoholic cirrhosis, phenytoin use, trauma, acquired
immunodeficiency disease syndrome (AIDS), Peyronie disease (idiopathic fibrosis
of the corpus cavemosum) and Ledderhosc disease (fibrosis o f the plantar fascia,
resulting in a similar deformity) On examination the thickened palmar aponeurosis
can be felt. Treatment is by excision of the thickened part o f the aponeurosis.
Trigger finger is characterised by difficulty in extension o f a finger caused by a
disparity in size between the flexor tendon sheath and the tendon within i t The
finger remains flexed until passively extended with a “click” . There is no
association with alcohol excess.
Asterixis describes a jerking flexion-extension movement of the hand seen when
the arms are placed in an outstretched position with the wrists cocked back. It is
associated with hepatic encephalopathy and should be tested for in all patients with
liver disease who present with an acute deterioration o f disease, w orsening ascites,
confusion, agitation, reversal o f sleeping partem and stupor.

200. T he answ er is C: Kayser-Fleischer rings 2 re seen in W ilson’s disease, in


which copper is deposited in vital rrrv^r^ due to an error in m etabolism , which
n»«n< that h cannot be secreted into die bile. Kayser-Fleischer r in ^ are due Co the
deposition o f copper in Descemct’s membrane, which is located between the
coraeal stroma 2 nd endothelium in the eye. They appear 2 3 brown-green rings
around the in s and are seen in up to c i patients with W ilson's disease In most
paring** they can cm y be seen using a sht-Lssp exam ;rar.ee. ahhncgh they may be
visible with the caked eye in pariem whh light Hue eyes aod in pBticxtts wish
advanced disease.

201. T he answer is B: From the a c r a a s above, Bobd-C heri syndrome <Bj


(winch presents with a triad o f symptoms: acme abdominal pern, hepatom egaly and
ascites), is the most likely cause of hepatomegaly The condm oa resorts ham
bcpeeic vein omficrw obstruction o f which: • There is no known carry in 50% of
suffererr, • o f the remaining of patients rHagnrtsrri with B odd-O kac
syndrome, 75% o f these are cue to thrombosis o f the hepabc vein (primary 3odd-
Chiari syndrome) and 25% are due to external compression o f the hepatic vein
(secondary Budd-Chiari syndrome).
The causes o f hepatomegaly can be classified according 10:
1 Maligancy. primary* (e.g. HCC) or secondary
2 Hepatic congestion secondary to: right heart failure, Budd-Chiari syndrome.
3 Infection: hepatitis (secondary to viruses, malaria, shistosomiasis, amoebic
abscess, hydatid cyst), infectious mononucleosis.
4 Haemaiolcgical: leukaemia, lymphoma, myeloproliferative disorders, suqh ,as
myelofibrosis, sickle-cell disease, haemolytic anaemias.
5 Anatomical: Riedel’s lobe.
6 Other causes include early cirrhosis, fatty liver, porphyria, amyloidosis,
Gaucher’s disease.

202. The answ er is D: Oesophageal varices arise as a result o f portal hypertension


(>10 mmHg) which leads to dilated collateral veins at sites of portosystemic
anastomosis (e.g. the lower oesophagus). The causes o f portal hypertension can be
divided into: (1) Pre-hepatic: portal-vein thrombosis, splenic vein thrombosis, (2)
Hepatic: cirrhosis (accounts for 80 per cent o f causes o f portal hypertension),
shistosomiasis (most common cause worldwide), sarcoidosis, myeloproliferative
disease, congenital hepatic fibrosis; and (3) Post-hepatic: Budd-Chiari syndrome,
right heart failure, constrictive pericarditis, veno-occlusivc disease. Once portal
pressures are >12 mmHg, variceal bleeding may develop. Prophylaxis for the
prevention o f variceal bleeding can be divided into: (1) Primary" non-sclective (T
blockade (e.g. propranolol) and/or endoscopic banding ligation; (2) Secondary (i.e.
after an initial variceal bleed: non-selcctivc ^-blockade, endoscopic banding
ligation, transjugular intrahepatic portosystemic shunting (TIPPS) for varices
resistant to banding or surgical shunts if TIPPS is not possible.

203. The answ er is D: Finger clubbing (A), palmer erythema (B), spider naevi (C)
and jaundice (E) are all known clinical signs o f chronic liver disease. Others
include bruising and liver flap (secondary to hepatic encephalopathy). Koilonychia
(D) refers to spooning o f the nails and occurs in iron deficiency anaemia. It is
leuconychia (whitening o f the nails due to hypoalbuminacmia which can occur due
to chronic liver disease, nephrotic syndrome, malnutrition) that is seen in chronic
liver disease.

204. The answ er is A: This patient is suffering from primary biliary cirrhosis
(PBC) (A) which is characterized by chronic granulomatous inflammation leading
to damage of interlobular bile ducts. This chronic inflammatory process leads to
cholestasis, cirrhosis and portal hypertension. The cause o f PBC is thought to be of
autoimmune origin (women being more affected than men) and is associated with
various autoimmune conditions. Patients are often asymptomatic and diagnosis is
usually made when abnormal LFTs are detected with an abnormal rise in serum
ALP. Symptoms include lethargy and pruritus which can occur before the
presentation o f jaundice. Signs include jaundice, xanthelasma, xanthomata, skin
pigmentation, splenomegaly and hepatomegaly. Investigations include blood tests:
(1) L Frs (raised ALP, y-GT, with mildly elevated AST and ALT. In late disease
there is a raised bilirubin level and low levels o f albumin with an increase in the
prothrombin time); (2) Ninety-eight per cent of patients with PBC are anti-
mitochondrial antibody (AMA) positive. ANA, SMA and ANCA autoantibodies
may also be present but at low titres; (3) JgM (usually raised); (4) Raised levels of
~r
rSH an<J cholesterol may be present. Performing radiological imaging, such as
ultrasound, will exclude extrahepatic cholestasis and liver biopsy will confirm
granulomas (not specific to PBC) surrounding the bile ducts, progressing to
cirrhosis.
Treatment is divided into
• symptomatic - for symptoms of pruritus (e.g. colestyramine), diarrhoea (e g.
codeine phosphate) and osteoporosis (e.g. bisphosphonates);
• specific - vitamin A, D. K supplementation, ursodeoxycholic acid for improving
jaundice and ascites;
• liver transplantation for end-stage liver disease.
Wilson’s disease (B) is an autosomal recessive disorder (of gene on chromosome
L3 that codes for copper transporting ATPase) resulting in toxic accumulation of
copper in the liver and central nervous system. Twenty-four hour urinary copper
excretion is high (>100 pg/24 h with normal levels being <40pg) with low copper
and caeruloplasmin levels. Diagnosis can be confirmed by genetic testing and liver
biopsy. Patients present with signs of liver disease. Kayser-Fleischer rings are
pathognomonic (copper deposits in iris). Hereditary haemochromatosis (C) is an
inherited condition characterized by a disorder in iron metabolism. There is
increased intestinal iron absorption which is deposited in multiple organs such as
the liver, heart, pancreas, etc. LFTS are usually elevated with raised ferritin and
serum iron levels, and low total iron-binding capacity (TTBC). Diagnosis can be
confirmed with HFE (mutation in HFE gene is responsible for hereditary
haemochromatosis) genotypmg and liver biopsy. Patients are initially
asymptomatic then eventually experience arthralgia and tiredness with slate-grey
skin pigmentation with late disease progression. Iron deposition also occurs in the
pancreas leading to impaired insulin secretion and eventually diabetes (also known
as bronze diabetes). Primary sclerosing cholangitis (D) is a condition o f unknown
cause which is defined by nonmalignant, non-bacterial inflammation, fibrosis and
Strictures o f the Lntra and extra-hepatic bile ducts. It is serum A M A negative but
ANA, SMA and ANCA may be positive. Diagnosis is made with M RCP and liver
biopsy. Patients are initially asymptomatic and may present with jaundice, pruritus,
abdominal pain and fatigue. Alcoholic liver disease (E) is incorrect here as there is
mo mention, in the clinical scenario, of raised AST/ALT levels which would signify
hepatocellular damage.

205. T he answer is A: The hepatopuimonary syndrome is characterized by


platypoea (difficulty breathing when going from lying to sitting) and orthodcoxia
{decreased oxygen saturation upon sitting). It is associated with lefl-lo-right
shunting of blood. Alpha-1 antitrypsin deficiency and primary biliary cirrhosis are
associated with the parenchymal lung diseases mentioned in B and D.

206. The answer is D: Retroperitoneal, between the ovarian vessels and the renal
-veins. Retroperitoneal varices occur between the mesenteric vessels and renal (or
gonadal, or iliac) veins However, in patients with infrarenai inferior vena.cava
obstruction, the ovarian (gonadal) vessels act as a collateral drainage for the
system ic calculation by facilitating drainage form the internal iliac v e s s e ls to the
left renal vein and the cava itself. However, this does not involve a portosystem ic
anastom osis. Other sites o f collateralisation (between the portal and system ic
circulations) include:
• The iccta n , w here the superior rectal from inferior mesenteric vein anastom oses
w ith the m iddle and inferior rectal veins/pudendal vein.
• The paraumbilical region, where the left portal/paraumbilical vein anastom oses
wish tbe vestigial umbilical vein/superficia\ epigastric and caput medhisa may
result.
• The distal esophagus, where the left gastric vein anastomoses with oesophageal
branches c f die azygous vein.
• inerahepsiic. between the portal vein and the inferior vena cava.

297. The answer b C: Melena (from die Greek melaina, meaning “black**) or
black twry stools occurs from approximately 150-200 mL o f blood in th e G1 tract
M fifn c is present in approximately 70% o f patients with upper GI bleed and 33%
o f pwbute with low er GI bleed. Black nontany stool may result from as little as 60
m l. o f Mood. Blood from the jejunum or duodenum m ust remain in the G l tract for
at least 8 hours before it turns black. Stool may remain black and tarry fo r several
days after bleeding has stopped.

299. The answer b A: Lactulose is a poorly absorbed sugar m etabolized by


co k n c bacteria yielding lactic acid. The salutary effects o f this agent axe related
both id the acidification o f the fecal stream, resulting in the trapping o f am m onia as
a n o o r a in the stool, and to its cathartic action. The usual dosage o f lactulose is
15—30 mL orally 3 or 4 times daily or in a quantity sufficient to result in several
loose bowel m ovem ents daily. The principal adverse effect is excessive diarrhea,
w ith —gitewr fluid ana electrolyte imbalance.

299. The answ er b B: Hepatocellular failure is associated with hyperdynamic


c a r a b tim and system ic vasodilatation, with increased vascular capacitance. Most
sodium and water retention.

219. The a s w e r b B: She has clinical evidence o f chronic liver disease and
ponaftiyperiengaon. The 2 mainconditions causing pigmentation and chnonic liver
< f a e » are primary bthary cirrhosis (PBC) and haemochromatosis. PB C is a
chrooir d w tan ric inflammatory liver disease, the aetiology o f which is probably
■ m m b c . It m ost commonly affects middle-aged women.There is jaundice with
sfcm [ ig w r m io o , risk o f developing oesophageal varices and fat malatbsorption,
leading id deficiency o f the vitam ins A, D, E, K (henceosteom alacia andalso
braising). Serum anrimitochoodrial antibody is positive in 95-99% cases.
211. The answer is A: The most significant abnormality is the low Fibfinbgen.
Therefore the best product to correct the fibrinogen out o f those given is
cryoprecipitate. To correct a coagulopathy you need to aim for Fibrinogen > 1.Og/1,
Platelets >50 x 109/L, PT and APTT < 1.5 upper range o f normal- so from the
results you can see the most significant abnormality is the low fibrinogen, the
platelets are low and APTT/PT prolonged but not really sufficient to cause
bleeding.

212. T he answer is E: Carnitine is the treatment o f choice for valproate


hepatotoxicity with features o f hepatic encephalopathy. Carnitine deficiency may
be one explanation for the hepatotoxicity and hyperammonemia related to valproate
toxicity. Carnitine may attenuate the effect o f valproate hepatotoxicity.

213. T he answer is E: NASH and ASH are indistinguishable histologically.


History and clinical examination differentiate the 2 diagnoses.

H epatocelluar carcinoma

214. T he answer is C: Chronic hepatitis B and C infections are associated with a


significantly increased risk o f hepatocellular carcinoma. Hepatocellular carcinoma
is also an important consequence of liver cirrhosis and should be suspected in any
patient with rapid development o f weight loss on a background o f cirrhosis. Within
the general population the prevalence is four per 100 000 people. Investigations
include serum alpha-fetoprotein, followed by either liver biopsy or computed
tomography (CT) imaging. Resection is possible for early-stage cancers. The
prognosis is extremely poor for non-resectable tumours, and there is a 20% 5-year
survival rate for operable tumours.

215. T he answer is A: 50 to 80% o f hepatocellular carcinomas are associated with


high serum levels o f a-fetoprotein (AFP), a tumour marker, which is also linked to,
and elevated in, testicular carcinomas. Serum levels o f AFP can be monitored
either post-surgical resection (if the tumour is solitary) or post chemotherapy;
falling or rising levels post treatment could be indicative o f disease remission or
progression, respectively. CEA (B) is primarily linked with colorectal carcinoma.
CA 15-3 (C) is linked with breast carcinoma. HcG (D) and CA 125 (E) are usually
associated with ovarian carcinoma.

216. T h e answer is D: Chronic hepatitis B (A ) and C (C) infections, liver cirrhosis


(B ) and aflatoxin (E) (a carcinogen from the mould Aspergillus flavus) are all
known predisposing factors for developing hepatocellular carcinoma. Chronic
inflammatory changes results in hepatocyte damage and mutation in the cellular
reparation machinery. Hepatitis A (D) does not usually lead to chronic infection
and thus is not deemed to be a predisposing factor to hepatocellular carcinoma.
Esophagus and Stomach Disorders

217. A nsw er is C: (Prostaglandins)

218. A nsw er is B: Disruption of the function of a mismatch repair gene, that, ui


turn, leads to the development o f gastric cancer

219. A nsw er is D: Administration of secretin to a healthy subject

220. A nsw er is C: Manometric findings are reasonably specific for mechanical


obstruction even when radiographic studies arc negative. Patients who haw a
specific diagnosis that is associated with dclaw d gastric emptying, such as
scleroderma, amyloidosis, or diabetes mellitus, do not need an additional diagnostic
test Rumination is diagnosed on the basis of the history; manometric findings are
less sensitive and specific. Gastroduodenal manometry is invasive, not always well
tolerated, and unlikely to be helpful in a setring in which other assessments of
gastrointestinal function (eg, scintigraphy) are normal.

221. A nsw er is C: High-risk medical groups for prophylaxis for stress ulcer are
patients with respiratory failure that requires ventilation for more than 48 hours and
coagulopathy (INR >1.5, platelet count <50,000/mm3 (50*109LJ before the
administration o f anticoagularion medication).

223. The answ er is C: Vitamin B12 (cobalamin) deficiency' is a common cause of


macrocytic anemia and has been implicated in a host of neuropsychiatric
conditions. The widespread use o f gastric acid-blocking agents, which can lead to
decreased vitamin B12 levels, may contribute to the development of vitamin B12
deficiency. Given the widespread use of these agents. Vitamin B12 deficiency is
associated with hematologic, neurologic, and psychiatric symptoms. Neurologic
manifestations from vitamin B12 deficiency include paresthesias, peripheral
neuropathy, and demyelination o f the corticospinal tract and dorsal columns
(subacute combined systems disease). Vitamin B 12 deficiency also has been linked
to psychiatric disorders, including impaired memory, irritability, depression,
dementia, and, rarely, psychosis. Dietary sources o f vitamin B12 arc primarily
meats and dairy products. In a typical Western diet, a person obtains approximately
5 to 15 pg o f vitamin B12 daily, which is far greater than the recommended daily
allowance o f 2 pg. Normally, individuals maintain a large vitamin B12 reserve,
which can last 2 to 5 years even in the presence o f severe malabsorption. However,
nutritional deficiency can occur in specific populations. Elderly patients and
chronic alcoholics are at especially high risk. The dietary restrictions o f strict
vegans make them another, less common at-risk population. The role of BI2
deficiency in hyperhomocysteinemia and the promotion of atherosclerosis arc
under investigation. Diagnosis o f vitamin BI2 deficiency is based on measurement
of serum vitamin B12 levels; however, about half o f patients v/ilfi subclinicaJ
disease have normal B I2 levels A more sensitive method of screening Jbr'viiamm
B I2 deficiency is measurement of serum methylmalonic acid and homocysteine
levels, which arc increased early in vitamin B I2 deficiency. Use of the Schilling
test for detection o f pernicious anemia has been replaced for the most part by
serologic testing for parietal cell and intrinsic factor antibodies. Contrary to
prevailing medical practice, supplementation with oral vitamin B12 is a safe and
effective treatment for the B I2 deficiency state. Even when intrinsic factor is not
present to aid in the absorption of vitamin B12 (pernicious anemia) or m other
diseases that affect the usual absorption sites in the terminal ileum, oral therapy
remains effective.

224. T he answer is A: Helicobacter pylori is a bacteria found in the stomach that


is present in >80% o f patients with duodenal ulcers and up to 60% o f those with
gastric ulcers. The incidence a p p e a r s to increase with increasing age. Most H
pylori colonization is asymptomatic. Test sensitivity is reduced if the patient is
taking Proton Pump Inhibitors (PPIs), bismuth or antibiotics. Tests include urea
breath testing. The patient ingests a urea solution with a carbon isotope and then
breathes into a container, in the presence o f H. pylori, urease hydrolyzes the urea to
release labeled C 02, which can be detected by a mass spectrometer. A stool
antigen enzyme immunoassay is reliable in confirming successful treatment, but
should not be used to test for eradication o f H. pylori until at least 4 weeks after
completion of therapy. Serology antibody tests for H. pylori are useful in ruling out
the diagnosis, but they lack specificity and arc not reliable (because o f persisting
antibodies) for documenting eradication. The gold standard for diagnosis is biopsy
and histologic examination.
Eradication rates with commonly prescribed triple therapy regim ens have fallen
below 80%. Quadruple therapy and sequential therapy have higher rates o f
eradication. Quadruple therapy should be considered for initial treatment in areas
with a high rate (>20% ) o f clarithromycin-resistant H. pylori. Triple therapy
includes clarithromyscin, plus amoxicillin or mctronidiazole, plus a PPI. Quadruple
treatment includes bismuth subsalicylate (PeptoBismol), plus mctronidiazole, plus
tetracycline plus a PPL Patients should be tested for successful eradication o f H.
pylori and those still infected after treatment with two different regimens should
receive salvage therapy with a different regimen, such as a PPL amoxicillin, anu
levofloxacin (Levaquin and others), if needed.

225. T he answer is E: Barrett's esophagus is the result o f chrouic


gastroesophageal reflux. The condition causes metaplasia and transformation o f
squamous to columnar epithelium in the areas affected. Patients usually report
symptoms o f pyrosis and, occasionally, dysphagia if strictures develop. Men are
more commonly affected than women. The diagnosis is made w ith csopbagoscopy
and biopsy of suspected areas. Treatment is accomplished w ith H2-blockers and
PPIs. PPIs strongly inhibit gastric ac:4 secretion. They act by irreversibly inhibiting
the H+-K+ adenosine triphosphatase pump o f the parietal cell. By blocking the
final common pathway o f gastric acid secretion, the PPls provide a greater degree
and duration o f gastric acid suppression compared with H2*receptor blockers. The
PPls are fairly well tolerated. The most common side effects are nuusco, diarrhea,
constipation, headache, and skin rash. PPls are more expensive than standard-dose
H2-receptor blockers or prokinetic agents. However, when prescribed appropriately
to patients with severe symptoms or refractory disease, the PPls are more cost-
effective because of their higher healing and remission rates and the consequent
prevention o f complications. Occasionally, severe cases of Barrett's esophagitis are
treated with surgery Because o f a 10% increased risk for the development of
adenocarcinoma in ibe atYected areas, follow-up with endoscopy every 3 to 5 years
is indicated, although screening endoscopy time frames are controversial.
Treatment o f gastroesophageal reflux disease associated with B arren's esophagus
has not been shown to eliminate the metaplasia o f that condition or the risk o f
malignancy . Consequently, patients with Barrett's esophagus require periodic
endoscopic biopsy to assess esophageal tissue for malignant changes.

226. T he answ er is B; Histamine recertor antagonists inhibit the H2 receptors o f


the gastric parietal cells, thereby suppressing acid secretion.
■Antacids buffer gastric acid, and are mainly used as needed for breakthrough ulcer
pain in patients taking concurrent proton pump inhibitor or If2 blockers until
healing occurs.
Misoprostol is a prostaglandin analog that increases mucous bicarbonate
production by increasing mucosal blood flow. Misoprostol may prevent ulcer
formation in patients on concurrent NSAID therapy.
Proton pum p inhibitors decrease acid by blocking its secretion at the proton pump
on gastric parietal cells. When compared with H2 blockers, proton pump inhibitors
heal ulcers faster.
Sucralfate protects ulcer from acid exposure by forming a sticky gel to allow
healing.

227. T he an sw er is D: This pattern o f symptoms most closely represents achalasia,


which is a dysmotility disorder o f the distal oesophagus. In achalasia there is
difficulty in swallowing both liquids and solids. A barium swallow is often
diagnostic and will show a narrowed distal oesophagus with dilatation of the
segment proximal to this (a bird's beak appearance) and a lack o f pcristalis in this
area. Dysphagia lasting m ore than 3 weeks, however, always warrants an
endoscopy to exclude a malignant stricture.

228. T he answ er is D: Aphthous mouth ulcers are a common idiopathic beoign


condition affecting up to 20% o f the population. D ie incidence rises dramatically in
the presence o f inflammatory bowel disease. Although very painful, recurrent
aphthous ulcers in the absence o f gastrointestinal symptoms in a young person arc
likely to be benign. In recurrent and severe cases it may be appropriate to do blood
investigations including scrum B12 and folate. In cases t>f a single prolonged ulcer

1 4 A
o f greater than 3 w eeks' duration it is prudent to take a biopsy»to.exclude a
carcinoma. Treatment is symptomatic with topical analgesic agents.

229. T he answer is B: The most common causative agent that gives rise to
approximately 90% o f duodenal ulcers is infcctioo with H. pylon (B) (a helical-
shaped gram-negative microaerophiiic bacterium). The bacteria favours low pH
environments and, with the help of its flagella, moves to the epithelial lining o f the
stomach and duodenum. The bacterium produces ammonia and proteases w hich
break down the epithelial linings of the stomach and duodenal mucosa causing
ulceration. Non-steroidal ami-inflammatory drugs (NSAIDs) (A), alcohol abuse
(C), chrooic corticosteroid therapy (D) and Zollmger-Ellison syndrome (E)
(increase in gastrin production, from e.g. a gastrinoma, which stimulates the
parietal cells of the stomach to produce excess hydrochloric acid leading to peptic
ulceratioo) arc all less common causes o f peptic ulceration.

230. T he answer is D: All the above investigations have been shown to be useful
in the diagnosis o f a hiatus hernia. However, upper G1 barium meaJs/swallows (D)
have been shown to be the most definitive modality in diagnosing hiatus hernias.
Chest x-rays (B) may be normal, but in some cases may show an air fluid level
above the level o f the left hemi-diaphragra. Upper G1 endoscopy (C) is com m only
used to assess symptoms of dyspepsia and has not been shown to be as sensitive a s
barium studies in the detection of hiatus hernias. In the UK, C T scanning (A ) is not
routinely used for the investigation o f hiatus hernias, but the latter are incidentally
detected on scanning o f the abdomen for the investigation o f other pathology.
Compared to the barium study. CT scanning delivers relatively high levels o f
radiation. Positive results obtained w ith ultrasound scanning (E) may lead to
inconsistent and lalse-po&itive/oegative results due to the operatonissociatcd
variability regarding technical experience.

231. T he answer is C : H. pylori resides in the mucus layer o f the stomach, where
it exerts its urease activity. It does not invade the epithelium. 'Hie production o f
ammonia, the stimulation o f acid secretion, and the disruption to the protective
mucus layer arc three mechanisms by which H. pylori prom otes injury. H. pylon
also stimulates interleukin-8, a cytokine associated with inflammation. H. pylori is
causally associated with gastric adenocarcinoma and M ALT lymphoma o f the
stomach. Eradication o f H. pylori has been shown to cause regression o f M A LT
lymphoma, but it has not been shown to prevent adenocarcinoma o f the stom ach.
Three- or fourdrug regimens are superior to two-drug regimens.

232. T he answer is D: Infection usually occurs early in life and is related tp classic
socioeconomic indicators such as poverty, domestic crowding, unsanitary living
conditions, and unclean water. It is much more common in developing countries.
233. T he answer is A: Being of female sex affords a certain amount o f resistance
to the development o f duodenal ulceration and this is markedly seen during
pregnancy. Steroid use has been shown to predispose to the formation o f ulcers by
mechanisms that involve both a disturbance in acid regulation and secretion and
interference with the normal protective mucosal layer o f the stomach Cushing’s
ulcer i s the eponymous name given to the dev elopment o f acute peptic ulceration
secondary to head injury, major surgery or acutely stressful conditions, and a
strong association between ulcer formation and severe bums has also been
described (Curling’s ulcer). These findings form the basis for the routine use o f
proton pump inhibitors such as omeprazole as part o f the gastrointestinal
prophylaxis of the critical care bundle in situations of acute stress.

234. The answer is E: Answers A -D are all termed obstructive causes ol


dysphagia. The causes can be categorized into:
Obstructive
• Oesophageal carcinoma
• Peptic strictures (D)
• Oesophageal web/ring (B)
• Gastric carcinoma
• Pharyngeal carcinoma (A)
• Extrinsic pressure from, for example, lung carcinoma, retrosternal goitre (C)
Oesophageal motility disorders
• Achalasia (E)
• Systemic sclerosis
• Stroke
• Myasthenia gravis
• Neurological degenerative conditions, e.g. motor neurone disease, Parkinson’s
disease
Others
• Oesophagitis
• Pharyngeal pouch
• Oesophageal candidiasis

235. T h e answer is D: Laparoscopic fundoplicationis the treatment of choice for


patients with GORD refractory to or intolerant of, Proton Pump Inhibitor therapy.
The patient should have had an endoscopy at least 6 months prior to surgery to
exclude any unsuspected pathology - Barrett’s oesophagus or adenocarcinoma. An
oesophageal transit study is indicated to rule out a primary motor disorder (eg
achalasia, scleroderma) when suspected and to rule out aperislalsis, which may
result in post-operative dysphagia after some forms of fnndoplication.

Inflammatory Bowel Diseases


236. The an sw er is E: Crohn’s disease is characterized • by a transmural
inflammation o f the GI tract, ft may affect any part o f the Gl tract but is usually
associated with the terminal ileum, the colon, or both On colonoscopy, areas of
ulceration and submucosal thickening give the bowel a cobblestone appearance,
with some skipped areas of normal bowel, fn addition to the transmural
inflammation, there are granulomas, abscesses, fissures, and fistula formation.
Symptoms include fever, weight loss, abdominal pain, dianhea, and growth
retardation in children. In children, Crohn’s disease is more com m on than
ulcerative colitis. Complications include intestinal obstruction, toxic megacolon,
which is usually more common in ulcerative colitis; malabsorption; intestinal
perforation; fistula formation; and development of gall and kidney stones. There is
also an increased risk —Gve times the average—for bowel cancer. Other areas may
be affected, including the following:
• Joints: arthritis, ankylosing spondylitis
• Skin: erythema nodosum, aphthous ulcers, pyoderma gangrenosum
• Eyes: episcleritis, iritis, uveitis
• Liver: fatty liver, pericholangitis
The diagnosis is usually made with colonoscopy or flexible sigmoidoscopy with
biopsy or with x*ray contrast studies. Treatment involves the use o f oral
corticosteroids or steroid enemas, ciprofloxacin, metronidazole, antidiarrhcal
agents, and sulfasalazine, olsalazine, or mesalamine, all three of which contain 5-
aminosalicylic acid. Infliximab is a drug that has been approved for the treatment
o f Crohn’s disease. The drug is a potent antibody to tumor necrosis factor that is
elevated in patients with Crohn’s disease and can help close fistulas in up to 60%
o f patients. In severe cases, total parenteral nutrition m ay be necessary w ith surgery
to remove the ulcerated bowel. Adalimumab is also used.

237. The an$>ver is D: Ulcerative colitis is characterized by inflam m ation o f the


bowel that is limited to the mucosa] surface and subm ucosa o f the bow el wall (i.e_
it is not transmural like Crohn’s disease). The area o f involvement is localized to
the colon and rectosigmoid area in a continuous fashion; this is unlike Crohn’s
disease, which shows skipped areas o f involvement. Symptoms in clu d e bloody
diarrhea, abdominal pain, fever, and tenesmus. Complications include intestinal
perforation, development of toxic megacolon, and development of cancer (which is
more commonly seen in patients with ulcerative colitis than in those w ith Crohn’s
disease). Extracolonic involvement affects the skin, eyes, joints, and liver,
however, the kidneys are not involved (as they are in C rohn’s disease). D iagnosis is
accomplished in the same manner as in Crohn’s disease (i.e., colonoscopy or
flexible sigmoidoscopy with biopsy or with x-ray contrast studies). Treatm ent of
ulcerative colitis is similar to that for Crohn’s disease; however, the oral form s of
5-aminosalicylic acid (eg., sulfasalazine, olsalazine, mesalamine) are more
effective in controlling recurrences and the severity o f outbreaks in ulcerative
colitis. Close follow-up is necessary for ulcerative colitis and C ro h n ’s disease
because o f the increased risk o f developing bowel cancer.

H 14.
J
238. The answer is B: The clinical pictures of inflammatory bowel disease can
often be very similar - classically a young woman with diarrhoea, pain and weight
loss Crohn’s disease that is only located in the distal colon can appear
indistinguishable from ulcerative colitis (UC) except on histology The
inflammation in Crohn’s disease affects the entire thickness of the bowel wall
which is why it more often leads to complications such as fistulas and abscesses
whereas in UC the inflammation primarily affects the nuieosa and submucosa
leading to ulcer formation and crypt abscesses Between ulcers, pseudopolyps can
form, which tend to be very' friable and easy to bleed. Differentiation in tins case
from gastrointestinal tuberculosis is difficult, but TB is more likely to produce
caseating granulomas. However, in regions endemic with tuberculosis or in high-
risk patients, a thorough history and high index of suspicion is required, even
without the presence o f caseating granulomas, and appropriate staining should be
undertaken for histological specimens. Irritable bowel syndrome does not cause
histological changes.

239. The answ er is B: Inflammatory' bowel disease (e.g. ulcerative colitis and
Crohn’s disease) is a known gastroenterological cause o f finger clubbing along
with liver cirrhosis, primary biliary cirrhosis, oesophageal Iciyomyoma, coeliac
disease and achalasia Therefore, (B) is the most likely answer here.

240. The answer is D: 80 to 100% of patients with PSC will have ulcerative colitis
(D). On the other hand, 3% of patients with ulcerative colitis will have PSC.
Thyroid disease (A), systemic sclerosis (B) and rheumatoid arthritis (C) are
associated with primary biliary cirrhosis.

241. The answer is D: The most likely diagnosis is ulcerative colitis (UC) (D)
based on the histological results of the rectal biopsy. The findings o f inflammatory
infiltrates coupled with mucosal ulcers, goblet cell depletion and crypt abscesses
are highly suggestive o f a diagnosis o f UC. UC is described as a relapsing and
remitting inflammatory bowel disorder o f the colonic mucosa. The condition
usually starts at the rectum (proctitis in 50%) and spreads proximally, in a
continuous fashion, to affect parts of the colon (e.g. left-sided colitis in 30%) or the
entire colonic tract (pancolitis in 20%). UC tends not to spread beyond the
ileocaecal valve but may cause a condition called ‘backwash ileitis’. Histologically,
Crohn’s disease (A) is characterized by transmural, non-caseating granulomatous
inflammation, coupled with fissuring ulcers, lymphoid aggregates and neutrophil
infiltrates. Crohn’s disease can affect any part o f the GI tract from the mouth to the
anus (but favours the terminal ileum in 50%) and is also characterized by skip
lesions (unaffected bowel between areas o f active disease) whereas in UC, disease
spreads from the rectum to the ileocaecal valve in a continuous fashion depending
on the stage of disease. Pseudomembranous colitis (PC) (B) is characterized by the
formation of an adherent inflammatory membrane (the pseudomembrane)
overlying sites of muscosal injury within the colon. The histology of PC is
described as small surface erosions of the superficial colonic crypts coupled wiih
overlying accumulation of neutrophils, fibrin, mucus and necrotic epithelial cells
forming a ‘summit lesion’. l"hc toxins (toxin A and B) produced by the gram­
positive anaerobic bacillus, Clostridum difficile, are meant to be the cause of PC
There is normal histology of the bowel in irritable bowel syndrome (C).

242. The answer is E: Answers (A-D) are all known extraintestinal signs of
ulcerative colitis. Granuloma annulare is a benign chronic skin condition
characterized by dermal papules and annular plaques. It has been associated with
ooninsulin dependent diabetes meilirus, but is primarily idiopathic The papules can
occur anywhere on the body and usually resolve in a few months without treatment
being required.

243. T he answer is E: Toxic megacolon occurs in up to 5% o f cases o f ulcerative


colitis and usually occurs during the initial acute episode. The patient will appear
septic, apathetic, and lethargic, with high fever, chills, tachycardia, and progressive
abdominal pain, tenderness, and distention. The cause o f toxic dilation is unknown,
but precipitating factors may include use of antidiarrfaeal agents, vigorous use o f
cathartics or enemas, or barium enema examinations. Toxic dilation occurs
predominantly in the transverse colon, probably because in the supine position air
collects in the transverse colon.

244. The answer is C: a) False, b) False. c)T ruc d) False, c) False.

245. The answer u E: a) False, b) False, c) False, d) False, e) True.

246. The answer is C: Crohn's disease commonly presents with diarrhoea,


abdominal pain and weight loss. It can affect the whole gastrointestinal tract, the
commonest being ileocolitis. Anaemia is usually due to blood loss and less
commonly B12/folate malabsorption. AnabdominaJ mass is often palpable in
presence o f small bowel disease, which can lead to Vitamin K. malabsorption. Anal
tags, fissures, perianal fistulae and abscesses are associated with crohn's disease
and not ulcerative colitis.

M alabsorption

247. The answer is C. Celiac sprue is an inherited disorder that is characterized by


intolerance to gluten, a cereal-type protein found in wheat, rye, oats, and barley.
Symptoms in infancy include colic, failure to thrive, and, in severe cases, iron-
deficiency anemia with the development of edema. In adults, symptoms, include
abdominal bloating and discomfort, with diarrhea, anemia, weight loss, arthralgias,
and edema. Steatorrhea is usually present. Laboratory findings usually include
iron-deficiency anemia (in children), folate-deficicncy anemia (in adults), low
protein level. jnd electrolyte abnormalities and coagulation stucJic. mav be
abrp^rrruil
Celiac disease can be difficult to diagnose. IgA tissue transglutaminase antibodies
ttrCi) and IgA cndornysiaj antibodies (EMA) arc appropriate first-line serologic
tests to rule in celiac disease. The tTG test recommended single serologic test for
celiac disease screening in the primary care setting.
Anngliadio IgA and IgG antibodies are elevated in >90% of patients, however,
they are nonspecific and no longer recommended to test for celiac disease A
positive IgA tTG result should prompt small bowel biopsy to confirm the
diagnosis.
The Schilling test is used to diagnose permcipous aneamia in patients with vitamin
B12 deficiency, and the Scotch tape test is used in the diagnosis o f pin worms.
Withdrawal o f lactose from the diet to monitor for improvement of symptoms can
be utilized when assessing patients for Lactase deficiency. Treatment involves
dietary counseling to avoid gluten-containing foods and supplementary' vitamins. In
severe cases, corticosteroids are used to induce a refractory stage.

248. The answ er is B: There is an eightyfold increased relative risk o f intestinal


h-mphoma in coeliac disease compared to the normal population. These are
predominantly J-c e ll derived lymphomas, and the risk is reduced with effective
gluten exclusion. The other complications and associations listed are relevant to the
inflammatory bowel diseases.

249. The answ er is A: Coeliac disease causes a gluten-related enteropathy o f the


small bowel. When the patient consumes gluten-containing foods, a T-cell
mediated autoimmune reaction results in subtotal villous atrophy and crypt
hyperplasia o f the small bowel mucosa. Specifically, it is gliadin - a constituent of
gluten - which causes the immunological response. The result is malabsorption
leading to malnutrition, anaemia, steatorrhoea, abdominal pain, bloating and
fatigue. The initial investigation for coeliac disease is the measurement o f serum
anti-endo myseal or anti-transglutaminase IgA antibodies, which have been shown
to have sensitivities and specificities o f over 95%. Antibodies to alpha-gliadin and
reticulin can also be measured although they have lower sensitivities and
specificities. The gold standard diagnostic technique is duodenal or jejunal biopsy
taken during endoscopy. Gluten exclusion can reverse the disease process, and
therefore biopsy must occur before exclusion starts. Since this is an invasive and
unpleasant procedure it is usually performed as a second-line investigation in
patients with positive serology. If the biopsy suggests coeliac disease the patient
should be placed on a gluten-free diet and re-biopsied at a later date to assess small
bowel recovery.

250. The answ er is D: The terminal ileum is responsible for the absorption of
vitamin B12. If this vitamin is not supplemented, the patient will experience
symptoms o f glossitis, neuropathy, macrocytic anapmia. The proximal ileum is
responsible for absorption o f vitamin B2 and vrtamib C. The jejunum ;s responsible
for the absorption o f vitamin D folic acid and nicotinamide. The duodenum is
responsible for the absorption o f the minerals calcium and iron.

251. T he answer is C : The patient is suffering from coeliac disease which is a T-


celJ mediated autoimmune disease o f the small bowel characterized by intolerance
to aJcohol-soluWe proteins in wheat, barley, rye and oats (also known as prolamm)
leading to villous atrophy and malabsorption. Patients may present with
sleatorrhoea, diarrhoea, abdominal pain and bloating, nausea and vomiting, signs o f
microcytic anaemia (secondary to iron deficiency) or macrocytic anaemia
(secondary’ to vitamin B 12 or folate deficiency), weight loss, failure to thrive in
children.
Diagnosis is made by:
* testing for antibodies: a-gliadin, tissue transglutaminase and ami-endomysial (an
IgA antibody which is 95 per cent specific for coeliac disease unless the patient is
IgA deficient);
• duodenal biopsy which can be performed at upper GI endoscopy A im o f
treatment is to completely avoid gluten-containing food. From the list o f answers,
(C) is therefore the correct answer.

252. T he answer is E: Menetner’s disease, an uncommon disease involving the


stomach, is characterized by large gastric folds. Intravenous administration o f
radioactive-labeled albumin may show up to a 40% loss in the Gl tract in protein-
losing enteropathy, but is not available for routine clinical use. Treatm ent o f
protein-losing enteropathy is usually directed at the underlying condition.
— -■ — i — ■ -ii" r - -

253. T he answer is B: Postgastrectomy steatorrhea does not result from m ucosal


abnormality. The m ucosa is also normal in pancreatic steatorrhea. Postgastrectomy
maldigestion and malabsorption is caused by rapid gastric emptying, reduced
dispersion o f food in the stomach, reduced luminal levels o f bile, rapid transit o f
food, and impaired pancreatic secretory response.

254. T he answer is C : Dermatitis herpetiformis forms crops o f itchy blisters that


are commonly found on elbows, knees and scalp. The itch can drive people to
suicide and can be usually very effectively and with m uch gratitude relieved by
oral dapsone. Dermatitis herpetiformis is associated with coeliac disease and as (lie
risk o f lymphoma is greatly increased by having both conditions, regular
surveillance is recommended. Psoriasis docs commonly affect the elbow s but
usually does not present with itch and eczem a may present in such sites and with
pruritus but is not strongly associated w ith coeliac disease. Scabies should always
be kept in mind when considering itch, usually in children in the web spaces o f the
hands and feet. Track marks from the burrowing Sarcoptes scabiei mite can often
be seen and harvested for microscopic diagnosis.
255. The answ er is E: a) False. Melaena results from upper gastrointestinal
haemorrhage, from a site proximal to the transverse colon, b) False Obstructive
jaundice gives rise to pale stools, c) False. At least 50 mL of blood is needed, d)
False c) True StcatorThoea involves the passage o f pale, bulky, offensive stools.

256. The answ er is B: Whipple's disease is rare and affects most commonly
middle-aged males. It can affect any organ, but dominated by involvement of small
bowel, causing malabsorption. The organism (Trophcryma whippeli) can be
identified both between and within abnormal macrophages, which stain magenta
with PA$. Treat with prolonged antibiotics eg parenteral penicillin and
farej£0Q>cin for 2 weeks, followed by 1 year o?doxycycline.

257. The an sw er is C: Osteoporosis is present in 15% o f patients with iBD. Risk


factors for osteoporosis in this patient popalahon include frequent corticosteroid
use or comcosieroid use for 3 or more months, vitamin D deficiency, calcium 2nd
magnesium malabsorption, and inflammation leading to increased cytokine release
comriburing to bone resorption. Decreased physical activity is also a risk factor for
oslsoporosis. Patients with risk factors for osteoporosis should undergo screening
with 2 DEXA scan

Diarrhea

258. The an sw er is B: Pseudomembranous colitis is characterized by profuse,


watery diarrhea; abdominal cramps; low-grade fevers; and, occasionally,
heznaxocbezia. The eriologic agent is C. difficile, which produces a toxin that
causes the lesions affecting the colon. The condition is thought to be associated
with antibiotic use in the preceding 2 to 3 weeks (In some cases up to 6 weeks);
however, antibiotic use is not necessary for the condition to occur. The diagnosis
may' be achieved by a laboratory' stool test, which isolates the C. difficile toxin
Sigmoidoscopy or colonoscopy usually shows characteristic yellowish-white
plaques. Treatment includes the use o f metronidazole or vancomycin (which is
more expensive). Complications include dehydration, electrolyte imbalances,
intestinal perforation, toxic megacolon, and, in severe cases, death. Relapse may
occur in up to one-third o f patients after treatment

259. The answ er is A: Overgrowth o f Clostridium difficile in the intestine gives


rise to a condition known as pseudomembranous colitis. The condition results from
the use of antibiotics (especially clindamycin, ompicillin, and cephalosporins)
usually 2 days to 6 weeks after administration. In some coses, it'may occur without
recent antibiotic use Treatment involves the use o f metronidazole. For infections
unresponsive 10 metronidazole, oral vancomycin con be used. Mild coses can be
treated with cholestyramine resin. Saccharomyccs boullardii yeast may also be
helpful in treatment. Complications include dehydration with electrolyte
imbalances, intestinal perforation, toxic raegacolod, and, in s e v e r e cases, death
Relapses may occur in up to 20% of cases

260. The answer is C: Travel to the third world countries can be complicated by
traveler’s diarrhea. The incidence ranges from 4% to >50%. The most common
pathogens are enteropathogens (e g,, E. coli) in approximately 80% o f cases;
occasionally, viruses such as the Norwalk agent or rotavirus arc causative.
Traveler’s diarrhea usually is a self-limited disorder and often resolves without
specific treatment, however, oral rehydration b often beneficial to replace lost
fluids and electrolytes. Clear liquids arc routinely recommended for adults.
Travelers who develop three o r m o r e loose stools in an 8-hour period—especially if
associated with nausea, vomiting, abdominal c r a m p s , fever, or blood in stools—
may benefit from antimicrobial therapy'. Antibiotics are usually given Cor 3 to 5
days. Currently, flucr oquircolooes are the drugs of choice. Commonly prescribed
regimens are 500 mg of ciprofloxacin twice a day or 400 m g o f norfloxacin twice a
day for 3 to 5 days. Trimethoprim-sulihiEcthoxazoLe and doxycycline are no longer
recommended because o f the high level of resistance to these agents. Bismuth
subsalicylate also may be used as treatment: 1 fluid ounce or two 262-mg tablets
every 30 minutes for up to eight doses in a 24-bour period, which can be repeated
on a second day. If diarrhea persists despite therapy, travelers should be evaluated
and treated for possible parasitic infection. The traveler should also take
precautions by eating only freshly prepared foods that are adequately cooked,
eating fleshly peeled fruits, drinking only boiled or bottled water, and avoiding tap
water and ice made from tap water (even in alcoholic drinks).

261. The answer is C: Irritable bowel syndrome is defined as abdominal


discomfort or pain associated with altered bowel habits for at least 3 days/month in
the previous 3 months, with the absence of organic disease. Cramping abdominal
pain is the most common symptom along with diarrhea, constipation, or alternating
diarrhea and constipation. Tbc goals o f treatment are sym ptom relief and improved
quality of life. Exercise, antibiotics, antispasmodics, peppermint oil, and probiotics
appear to improve symptoms. Over-the-counter laxatives and antidiarrheals may
improve stool frequency but not pain. Treatment w ith antidepressants and
psychological therapies are also effective for improving sym ptom s compared with
usual care. Lubiprostonc is effective for the treatment o f constipation-predominant
irritable bowel syndrome

262. The answer is C: PPls such us omeprazole effectively inhibit acid production
in the stomach. This reduces symptoms o f acid-mediated gastritis, peptic ulcer
disease, and gastroesophageal reflux. However, this reduction in stomach acidity
can cause unintended consequences involving processes that uro physiologically
dependent on low pH in the gastrointestinal tract. "Ibeorctical risks include
decreased levels o f vitamin R12, lion, and/or magnesium; decreased bone density;
an increase in gut infections or pneumonia, an increase in gastrointestinal
ncopluMife, ami changes in absorption oi other medications The evidence in
conflicting on some o f these risks. hut consensus is cmciging that chrome use ol
ITIs increases the risk for pneumonia uiul gut infections, pnm anly < loMiidmm
difficile colitis. PPls may also decrease
Kmc density in subsets o f patient*. These risks need to Ik* weighed against the
benefits that these medicines provide Ik foie prescribing them on a longterm basis

263. T he answer Is I): Mclcnn is the passage of black tarry stools, wlmh i*.
secondary to Cll bleeding In most eases, (lie source is located tn the up|Kr <il tr.-u i,
however, a source in die distal right colon or small intestine cun also cause rnclcna
Appioximatdy 100 to 200 m b of blood loss is needed to cause rnclcna. Other
causes lor black stool* that are often confused with mclcnn include iron, br.muth,
licorice, blueberries, and lead. Beet* and tomatoes can .sometime* nuke Mooh
appear reddish.

264. The answer is 0: Diverticulosis, an outpouching of the bowel wall, increases


ua frequency after 40 yean o f age. Acquired diverticular disease affects
approximately 5% to 10% o f the W eston population older than 45 ye2 n and
approximately 80% o f persons older than 85 years. It is more common in the
sigmoid and distal colon. Colonic diverticula are related primarily to two factors:
increased intraluminal pressure and a weakening o f the bowel wall. Patients with
known diverticula have been found to have elevated resting colonic pressures. The
W eston diet, winch tends to be low in dietary fiber and high in refined
carbohydnces, is also believed to be a contributing factor. The condition occurs
when (here is herniation o f bowel mucosa and submucosa through muscular layers
o f the colon. Inflammation o f die small bentiatioos, referred to as diverticulitis,
occurs in approximately 10% to 20% o f patients with diverticulosis and more
commonly in men
Most patients with diverticulosis asymptomatic. Symptoms o f diverticulitis
;nrhirtr lower abdnmmal pain usuafly located on the left that may be steady or
■damning and is relieved whh a bowel movement, anorexia, nausea,
vomiting, and constipation. Physical examination usually shows abdominal
tenderness and guarding aod, occasionally, a palpable .vMnminal or rectal mass
with abscess formation. Occuh blood is present in approximately 20% o f parienrs
Diverticular hemorrhage occurs in 3% to 5% o f patients with diverticular disease*
Fever and an increased WBC count may also be presets. Previously, diverticular
disease* was diagnosed using a contras barium enema. However, because of the
possibility c f an obstructing fecabth bring dislodged by insufflation and cyreino
bowel perforation, CT scanning is now the diagnostic procedure o f choke.
Treag n c s i o f diverticulitis can take place on an outpatient basis for a patient with a
sfiti first acack who is able to tolerate oral hydration and an antibiotic. Treatment
consists c f a liquid diet and 7 to 10 days o f therapy with brood-spectrum
astirrxrebu ls such as ujctmcgdazole znd ciprofloxacin. Patients with severe
or those cannot tolerate oral hydration cr who have pam severe enough
lo rr«|mir narcotic iuiiily,oi:t, r»|ioul«l Ik* hospitalized Jtrcauvc frcdnij/ i i kmni c *
Inlnuolonic prcvsurn, palicnn should receive nothing hy month and diould hr
treated with ini in venom triple llicnipy con'duling of ninpinllin, y.cnlnmiun, arid
metronidazole. Alternative morvilherupy includo pjjicrucillm or (ji/olmcf nri II
naicotici me required for pmn control, meperidine is recommended bccon-c
morphine ml fiilc uniivo colonic ipann. If the pain, fever, and lcukocyl<e,i k do not
tcvolvc within 1 dnyri, further imaging •dudio a/e indicated If mi :ibv.cv, r.
uncovered mid i*» '?> cm in m / c , GI -guided dr'iirci^c mid odequa/*; aolibiofi'.
coverage nhotild be conriidcrcd.
Approximately ?.()% of palienh wiUi divcrticuhln require Mi/grry indication*, for
emergent surgery include peritonitis, uncontrolled vcpr.i,, vivccral perforation,
colonic oh* true* ion, or acute deterioration. \Ur*d rejection n ur.uaJJy
recommended for recurrent epivxlcs of diverticulilw or if fistula, are pre’vent

265. The answer is R : }4o& eases o f acute diarrheal illness are caused by viral
infections (Norwalk vims, rotavirus, enterovmees, and adenoviruses). Other
etiologies o f diarrhea include antibiotic-induced coHm, inflammatory bowel
disease, lactose intolerance, obstruction (paradoxical diarrhea from im paction;,
bacterial toxins, food toxins (mushrooms, ciguatera), individual sensitivity to food,
and systemic illnesses (malabsorption and malignancy).

266. The answer is B: 2 ) False. Diarrhoea or cccstipahon may occur. Explosive


diarrhoea sagged* an infective cause, b) True. c) False. This is a feature o f
organic disease, not a functional disorder. d) False, e) False. This is a sign o f
malabsorption.

267. The answer is A: Enterotoxigenic E Coli is the commonest cause o f travellers


diarrhea and is usually a self Limiting condition. U sually no treatment nor
investigation is required for this brief diarrhoea! illness Other causes that may be
associated with prolonged diarrhoea include Giandia and aznoebtaszs. Chronic
diarrhoea marts investigation.

Gastrointestinal T um ors

268. Answer is: B : Osier-Webcr-Rendu syndrome. FAP is inherited cood in cc


where there axe > 100 adenomatous polyps in the colon and has a high m alignant
potential It is also referred to as Gardner's syndrome, if there are extra-colonic
lesions. These include benign lesions such as: osteomas; dental ^hnnrTre»br*»< Like
super-numerary teeth; skin lesions like epidermoid cysts* sebaceous cysts, fibromas
and lipomas; adrenal adenomas and nasal angiofibromas. Mali grant lesions include
duodenal, periampullary, thyroid, pancreatic, and gastric tumours;
hepatoblastomas, and tumours o f the central nervous system. Peutz-Jegber is
another inherited condition where there may be hyperptgmenstioa o f the x o a ih
and limbs along with development o f hamartomatots polyps throughout the
gastrointestinal tract. Both UC and Crohn's are forms o f inflammatory bowel
disease (IBD) and have an increased risk o f developing colorectal cancer. The risk
develops after 7 years of disease duration and depends on the extent o f bowel
involvement and the severity of bowel inflammation. Over 70% o f patients with
PSC have underlying UC and are at risk o f cholangiocarcinoma and colorectal
cancer; it is for this reason that they should have annual bowel colonoscopic
screening. Osler-Weber-Rendu syndrome or Hereditary Haemorrhagic
Telangiectasia (HHT) is an autosomal dominant condition associated with
epistaxis; cerebral, gastrointestinal, hepatic and pulmonary arterio-venous
malformations (AVM) and muco-cutaneous telangiectasias that can cause bleeding;
and iron deficiency anaemia,. They are not associated with cancer.

269. Answer is D: Most neuroendocrine tumors of the pancreas are not associated
with MEN 1 syndrome, which does not include pheochromocytoma. M any
neuroendocrine tumors produce pancreatic polypeptide, but this hormone does not
cause watery diarrhea. At least half of all islet cell tumors are nonfunctioning. W ith
MEN 1 syndrome, neuroendocrine tumors are more often multifocal and invasive.

270. The answer is D: The U.S. Preventive Services Task Force (USPSTF)
recommends screening for colorectal cancer in adults 50 to 75 years o f age using
colonoscopy, sigmoidoscopy, or high-sensitivity FOBT. Studies show that the
optimal intervals for these tests are colonoscopy every 10 years; high-sensitivity
FOBT annually; and sigmoidoscopy every 5 years combined with high-sensitivity
FOBT every 3 years. Sigmoidoscopy every 5 years without high-sensitivity FOBT
is significantly less effective in detecting colorectal cancer than are other screening
tests. The USPSTF concluded that there is insufficient evidence to determine the
net benefit of CT colonography and fecal DNA testing.

271. TJbe answer is B: CRC is the leading cause of death due to cancer in the US.
The major factors that increase the risk of colon cancer include
1. Age: More than 90% o f people diagnosed with CRC are older than 50 years.
2. Having a personal history of colorectal polyps (risk increases with multiple
polyps, villous polyps, and larger polyps).
3. Having a personal history of cancer: Rectal cancer has a higher incidence o f
local recurrence than proximal cancer (20% to 30% vs.2% to 4%).
4. A history of inflammatoiy bowel disease: Those with ulcerative colitis and
Crohn’s disease have a prevalence of ~3% , with a cumulative risk of CRC o f 2% at
10 years, 8% at 20 years, and 18% at 30 years.
5. A family history o f CRC is present in 10% of adults and about 25% o f cases.
Individuals with one or more first-degree or two or more second-degree relatives
with CRC are at higher lifetime risk of developing CRC, which ranges from two- to
sixfold.
6. Certain genetic syndromes:
• Individuals w ith Lynch syndrome (hereditary nonpolyposis colorectal cancer
[HNPCC]) have a 2% to 3% risk o f developing CRC. They are also at an increased
risk for endom etrial, stomach, ovarian, pancreas, ureter and kidney, biliary tract,
and brain cancers. This syndrome should be considered in people w ith m ore than
one family m em ber w ith CRC or other Lynch sy n d ro m e- related cancers o f small
bowel, ovary, o r endometrium w hich has occurred at a young age (30s and 40s).
• Fam ilial adenom atous polyposis (FAPP): Tum ors beginning in th e 20s, and
nearly all w ill develop CRC (usually before age 50). • Individuals w ith P eutz-
Jeghers syndrom e (often have freckles [mouth, hands, feet] and large polyps in GI
tract) are at an increased risk o f CRC.
7. A personal history o f childhood cancer requiring abdom inal radiation therapy.
8. Race and ethnicity issues:
• A frican A m ericans have highest CRC incidence and mortality rates in the US.
• Several different gene mutations have been identified am ong Ashkenazi Jews.

272. T h e a n s w e r is B: Despite some decline, distal tumors are still the m ost
common. The fact that up to 60% o f tumors are located in the rectosigm oid is the
rationale for screening via flexible, fiberoptic sigmoidoscopes: Occult blood testing
and colonoscopy are other possible screening techniques.

P an crea tic D iseases

273. A n sw er is C : The cause o f acute pancreatitis is not clearly a predictor o f


severity. A ge older than 60 years, a body mass index greater than o r equal to 30,
very increased level o f C-reactive protein, pleural effusion, and pulm onary
infiltrates are all predictors o f severity. Persistent organ failure as w ell as multiple
organ failure is am ong the strongest predictors o f severity.

274. T h e a n s w e r is B: Acute pancreatitis is caused by biliary tract disease,


alcoholism, hyperlipidemia, hypercalcemia, hyperparathyroidism, trauma,
m edications (e.g., ftirosemide, valproic acid, sulfasalazine), infections, and
structural abnorm alities o f the biliary tract. Symptoms include constant, boring,
abdom inal pain that radiates to the back; nausea; and repeated vom iting w ith a low-
grade fever. Physical examination shows a distended rigid abdomen w ith positive
peritoneal signs, tachycardia, tachypnea, and signs o f dehydration and shock.
Laboratory tests show an elevation in serum lipase (more sensitive) and amylase,
elevated W BC count (12,000 to 20,000 per m m 3), elevated liver function tests,
increased bilirubin, hyperglycemia, and hypocalcemia. Chest radiographs may
show pleural effusions. Abdominal films may show the presence o f a sentinel loop
(ileus o f the transverse colon). Ultrasound or CT exam ination may show evidence
o f gallstones, dilation o f the com mon bile duct, or edema o f the pancreas.
Pancreatitis associated with hem orrhage or necrosis o f the pancreas h as a mortality
rate that approaches 50%. Hemorrhage is suspected if there is a grayish-blue
discoloration o f the back or flanks o f the patient’s body (Grey T u rn er’s sign) or
ultecUng the periumbilical area (Cullen’s sign). Treatment involves bowel rest with
nasogastric suction and fluid resuscitation with correction of electrolyte
disturbances. Ranson’s criteria arc used to predict outcome at the tunc ot
admission:
l* Older than 55 years 2« WBC count > 16,000'mm3
3* Serum glucose >200 mg/dL 4* LDH >350 III per L
5* AST level >250 IU/L 48 hours after admission
6* Greater than 10% decrease in hematocrit
7* Elevation in BUN >5 mg/dL 8» Serum calcium levels ^ mg/dL
9* Base deficit >4 mEq/L 10» Arterial P 02 <60 mm Hg
11 • > 6 L of fluid deposition in the body's interstitial spaces
If < 3 signs are present at the time o f admission, the mortality rate is <5%. 'Hie
presence of > 3 signs on admission has an associated mortality' rate of 15% to 20%
If > 7 signs total are present, the mortality rate approaches 100%.

275. The answ er b D: Acute pancreatitis usually results from alcohol abuse, bile
duct obstruction, or severe hypertriglyceridemia. Patients with acute pancreatitis
present with mild-to-severe epigastric pain with radiation to the flank, back, or
both. Classically, the pain is characterized as constant, dull, and boring and is
worse when the patient is supine. The discomfort may lessen when the patient
assumes a sitting or fetal position. A heavy meal or drinking binge often triggers
the pain. Nausea and nonfeculent vomiting are present in the vast majority of
patients.
Serum amvlasc and lipase (the most sensitive and specific laboratory indicator for
pancreatitis) levels are used to confirm the diagnosis o f acute pancreatitis. CT will
confirm the diagnosis, assess severity, establish a baseline, and rule out other
possibilities. CT scan with IV contrast at day 3 can assess the degree o f necrosis
when necrotizing pancreatitis is suspected (02 saturation <90%, systolic BP <90,
worsening symptoms).
IV rehvdration should usually be aggressive, with close attention to blood pressure
and cardiac and pulmonary status. Withholding food by mouth docs reduce pain;
however, the use o f a nasogastric tube with suction is indicated only for intractable
emesis. In mild pancreatitis, oral intake should be withheld until the nausea and
vomiting subside. Total enteral feeding beyond the ligament o f Treitz is considered
after 48 hours, if oral feeding will not be possible within 5 to 7 days. Systemic
antibiotics remain controversial.

276. The answ er is A: From the history, it is clear that the patient is suffering
conjugated hyperbilirubinaemia with symptoms o f jaundice coupled with dark
urine and pale stools. The liver function tests support a diagnosis of cholestasis -
bilirubin of 7 mg/dL, with an unparalleled rise in ALP (350 iu/L). AST and ALT
are mildly elevated in comparison. Therefore, from the list of possible answers,
gallstones (A) are the most likely diagnosis. With viral (B), alcoholic hepatitis (C)
and autoimmune hepatitis (E) one would expect elevation in ALT and AST
enzymes Hue to hepatocellular damage There is no history o f weight loss which
makes pancreatic carcinoma (D) unlikely

277. The answ er Is A: Jaundice is a feature o f acute pancreatitis due to


inflammation of the pancreatic head obstructing bilious outflow Psendocvst
formation is a late complication o f pancreatitis and usually takes about a week to
develop thus not seen in the acute inflammation Hypocalcacmia as opposed to
hypercalcaemia is often seen and is useful as one of the Glasgow criterion for
predicting severity o f pancreatitis along with a low albumin. Bowel necrosis »s not
associated classically with pancreatitis.

Miscellaneous

278. The answer is B: Hemochromatosis is a result o f excessive iron deposition in


the body (hemosiderosis) that leads to damage of bodily tissues Primary hereditary
hemochromatosis is an autosomal-recessive trait Persons who are hom ozygous for
the HFE gene mutation C282Y comprise 90% o f phenotypically affected persons.
It is the most common form o f hemochromatosis, affecting approximately 5 m
1,000 persons. End-organ damage or clinical manifestations o f hereditary
hemochromatosis occur in approximately 10% of persons homozygous for C282Y
Complications include
• Cirrhosis
• Diabetes mellitus
• Multiple joint pain
• Abdominal pain
• Chondrocalcinosis
• Bronze discoloration o f the skin
• Cardiomyopathy that may result in cardiac en largem ent CHF, and cardiac
arrhythmias
• Hepatomas
• Pituitary dysfunction leading to testicular atrophy and decreased sexual drive The
onset is usually in the fourth and fifth decades of life. T he condition is rare before
middle age. Diagnosis in women usually occurs after menopause because
menstrual blood loss helps provide protection from iron overload. Laboratory
findings show serum iron >300 mg per dL, serum ferritin >1,000 ng per inL, and
transferrin saturation >50%. Liver biopsy confirms the diagnosis w hen hepatic
siderosis and cirrhosis is suspected. Treatment involves phlebotomy (500 mL per
week = 200 to 250 mg iron) to remove excess iron from the body and the chelating
agent deferoxamine in severe cases, which promotes urinary excretion o f iron.
Family members o f those affected should be screened for hem ochrom atosis with
HLA typing and iron studies.

279. The answ er is C: Peutz-Jeghers syndrome is a familial autosomal dominant


condition that involves the developm ent o f multiple, benign, ham artomatous polyps
in the slontach and in the smull and large intestine Malignant change has occurred
but is rare riinsc affected ulso have mclanin-a^sociated l»rownish*bl;iik
hyperpigmenlalion mound the oral cavity, lips, soles o f the feet, and dorsum ol the
bunds Ihe condition usually causes no problems except in scscrc cases in which
abdominal pain, mtc »tinul obstruction, or bleeding can occur In then* severe eases,
suigcry may be considered.

280. The answ er is K: Ascites cun be described as the pathological accumulation


ol lluid in the ubdominal cavity Ascites occur scconJ.u) to
• conditions leading to venous hypertension (e g. cirrhosis, congestive heart failure,
constrictive pericarditis, Dudd-Chian s>ndrome. portal vein thrombosis);
• hy|)oalbuminucmia (e g. nephrotic syndrome, malnutrition!.
• mulignant disease (e g. secondary me lust uses ol carcinomas of breast, ovary,
colon);
• infections (e.g. tuberculosis),
• others (e g. pancreatic disease, osarian disease, myxoedema).
Answers A -D are all known causes of ascites that occur secondary to venous
hypertensiou. Nepltrotic syndrome (E), however, leads to ascites secondary to
hypoal buminaemi a.

4 NEPHROLOGY

G lom erulopathy

281. A nsw er B: Focal segmental glomerulosclerosis Hepatitis C-associated renal


diseases include mixed cryoglobulinemia, membranoproliferative
glomerulonephritis (MPGN), membranous nephropathy, and polyarteritis nodosa
(PAN). Membranous nephropathy. MPGN, and PAN can also be observed in
hepatitis B. Focal segmental glomerulosclerosis is not particularly associated with
hepatitis C.

282. A nsw er O: Hypocalcemia. Nephrogenic diabetes insipidus (and central


diabetes insipidus) can occur in patients with lithium nephrotoxicity. Distal RTA,
chronic interstitial nephritis, and nephrotic syndrome (related to minimal change
disease and focal segmental glomerulosclerosis) have been described in patients
with lithium nephrotoxicity. Whereas hyperparathyroidism with resultant
hypercalcemia has been described in patients taking lithium therapy, hypocalcemia
is n o t a manifestation o f lithium toxicity.

283. The an sw er is D: Goodpasture's syndrome is a condition manifested by


pulmonary' hemorrhages and progressive glomerulonephritis. Circulating basement
membrane antibodies are responsible for the renal and pulmonary abnormalities.
Patients with Goodpasture's syndrome are typically young males (5 to 40 years;

1 J !>
male. tcmalc ratio of 6.1); however, there is a bimodai peak at approximately oO
years of age Men and women are equally affected at older ages Symptoms include
severe hemoptysis, shortness o f breath, and renal failure Laboratory findings
include iron-dcficicncy anemia, hematuria, proteinuria, cellular and granular casts
in the urine, and circulating arUiglornerular antibodies. Chest radiographs show
progressive, bilateral, fluffy infiltrates that may migrate and are asymmetrical
Renal biopsy may be necessary to make the diagnosis Treatment involves
high dose steroids, immunosuppression, and plasmapheresis, which may help
preserve renal function. If significant injury to the kidneys occurs, then dialysis or
transplant may be necessary. Untreated, Goodpasture’s syndrome can be fatal.

284. The a n s w e r i s D : Group A beta-hemolytic streptococcal pharyngitis, scarlet


lever, and in rare cases asymptomatic carrier states are associated with
poststreptococcal complications. Children are most commonly affected in
streptococcal pharyngitis, acute rheumatic fever, pediatric autoimmune
neuropsychiatric disorders associated with streptococcal infection, and
poststreptococcal glomerulone-phritis. The hallmarks of rheumatic fever include
arthritis, carditis, cutaneous disease, chorea, and subsequent acquired valvular
disease. Pediatric autoimmune neuropsychiatric disorders include a subgroup o f
illnesses involving the basal ganglia in children with obsessive-com pulsive
disorders, tic disorders, dystonia, chorea encephalitis, and dystonic choreoathetosis
Poststreptococcal glomerulonephritis occurs most frequently in children between 2
and 6 years o f age with a recent history o f pharyngitis and a rash during the winter
months.
The clinical examination of a patient with possible poststreptococcal complications
should include an evaluation for signs of inflammation (i.e., CB C, erythrocyte
sedimentation rate, C-reactive protein) and evidence o f a preceding streptococcal
infection. Antistreptolysin O titers should be obtained to confirm a recent invasive
streptococcal infection. O ther important antibody m arkers include
antihyaluronidase. antideoxynbonuc lease B, and antistreptokinase antibodies.

28S. T he answ er b D: Nephrotic syndrome is a triad o f proteinuria,


hypoalbuminaemia and oedema- The proteinuria is classically quantified as more
than 3.5 g in 24 hours. When phaeochromocytoma is suspected, 72-boui unit,
collection is performed to look for urinary metanephrines and other catecholamine
metabolites. Quantification o f proteinuria, rather than simply dipstick testing, is
important as both hypoalbuminaemia and proteinuria are nonspecific and can be
present in important clinical dilTerentials (cardiac failure can feature mild
proteinuria, and liver failure can result in hypoalbuminaemia - both cause oedema
and fatigue). A newer test that can spare the hassle of 24-hour urine collection is
the urine protcin creatirunc ratio, which can reliably predict the am ount o f protein
excreted in urine over 24 hours from a random urine sample. N ephrotic syndrome
is most commonly caused by glorncrulonephn tides, and so further tesUng should
look for other features o f glome rulonephritis and causes thereof (renal autoimmune
screen, scrum and urine electrophoresis, urine microscopy) Adults with nephrotic
syndrome should all gel a renal biopsy to further elucidate the cause; in children,
minimal chance glomerulonephritis is the most common cause, and this normally
reverses with steroids, hence biopsy can be avoided unless there are other
concerning features or there is little response to steroids.

286. The answ er is C: Haematuria may be macroscopic with blood evident in the
urine or microscopic requiring urine dipstick testing The anatomical origin of
macroscopic haematuria can often be deduced from its piescnlalion in the urine,
although this should not be relied upon Bleeding from the bladder or above usually
presents throughout voiding, terminal bladder or prostatic bleeding occurs at the
end o f voiding, while urethral sites present at the beginning. Microscopic
haematuria identified by urine dipstick requires microscopic analysis to conffrm red
blood cell presence. Red cell casts are red blood cells that have leaked into renal
tubules and clump together forming a cast-like structure which is excreted into the
urine The presence o f red cell casts are therefore strongly suggestive o f glomerular
pathology. False-positive results may arise from haemoglobin or myoglobin in the
urine
IgA nephropathy or Berger’s disease ( C j is the mos1 common cause of
glomerulonephritis and may present at any age. Haematuria is usually macroscopic
and occurs in intervals corresponding with glomerular attacks, infections such as
pharyngitis can exacerbate the condition. I Icnoch-Schonlein purpura (A) differs
from Berger’s disease through more systemic involvement, often presenting with
arthritis of the large joints, abdominal pain and a characteristic purpuric rash of the
extensor skin surfaces. The absence of pain and genital symptoms excludes a UTI
(E). Diabetic nephropathy (D) typically presents with proteinuria and not
haematuria.
Benign prostatic hypertrophy (B) occurs in much older patients often alongside
poor urine flow.*

287. T he answ er is D: This patient is suffering from post-streptococcal


glomerulonephritis (D), which forms part o f the nephritic syndrome consisting of
haematuria (micro- or macroscopic), hypertension, proteinuria and oedema. In
severe cases, oliguria and uraemia can also occur. Patients usually suffer from a
streptococcal infection l-3 weeks prior to presenting with the above symptoms or
signs o f the nephritic syndrome. During this time, immune complexes are formed
and deposited in the glomeruli causing damage. The nephrotic syndrome (A)
involves albuminuria usually in the order of >3.5 g/d in adults causing oedema and
is also associated with hyperlipidaeraia (increased LDL/HDL ratio). Nephritic
syndrome (B) involves haematuria (micro- or macroscopic) alongside hypertension
and proteinuria Renal failure (C) is an abrupt reduction in kidney function, usually
*48 hours, with an increase in serum creatinine of >26 4pmol/L or reduction in
urme output A Von Oniwil/ tumour (F), otherwise known as renal cell cancer,
typically occyr m males (2 I male to female ratio) originating from the proximal

J 37
tubular epithelium The average age of presentation is 30 years with symptoms
including pain, haematuria and usually a m ass in the tlank alongside other
symptoms o f malignancy such as weight loss.

288. T he answ er is B: Diabetic nephropathy is an insidious complication of


diabetes which is often missed since renal compensation can cause patients to
present only when they are close to end-stage renal failure Detection o f
microalbumin (B) m the utidc has been shown to be a good m arker for identifying
patients most likely at risk o f further renal damage Urine dipsticks are not
sensitive enough to detect microalbumin which normally should not be present m
the urine. Measurement o f blood pressure (A) is a broad marker o f cardiovascular
integrity. It can both cause and be the product o f renal im pairm ent and is not
appropriately specific to monitor loss o f renal function. Sim ilarly, serum
electrolytes C D ) such as sodium and potassium can be abnormal due to a number o f
causes not necessarily related to loss of renal function due to diabetes, for example
S1ADH. Elevation in serum creatinine (C), which ls usually excreted by the kidney,
is a late marker of function renal impairment and not appropriate for early risk
identification. Urine dipstick for glucose fE) is a screening test for glycaenuc
control, however, it does not predict specific increased risk for ncphropathology
but simply increases the likelihood o f all diabetic complications.

289. T he an sw er is B: The symptoms and biochemical markers o f renal failure and


lung involvement, i.e. haemoptysis should raise the concern o f hcpotopulm onary
diseases such as Goodpasture's syndrome. A proliferative glom erulonephritis
associated with pulmonary infiltrates causing haemoptysis (som etim es massive in
nature) arc the pathological hallmarks of this aggressive disease. Antibodies to
basement membranes affecting both glomeruli and alveoli arc the typical antibodies
exhibited by patients with Goodpasture’s syndrom e Anti-neutrophil cytoplasm ic
antibodies (ANCA) are more strongly associated with systemic vasculitidcs such as
Wegener’s granulomatosis (PR3-ANCAX m icroscopic polyangiitis and C hurg-
Strauss disease (MPO-ANCA). Anti-SCL 70 is classically associated with systemic
sclerosis, anti-mitochondrial antibodies often seen in cirrhosis o f liver and biliary
tree and rheumatoid factor associated with rheumatoid disease and associated
syndromes, c.g. Felly’s syndrome. It is worth noting, however, th at no antibody is
entirely specific or sensitive to a particular disease and there are m any patients
without any disease who will exhibit positive antibody titres.

290. T he an sw er is (B). Post-streptococcal glomerulonephritis is associated with


streptococcal upper respiratory tract infections and usually presents anywhere
between a week and 3 weeks post-infection. Haem aturia, often described as dark or
smoky rather than frank blood, is frequently encountered. O edem a, especially o f
the periorbital region, is a com m on presentation in children with poststreptococcal
glomerulonephritis and accompanies proteinuria, hypertension and oliguria.
Treatment is usually ^ipportiyc with spontaneous resolution being the rule.
Minimal change glomerulonephritis is ihe most common cause o f the nephritic
syndrome in children. Symptoms arc similar with oedema, hypertension and
association witn upper respiratory tract infection; however, thromboeniNMic events
due to loss of anti-clotting products in the urine make the diagnosis more likel>
Corticosieroids form the mainstay of treatment
Henoch-Schfinlcin purpura is a vasculitis o f small vessels that affects vouog males
twice as frequently as females and manifests as punctate bleeding points associated
with abdominal pam and gastrointestinal bleeding. Characteristic 'bruising' occurs
over the buttocks and lower limhs and may sometimes be mistaken for non-
accidental injury in a child.
Bentcr's disease is the commonest primary glomerulonephritis worldwide. Immune
complex deposition is thought to be part o f the pathophysiology of Berger's disease
with ensuing glomerular damage and renal failure. Treatment may be supportive
only if the degree of renal failure is mild
Rapidly progressive glomerulonephritis describes a pathological reduction in
glomerular filtration rate over a short space o f time and mas be associated with
anti-glomerular basement membrane annbodies, immune complex deposition or
vasculitis-associated disease. Imniunosuppcession is the treatment modality of
choice.

291. T he answer is E: Haematuria is seen in nephritic conditions such as


glomerulonephritis and is not associated with nephrotic syndrome by definition.
Components o f the nephrotic syndrome are proteinuria defined as protein loss in
the urine at a rate of greater than 3g in a 24-hour period, nypoalbuminacmia
defined as an albumin level < 30g/L and oedema. Fourth and fifth elements are
sometimes described as port o f the syndrome' and are hypercholesterolaemia and
normal renal function respectively. Nephrotic syndrome can occur as a primary
event or a secondary phenomenon but this definition is not clinically of great
benefit in the management o f cases o f nephrotic syndrome as treatment is
supportive in the first instance whilst the underlying cause is sought Management
focuses on use of diuretics to reduce oedema along with fluid and salt restriction,
adequate nutrition to replace protein loss (approximately l-2 g /kg/day) and venous
thromboembolism prophylaxis unless contraindicated. This is due to the loss of
clotting factors with heavy proteinuria and predisposes those with the nephrotic
syndrome to thromboembolism. Hypercholesterolaemia usually requires no active
treatment and will resolve alongside the nephrotic syndrome, however, if
persistent, a role for statins may be indicated.

292. T he answer is D: Rifamoicin is known for causing a reddish/orange


discolouration to the urine among other non-therapeutic effects that include liver
enzyme induction and resistance when used not in combination with other anti­
tuberculous medications. These effects are helpfully summed up by ‘the three R’s
of Rifampicin' (Rev., up Uver enzymes, Red'orange discolouration and Resistance
when used#
alone).•
I rue causes ot haemaluria are best described anatomically from the kidney
downwinds and include glomenilonephntic processes, polycystic kidney disease,
trmiuu and unnary tract stone. Infectious processes include TB. cystitis of the
bladder nnd schistosomiasis (especially if the patient has a positive travel history )
and prostatic carcinoma, urethral trauma, urethritis or neoplasm General causes o f
abnormal coagulation must always be e x c l u d c t l in any case o f bleeding and
inherited bleeding disorders such as hacmopliilia and uny derangement m
coagulation profile must be excluded.
Mularia can cause hneinuturia and a full history of recent travel mu>t be elicited
Investigation of this often benign but occasionally life-threatening parasitic disease
must be undertaken swiftly but should not delay empirical treatment if laboratory
turn around times o f thick and thin films looking for malarial parasites is slow.
Treatment of Plasmodium falciparum malaria is of vital importance due to the
association with cerebral malaria and should depend on local sensitivities to anti-
malarial drugs; however, a week’s course of quinine followed by fansidar (with
glucosc-6-phosphatasc deficiency. G6PD, investigation) is usually an adequate
treatment regime. G6PD deficiency poses a risk of haemolytic anaemia with certain
medications notably antimalarials and G6PD status is usually taken as soon as
possible to but not delaying the start o f treatment.

293. The answer is B: Although there is no known effective treatment for IgA
nephropathy, there have been reports of favourable response to long term
corticosteroid therapy. 80% adults with minimal change GN will respond to
steroids, although remissions can take up to 16 weeks. M embranous GN does not
respond to steroid treatment. No specific treatment isavailable to cause regression
o f amyloid deposits.

Renal Failure

294. The answer is D: Serum recombinant erythropoietin is used to treat refractory


anemia in patients with chronic renal disease. The synthetic drug replaces
erythropoietin that is normally produced by the kidneys. Although expensive, the
drug is indicated if the patient has significant anemia that is not caused by other
factors. The major side effect is hypertension, which must be monitored at regular
intervals. Other side effects include polycythemia, with the possible developm ent
o f thromboembolism, stroke, and Ml. The drug docs not appear to accelerate the
preexisting renal disease. Close monitoring o f serum hem oglobin is necessary w ith
the use of erythropoietin. Iron supplementation must be given to achieve an
adequate erythropoietin response.

295. T h e answer is A: Acute renal failure is divided into the following categories:
« Prerenal This is due to inadequate renal perfusion. It can be caused by volum e
depletion (dehydration), cardiac or hepatic failure, and sepsis. Laboratory tests
rc\cal a low Virihary .sodium (<-20 mEq/L) and a high urine-to-plasma crcahmnc
ratio 0 2 0 1V The BlTN-io-serum creatinine ratio is higher ilian 20.
• Inirarcnal This was previously known as acute tubular necrosis Causes include
ischemia, hypertension, vasculitis, metabolic disorders (e g ., hypercalcemia,
hyperuricemia), toxins, x-ray dyes, myoglobinuria, and medications (e g ,
aminoglycosides, penicillins, anesthetic agents) Laboratory tests show results
similar to postrenal azotemia.
• Postrenal This is usually caused by obstruction by renal calculi or bladder outlet
obstruction (prostate enlargement) Laboratory tests show a high urinary sodium
(>40 mEq/L) and a low urine-to-plasma creatinine ratio (<20.1). The BLTN-to-
serum creatinine ratio is lower than 20.
• Prerenal and postrenal causes for acute renal failure are potentially reversible. If
caught early, some forms o f intrarenal azotemia (c.g., drug effects, infections,
hypertension) can be reversed.

296. T he answer is B: In prerenal failure, tubular and glomerular function is


maintained. The most common cause o f intrinsic renal failure, traditionally known
as acute tubular necrosis and now called acute kidney injury, is ischemic acute
renal failure. It occurs when renal perfusion is decreased so much that the kidney
parenchyma suffers ischemic injury. During an ischemic insult from decreased
renal blood flow, cytokine-mediated alteration o f nitric oxide production and
increased production o f systemic vasoconstrictors cause worsening periglomeruJar
vasoconstriction and further depress renal blood flow which, along with
microvascular congestion, red blood cell trapping, and activation o f the coagulation
cascade, magnifies ischemic injury. During periods o f depressed renal blood flow,
the kidneys are especially vulnerable to further insult from nephrotoxic agents such
as radiocontrast agents and aminoglycosides. Individuals with chronic hypertension
develop altered renal autorcgulation that establishes conditions under which renal
ischemia can occur in spite o f systemic blood pressures that would be normal, not
low, for most patients.

297. T he answer is E: Microscopic examination o f the urine is very useful in


establishing the cause o f renaJ failure. Acute glomerulonephritis will produce red
cell costs in the urine. Acute interstitial nephritis will result in white cell casts and
eosinophils in the urine. In acute tubular necrosis, the tubular epithelium^ breaks
down and results in tubular epithelial cells and pigmented granular casts in the
urine. Prerenal azotemia results in hyaline casts in the urine, although a few hyaline
casts in a urine sample may be normal. Conditions that cause the filtration o f
pigmented proteins, such as myoglobinuria and hemoglobinuria, as you would see
in rhabdornyolysis. will result in brown granular casts in the urine.

298. T he answer is E: Hypotension occurs in up to 30% o f hemodialysis


treatments, usually caused by excessive ullrafiltration alter underestimating the
patient's ideal (dry) weight. Other complications that occur less frequently include
air embolism, cerebral edema from dialysis disequilibrium, hemorrhage from
accessing the vascular access site, hypercalcemia, hypermagnesemia, and
hypoglycemia.

299. The answer is A: It is recommended to keep the BUN lower than 100 mg/dL
However, there is no absolute BUN value at which to dialyze a patient. Each
patient should be evaluated by a nephrologist to assess the need for dialysis

390. The answer is E: Benign pro?tatic hypertrophy can cause armory retention
and increased pressure in the urinary outflow tract, and as this disease affects older
men. who can be quite reluctant to seek help with non-urgent medical problems,
this can get to the stage at which kidney function is impaired. Recurrent kidney
stones passing into the ureter car. -uuse scarring, especially at narrow sites such as
the vesico-ureteric junction. Retroperitoneal fibrosis typically involves a fibrosing
inflammatory reaction starting in the wall o f the aorta and spreading
retropcritoneally. The ureters can become embroiled in the Gbrous tissue
Obstruction is believed to be due to a loss of peristalsis rather than occlusion.
Schistosomiasis is common m the Middle East and parts o f Africa, featuring
granulomas forming around the eggs o f schistosomes in the urinary tract, and these
can obstruct the urinary tract. Systemic sclerosis leads to a fi brinoid thickening o f
the afferent arterioles, leading to reduced renai perfusion and thus renal
impairment, but it does not cause an obstructive uropathy

301. The answer is E: Amlodminc is a calcium channel blocker commonly used in


hypertension and does not affect serum potassium Bcndroflumcthiazide is a
thiazide diuretic and actually lowers serum potassium. Whilst it is largely
ineffective at lowering blood pressure in renal failure, it may be reducing the serum
potassium and so shouldn't be stopped. Furosemide is a loop diuretic, which also
lowers scrum potassium. If this man has come in with acute renal failure caused by
hypovolaemia, furosemide should probably be stopped. Simvastatin is a statin and
does not affect scrum potassium. Spironolactone is an aldosterone antagonist and
will increase the serum potassium, and for this reason it should be stopped in renal
failure, especially here, where the potassium level is raised. A serum potassium
level of 5.5 mmol/L is oot too high, but it should be checked that potassium is not
being added to (he patient’s fluid prescription, as well as confirming that he is
asymptomatic and checking his electrocardiogram (ECG) for changes.

302. The answ er is B: Hypertension is not acutely life threatening, whereas


acidosis, hyperkalaemia, pulmonary oedema and uraemic pericarditis can
potentially be life threatening. Reliable pharmacological therapies th a t.a re not
overcome by the ongoing renal failure exist for hypertension, whereas they do not
for severe acidosis, persistent hyperkalaemia, refractory pulmonary oedem a or
uraemic pericarditis. Vasodilation and decreasing vascular resistance does not
become impossible in renal failure. Acidosis can be tc nporarily am eliorated by use
oi sodium bicarbonate but it is unclear whether this helps in the long run. especially
if the renal failure does not reverse. Severe hyperkalaemia can lead to ventricular
fibrillation - monitoring should be instituted, calcium gluconate given to stabilise
the cardiac membrane, and insulin with dextrose and calcium resonium given to
lower. serum potassium. However if the renal failure does not reverse the
potassium will continue to rise. Morphine can relieve respiratory distress and
nitrates cun vasodilate and pool blood away from the lungs in pulmonary oedema,
but if the renal failure does not reverse, dial)sis will be needed to remove the
excess fluid Acute nses in urea can lead to pericarditis and even pericardial
effusions and tamponade, and only haemofiltration or haemodialysis can reduce the
scrum urea to prevent this if the kidneys do not soon recover.

303. T he answer is B: Hyperphosphatemia rarely causes any symptoms directly


Its secondary effects on calcium can result in bypocalcemic tetany or metastatic
calcification. The usual cause of hyperphosphatemia is uremia.

304. T he answer is £ : The typical renal lesions in gout are urate crystals in the
medulla or pyramids, with surrounding mononuclear and giant cell reaction. The
degree o f renal impairment, however, does not correlate with hyperuricemia, and
the decline in renal function correlates with aging, hypertension, renal calculi, or
unrelated nephropathy

305. T he answer is D: H vpertal^m ia is associated with CRT and is most


important in the setting o f acute on when levels can be rapidly elevated due to a
sharp fall in glomerular filtration rare CRT. Treatment o f acute severe
hyperkalaemia (potassium level above 6 Smmol/I. or electrocardiogram changes) is
managed firstly by IV injection o f lOmL of 10% calcium gluconate as
cardioprotecnon. Secondly infusion o f e-g. 10 units of short-acting insulin with
50mL o f 50% dextrose over 30 minutes will shift potassium intracellularly.
Correcting the underlying cause, e.g. hypovolaemia may stabilise the potassium
level, however, depletion o f total body stores o f potassium may be indicated and
can be achieved by using calcium resonium. Other electrolyte abnormalities may be
a dilutional hyponatraemia secondary to fluid retention, hyperphosphataemia and
associated hypocaJcaemia due to reduced renal hydroxylation o f vitamin D with
uraemia and metabolic acidosis due to impaired hydrogen ion excretion.
Neurological manifestations of CRT include restless legs syndrome, peripheral
neuropathy and non-specific signs such as dizziness and headache associated with
dialysis.

306. T he answer is B: Oliguria is not an indication for dialysis itself and the cause
should be investigated and treated, e.g. hypovolaemia. Manifestations of oliguria
such as fluid overload refractory to treatment or build-up o f toxins such as urea and
nitrogenous compounds may, however, influence the decision to dialyse. A useful
mnemonic fos remembering the indications for dialysis is ‘A EiO U' standing for
,— -
Acidosis, Electrolyte abnormalities, Ingestions, Overload and Uraemic symptoms.
Severe or worsening metabolic acidosis with a pH < 7.2 and persistent
hyperkalaemia >7mmol/L 2rc good markers of the need for dialysis Fluid
overload, e.g. pulmonary oedema, that is likely to cause respiratory embarra ^nert
and/or cardiovascular instability may be adequately managed with dialy -h
medical therapy nas failed. In the setting of acute overdose, especially with
metabolic derangement as may be seen with aspirin overdose, dialysis can be a
useful means of stabilising a patient until adequate acid-base homeosLas»_» :s
achieved.

307. The answ er is C: Caeruloplasmin and serum copper arc useful tests in the
investigation of Ever disease of unknown aetiology and arc sensitive indicators of
the presence of Wilson’s disease (hepatolenticular degeneration). A renal
ultrasound scan is useful to identify any obstruction, scarring from previous
infection or altered renal size indicative of long-term renal damage or chronically
reduced renal blood flow Safe and non-invasive to perform this should always be
considered in the investigation o f renal disease. Complement levels (C3 and C4) as
well as double-stranded DNA (dsDNA) and erythrocyte sedimentation rate (ESR)
should be used as a screen for systemic lupus erythematosus (SLE, lupus) nephritis.
Generally, an elevated ESR and anti-dsONA and low C3 and C4 levels are
associated with active nephritis. Anti-neutrophil cytoplasmic antibodies (ANCA)
are useful in the detection o f vasculidc diseases affecting the kidney such as
Wegener’s granulomatosis and Churg-Strauss disease among many. Bence-Jones
protein (light-chain fragment) is a simple and effective screening tool for suspected
myeloma with associated light-chain nephropathy and should never be omitted
from the diagnostic work-up o f renal failure.

308. The answ er is A: Oliguria defined as < 400tnl urine/day. Red cell casts
present in acute glomerulonephritis, renal vasculitis, accelerated hypertension and
interstitial nephritis. Pre-renal failure is renal dysfunction due to hypoperfusion
(urinary sodium < 20, urine osmolality > 500, urinc/plasma ratio > 8, and
urine/plasma creatinine > 40) and acute tubular necrosis is acute renal failure due to
circulatory compromise and/or nephrotoxins (urinary sodium > 40, urine
osmolality < 350, unnc/plasma ratio < 3, and urine/plasma creatinine < 20).

309. The answ er is A: Causes o f acute renal failure can be divided into pre-renal.
renal and post-renal.
Pre-renal:
o Hypovolaemia (gastroenteritis, bums, sepsis, haemorrhage, Nephrotic
Syndrome),
o Circulatory failure.
Renal:
o Vascular: HUS, vasculitis, embolus, renal vein thrombosis
o Tubular:.acute tubular necrosis, Ischaemic, toxic, obstn clive.
o Glomerular glomerulonephritis,
o Interstitial, interstitial nephritis, pyelonephritis
o Acute chronic renal failure.
Post-renal.
obstruction, either congenita) or acquired. Although Alport’s Syndrome is
associated with end stage renal failure, this usually progiesses gradually so that it
(.►ccurs in adult life

310. The answer is B: Acute tubular necrosis (B) is most commonly due to renal
ischaemia, as in this case, though direct pharmacological toxicity can also be the
cause among many others including haemorrhage, diuretics, contrast during
radiological procedures and heart failure. The clinical course is dependent on the
offending factor and degree o f damage but most commonly early oliguria followed
by recovery of renal function with an increase in renal output. GKR. however, may
remain low due to tubular damage. Full renal capacity' is usually regained within 6
weeks of the initial stressor.
Haemolytic uraemic syndrome CHUS) (A) defines the acute injury to the kidney
from RBC fragmentation which usually originates from thrombosis within arteries
HUS is therefore the triad o f microangiopathic haemolysis, thrombocytopenia and
acute renal injury.
The sy ndrome o f inappropriate anti-diuretic hormone (C) ( SI ADIT) is the result o f
inappropriately elevated levels o f ADH causing the acute retention of water. As a
result, there is hyponatraemia and reduced serum osmolality which is not present in
this patient In severe cases, patients can become very' agitated and at risk of
seizures.
HIV nephropathy fDi is a common occurrence in HIV sufferers and can be due to
direct HIV infection. Features include nephritic range proteinuria, large kidneys on
ultrasound scan and typically collapsing focal segmental glomerulosclerosis on
renal biopsy. In this acute case, the impact o f renal hypoperfusion is the likely
cause o f the patient's presentation. Although this is an example of acute renal
failure (E), the specific cause is the most appropriate answer.

Miscellaneous

311. Answer D : Decreasing calcium intake t


Decreasing calcium intake may paradoxically increase the risk o f calcium oxalate
slonc lormaiion because there will be less intestinal calcium available to bind up
oxalate, which may result in increased oxalate absorption and increased risk of
calcium oxalate stone formation. All the other measures listed should be
recommended lo palicnls with known calcium oxalate stone disease to minimize
the risk ol recurrence

312. I he a u sm i is A: Type I (distal) K I A is u disorder that affects adults und is


considered a larmlml disorder in children. Sporadic cases may he primary
(especially in women) or secondary ( e g , to an autoimmune disease such as
SjOgren's syndrome; medications including amphotericin B or lithium therapy
kidney transplantation; nephrocalcinosis, renal medullary sponge kidney; chronic
renal obstruction). Familial eases may be autosomal dominant and are often
associated with hypercalciuria. In type I RTA, the urine pH is never <5.5
Type II (proximal) RTA is associated with several inherited diseases (eg.,
Fancom’s syndrome, fructose intolerance, W ilson’s disease, Lowe’s syndrome),
multiple myeloma, vitamin D deficiency, and chrome hypocalcem ia with
secondary hyperparathyroidism. It may occur after renal transplant exposure to
heavy metals, and after treatment with certain medications, including
acetazolamidc, sulfonamides, tetracycline, and streptozocin.
In type II RTA, the ability o f the proximal tubules to reabsorb H C O 3 - is
decreased, so that urine pH is >7 at normal levels o f plasma HC03 but 52 may
be <5.5 at low levels of plasma H C03 -. Type III RTA is a combination o f types I
and II and is seldom seen.
Type IV RTA is a condition associated with mild renal insufficiency in adults with
diabetes mellitus, human immunodeficiency virus (HIV) nephropathy, or
interstitial renal damage (SLE, obstructive uropathy, sickle cell disease). It may
also be produced by drugs that interfere with the renin-aldosterone system (c.g.,
NSAlDs, angiotensin-converting enzyme (ACE) inhibitors, potassium -sparing
diuretics, trimethoprim) Aldosterone deficiency or unresponsiveness o f the distal
tubule to aldosterone results in type IV RTA This reduces potassium excretion,
causing hyperkalemia, which reduces ammonia production and acid excretion by
the kidney. Urine pH is usually normal.
Types I and II RTA are associated with chronic metabolic acidosis, m ild volume
loss, and hypokalemia. Hypokalemia may lead to m uscle weakness, hyporeflexia,
and paralysis. Type I RTA has decreased citrate excretion in the urine, increased
mobilization o f bone calcium, and hypercalciuria, w hich results in osteopenia, bone
pain, and kidney stones or nephrocalcinosis. Renal parenchymal d am ag e and
chronic renal failure may develop. Type IV RTA is usually asym ptom atic with
only mild acidosis, but cardiac arrhythmias or paralysis may develop if
hyperkalemia is extreme. Sodium bicarbonate relieves symptoms and prevents or
stabilizes renal failure and bone disease. In adults with type I R T A , sodium
bicarbonate eliminates acidosis and reduces the occurrence o f kidney stones. In
type II RTA, the plasma H C03
- cannot be restored to the normal range. H C 03 - replacement should exceed the
acid load o f the diet. Additional H C 03-replacem ent increases potassium
bicarbonate losses in the urine. Citric acid/sodium citrate (Bicilra) or potassium
citrate/cilnc acid (Polyeitra-K) can be substituted for sodium bicarbonate and may
be better tolerated. Potassium supplements may be required in patients who
become hypokalemic when given sodium bicarbonate, but are not recom m ended in
patients with normal or high serum potassium levels. In type IV RTA, the
hyperkalemia is treated with fluid administration and potassium -depleting diuretics.
A few patients may need niinerulocorticoid replacem ent therapy
^13. The answ er is U: A positive unne dipstick for hemoglobin results from tree
hemoglobin or myoglobin in the urine. Free hemoglobin appears in the urine when
there is intravascular hemolysis. Once haptoglobin becomes saturated, free
hemoglobin spills info the urine. A common cause is a transfusion reaction
Myoglobinuria is associated with rhabdom>oiysis and occurs when there is
significant muscle injury with the release of myoglobin into the bloodstream
Causes include electrical shock or massive muscle trauma. Other causes ma>
include toxin exposures, metabolic disorders, inflammatory conditions, and
infection. Myoglobinuria causes a positive unne test for blood in the absence of
urinary erythrocytes.

314. The answ er is B: Renal blood flow is autoregulated so to sustain pressure


within the glomerulus, by angiotensin II—related vasoconstriction. ACE inhibitors
impair renal autoregulator)' function, resulting m a decreased glomerular filtration
rate and can cause acute renal injur)'. This is usual 1)’ reversible if recognized and
the offending medication stopped. NSAlDs can exert a similar effect, but they can
also cause glomerulonephritis and interstitial nephritis. Statins, haloperidol, and
drugs of abuse (cocaine, heroin) can cause rhabdomvolysis with the release of
myoglobin, which causes acute renal injury. Thrombotic microangiopathy is a rare
mechanism o f injury to the kidney and may be caused by clopidogrel, quinine, or
certain chemotherapeutic agents.

315, The an sw e r is C: Glomerulonephritis and lupus nephritis are both


autoimmune in origin and aside from supportive measures and renoprotection with
drugs including ACE inhibitors, immunosuppression is commonly .used and works
with varying effect.
Svstcmic sclerosis can affect the afferent arterioles w ith fibrinoid thickening and
vessel narrowing, which can com e on rapidly ("scleroderma renal crisis”).
Immunosuppression is often unhelpful, whereas ACE inhibitors are o f immense
benefit and can halt progression o f loss of renal function and sometimes even
partially reverse it.
In severe renal artery stenosis. ACE inhibitors can further reduce or abolish
glomerular filtration. This is because angiotensin normally increases glomerular
capillary' perfusion pressure, and so when ACE inhibitors reduce this ability, with a
background o f already grossly reduced renal perfusion, the glomerular filtration
rate will fall. For this reason, the response to ACE inhibitors should be monitored
in patients with renal failure (and especially so in patients with known peripheral
arterial disease), and if renal function worsens, various imaging techniques can be
used to look for renal artery stenosis (such as Doppler ultrasound and renal
arteriography). Overall, whilst ACE inhibitors should be avoided where possible in
patients with renal artery stenosis, they arc beneficial in protecting the kidneys of
renal failure patients, and under specialist supervision can be started at a reduced
dose in patients with renal artery stenosis, especially if there is hypertension that
cannot be controlled by other means.
.1 1 6 . I lie a rower i\ A: Cerebral a n e u r y s m s a r e associated uiih p o l y c y s t i c k i d n e y
disease, and thus the risk o f subarachnoid haemorrhage is increased. For this
reason, blood pressure should be monitored carefully and hypertension treated
swiftly. Hepatic cyst3 and pancreatic cysts are a s s o c i a t e d , but only occasionally can
be large and symptomatic (dull ache, acute pain caused by rupture, or obstructive
symptoms such as obstructive jaundice). Mitral valve prolapse and mitral
regurgitation are a l s o l i n k e d . B l a d d e r d i v e r t i c u l i a r c usually secondary to urinary
Outflow obstruction and can become infected due to stagnant unne or obstruction
of their outlet. They are not associated with polycystic kidney disease.

317. The answ er is D: T h e mainstay of treatment in diabetic patients with new


onset proteinuria is to aggressively control blood pressure, ideally below 130/80
mmHg. A CE inhibitors (D) are therefore first-line therapy, angiotensin receptor
blockers can also be used. Oral hypogjycaemic (A) agents should be avoided since
they are excreted by the renal system. ACE il antagonists (B) are secondline
treatment if there arc contraindications to ACE inhibitor use. In patients with
persistent proteinuria due to diabetic nephropathy, ACE anck A R B may be
combined together provided the patient is monitored regularly under specialist care.
Renal dialysis (E) is only needed if the patient has progressive end stage renal
failure. Cholesterol-lowering therapy (C) would be useful in low ering patient’s
cardiovascular risk factor but does not help in improving impaired renal function.

318. The an sw er is D: Thiazides and furosemide lead to metabolic alkalosis. There


are two components to the development of metabolic alkalosis: volum e depletion
and electrolyte imbalance; specifi cally hypochloremia and hypokalem ia. Volume
contraction leads to increased sodium reabsorption and bicarbonate retention. The
diuretic-induced hypochloremia and hypokalemia lead to persistence o f the
alkalosis because the hypokalemia causes hydrogen to be exchanged fo r sodium
rather than potassium at the distal convoluted tubule.
Answer A is incorrect. Neither potassium-sparing diuretics uor acetazolam ide
cause metabolic alkalosis. Potassium-sparing diuretics cause m etabolic acidosis by
inlubiting sodiunihydrogen exchange channels, and acetazolam ideprom otes the
loss of bicarbonate in the urine, causing metabolic acidosis.
Answer B is incorrect. Acetazolamide inhibits the enzyme carbonic anhydrase,
which is important in the rcabsorption of sodium , bicarbonate, and chloride at the
proximal tubule. Since it promotes the loss o f bicarbonate in the urine, it tends to
cause metabolic acidosis.
Answer C is incorrect. The potassium-sparing diuretics, such as spironolactone,
inhibit aldosterone- sensitive sodium channels that excrete hydrogen or potassium
in exchange for sodium. Inhibition o f these channels may lead to hyperkalem ia and
metabolic acidosis.
Answer E is incorrect Thiazides do cause metabolic alkalosis by causing volume
depletion, hypochloremia, and hypokalemia. However, acetazolamide promotes the
loss of bicarbonate in the urine, causing metabolic acidosis.
VnaWVI I iv uwviwvi l twa \\K'\ vl>* *auw uu'talvlu alkahwt* hv tvui>tiig wluiiu'
\ U ^ u v i* tv%isv M\»n iiii ,i . and h \ iv kah ’uua llo>\\'Wi, (sq.iviHuu N|s%»»nn n
uwUlvliv u I'V inlul'ilm*; whom l » \ o w t u u ^ o vhamu'K

MM. I hi* nil«wn l» K: Uvu.il c Horn A IN o u lih m .*'% |Mlii'i»H with nontiv
|w|Yitl\ vl.UlWK'' \nl'l»*l llvd IlSjVlU’OMOIl l\IM iMUM* small NrSM’l ol'slUK'llOU, With
poliK nm w viwl.uloitlis ol imIiu Ip ImiIui nilau's tm«l lihnitoitl ucuosis ol iiMoimt
ul>‘lm lo hill! glonicillliil i<i|Mll<HY UiH

' 20, I'hv answer h A; llu* iim onlv ol lulmlomtcisiiti.il nt.*|>hrilis (A) is due to
ding hypei sensitivity unctions. most commonly penicillin 01 N S A lD s which .ire
commonly given, Patients typically present with lever, skin rashes anil may also
have painful joints. Mood results will alien have raised eosinophils. Eosinophils
arc involved in allergic responses, such as asthma and drugs, parasitic tnlcction and
ulsn tissue inilummntion.
Renal failure (U) is the sudden loss ol' renal function which in the acute phase is
reversible, plusma urea and creatinine typically increase due to the loss o f filtering
function o f the kidney ond patients tend to be oliguric rather than polyunc.
In diabetes (C), although patients would tend to visit the toilet more due to
hyperglycacmia causing an osmotic diuresis, other important features would
include weight loss, polydipsia and the presence o f glucose and possibly ketones on
urine dipstick.
A UTI (D) is characterized by features that include dysuna, elevated white cell
count and raised leukocytes and nitriles in the urine.
Reiter’s disease (E) is a sterile synovitis that typically follows an infection and
involves the classical triad o f urethritis, arthritis and conjunctivitis.

321. T he answ er is B: This patient has the classic triad of proteinuria, low serum
albumin and oedema that occurs in the nephrotic syndrome. This can occur due to a
number o f disease processes such as diabetes and SLE, as well as those specific to
the kidney, including minimal-change nephropathy and focal segmental
glomerulosclerosis. First-line management should include dietary measures to
restrict sodium intake and a diuretic (B). Polenta] diuretics include furosemide
which is often required to control any associated severe oedema.
High protein diets (A) do not have any benefit to the management of nephritic
syndrome, a normal low salt diet should be encouraged. Albumin infusion can be
used as adjuncts in patients who are resistant to diuretic therapy but never in
isolation as they have transient beneficial effects.
Bed rest (E) should also be discouraged in patients since coagulation factors, for
example antithrombin, are also lost as part o f the proteinuria creating a
hypercoagulable state, patients arc therefore at risk o f thromboembolism, including
renal vein thrombosis.
Therefore, prophylactic aniicoagulation (C) is desirable to protect against
bypercoagulation and should always be considered, Angiotensin-converting
enzyme (A C E ) inhibitors (D) protect against proteinuria by reducing the filtration
pressure upon th e glomerular capillaries.

322. The a n s w e r is D: The mainstay o f treatment in diabetic patients w ith new


onset proteinuria is to aggressively control blood pressure, ideally below 130/80
mmHg. A C E inhibitors (D) are therefore first-line therapy, angiotensin receptor
blockers can also be used. Oral hypoglycaemic (A) agents should be avoided since
they are excreted by the renal system. ACE II antagonists (B) are secondline
treatment i f th ere are contraindications to ACE inhibitor use. In p atients with
persistent proteinuria due to diabetic nephropathy, ACE and A R B m ay be
combined to g eth er provided the patient is monitored regularly under specialist care.
Renal dialysis (E) is only needed if the patient has progressive end stage renal
failure. Cholesterol-lowering therapy (C) would be useful in lowering patient’s
cardiovascular risk factor but does not help in improving impaired renal function.

323. T he a n s w e r is C: Acute hvperuricaemic nephropathy (Cl is a common


finding in p atien ts suffering from hyperuricaemia. This is a compion occurrence in
patients w ith increased cell turnover, such as myeloproliferative disorders or
following chemotherapy. Uric acid crystallizes w ithin the renal system causing
obstructions w hich can manifest as flank pain, oliguria, hypertension, oedem a and
uraemic sym ptom s.
Analgesic nephropathy (A) is usually due to chronic NSAED intake causing
papillary necrosis and tubulointerstitial nephritis. Presentation can include anaemia,
urinary tract infections and haematuria.
Acute tubulointerstitial nephritis (D) is a drug hypersensitivity reaction, usually due
to penicillin o r NSAID medication. Patients typically present with fever, skin
rashes and jo in t pain alongside an eosinophilia.
Renal infarction (B1 can be a difficult diagnosis to make due to the broad clinical
presentation th at can result, dependent on the degree o f ischaemia and necrosis that
may occur. In moderate to severe arterial occlusion, patients may present w ith pain
affecting the back, abdomen or flanks. Patients will often have several
cardiovascular risk factors that predispose them to thromboembolism, such as atrial
fibrillation, clotting abnormalities, etc. In this patient, polycythaemia alone is
unlikely to h av e caused a severe acute renal infarction in both kidneys and, given
the recent com mencement o f chemotherapy, an acute hyperuricaemic nephropathy
is more likely. Another potential differential is renal vein thrombosis.
Chronic renal failure (El occurs as a long-standing illness causing significant
reduction in renal function, usually in an insidious m anner rather th an th e abrupt
situation in this case.

324. T he a n s w e r is A: Alport syndrome fAl is caused by a genetic defect in type


IV collagen synthesis causing the triad o f herediatary nephritis, sensorineural
deafness and ocular abnormalities which can include cataracts and m acular retinal
flecks. Renal abnormalities a r c progressive in such patients and include proteinuria,
haematuria and eventually renal failure.
Thin basement membrane pgahropathv or benign familial haematuria (D) is a
common cause of asymptomatic haematuria. Apart born glomerular basement
membrane thinning, there are no other associated abnormalities and patients haw
an excellent prognosis. IgA nephropathy (D) is the most common cause of
glomerulonephritis and one of the most common causes for asymptomatic
baemauria. Glomerular attacks occur episodically and. during these, haematuria
presents. Features such as retinal flecks are not present.
Wolfram syndrome (Q is a rare genetic disease that causes diabetes insipidus,
diabetes mellitus, optic atrophy and deafness. This is not likely given the absence
of glucose on urine dipstick
Patients suffering from D ow n's syndrome (El have a range of abnormalities and
are often recognized from their characteristic facial appearance. The kidney,
however, tends to be spared in such patients Brushfield spots may be mistaken for
retinal flecks in such patients and u tile sensorineural deafness can occur in
Down's syndrome, this is not congenital as occurs in Alport's syndrome

You might also like